Jump to content

Wikipedia:Reference desk/Science

From Wikipedia, the free encyclopedia

This is an old revision of this page, as edited by 98.26.182.245 (talk) at 17:53, 10 August 2008 (→‎The Matrix: re). The present address (URL) is a permanent link to this revision, which may differ significantly from the current revision.

Welcome to the science section
of the Wikipedia reference desk.
Select a section:
Want a faster answer?

Main page: Help searching Wikipedia

   

How can I get my question answered?

  • Select the section of the desk that best fits the general topic of your question (see the navigation column to the right).
  • Post your question to only one section, providing a short header that gives the topic of your question.
  • Type '~~~~' (that is, four tilde characters) at the end – this signs and dates your contribution so we know who wrote what and when.
  • Don't post personal contact information – it will be removed. Any answers will be provided here.
  • Please be as specific as possible, and include all relevant context – the usefulness of answers may depend on the context.
  • Note:
    • We don't answer (and may remove) questions that require medical diagnosis or legal advice.
    • We don't answer requests for opinions, predictions or debate.
    • We don't do your homework for you, though we'll help you past the stuck point.
    • We don't conduct original research or provide a free source of ideas, but we'll help you find information you need.



How do I answer a question?

Main page: Wikipedia:Reference desk/Guidelines

  • The best answers address the question directly, and back up facts with wikilinks and links to sources. Do not edit others' comments and do not give any medical or legal advice.
See also:


August 4

Heparin lock

I've been undergoing minor testing in the hospital for several days, during which I've had a Heparin Lock in my arm. I've gone looking online for information about it, which has led me to conclude that it's potentially notable, even though there's no article about it. Does anyone know much about such a device? The medical staff have answered all my medical questions, so I'm not asking for medical advice for my specific situation: I'm simply more interested in information about the device in general. I've learned somewhat about it from the websites I've looked at, but as a distinctly non-medical person, I don't understand too much about it, except as it applies to my testing. Could someone explain the basics of this device: what it was originally meant for, what it's used for (in general today), information like that? Nyttend (talk) 01:32, 4 August 2008 (UTC)[reply]

A heplock provides access to the bloodstream for easy injection or sampling. Heparin is an anticoagulant, and is used to prevent the port from clotting. The alternatives would be to poke you with a new needle each time or to require you to have an IV drip to keep the line open. DMacks (talk) 05:17, 4 August 2008 (UTC)[reply]
It looks like there's a passing mention in port (medical), but it could use some beefing up. It'd be dandy if someone with suitable expertise could have a look, and perhaps create appropriate redirects. TenOfAllTrades(talk) 13:46, 4 August 2008 (UTC)[reply]

five kindoms

tigar which filam? —Preceding unsigned comment added by 121.247.108.75 (talk) 02:43, 4 August 2008 (UTC)[reply]

Your question is so full of typos and missing grammar that it is not possible to answer. It appears that you are asking which Five Kingdoms movie has the Tiger - but that is only a guess. Try spelling actual words and using complete grammar and someone will probably be able to give you an answer. -- kainaw 02:53, 4 August 2008 (UTC)[reply]
You can learn which phylum the tiger is in by looking at: Tiger. Dragons flight (talk) 02:55, 4 August 2008 (UTC)[reply]
Look at the box on the right. You don't need to read the article. --Bowlhover (talk) 03:42, 4 August 2008 (UTC)[reply]
BTW, the Five kingdoms model, while still taught in some schools is generally considered a rather poor model nowadays. One of the most obvious flaws is it puts Archaea and Bacteria together into Monera. Nil Einne (talk) 13:27, 4 August 2008 (UTC)[reply]

chordee

can steroid help cure chordee? —Preceding unsigned comment added by 116.71.150.79 (talk) 06:52, 4 August 2008 (UTC)[reply]

If you look at chordee it says treatment is best carried out in infancy by surgical means. If there was a treatment involving steroids I guess that would be the preferred treatment. Richard Avery (talk) 07:38, 4 August 2008 (UTC)[reply]

A lot of quasi-related questions

Sorry, I know these questions are going to be odd, but I couldn't find the answers on the net, so please bear with me. First, when we submerge something in water, there will be an upward force (buoyancy) acting on the object, right. The magnitude of this force (as per the wikipedia article) will equal the weight of the water displace, but why, intuitively, should this be? Secondly, an object that floats at the surface will also float at the bottom of the ocean, but how is this possible considering that there's the weight of the entire ocean above it that it has to contend against? The only explanation for this that I can think of is that water is a liquid, and so because the object can pass through it, it's weight is meaningless. But if this was the case, why do people keep mentioning the weight of the atmosphere above us as important (i.e. puts stress on objects, causes pressure, etc.) And why does air pressure drop as we travel farther from the surface of the earth is the weight of the atmosphere above us isn't significant? Finally, is bouyance responsible for convection (why hot air rises)? I would think so (hot air expands, thus less dense), but I wasn't sure. Thanks in advance! —Preceding unsigned comment added by 76.68.246.7 (talk) 09:16, 4 August 2008 (UTC)[reply]

By pushing an object into water, you displace water, and raise the water level slightly, so you're work done to push the object into the water, is stored as gravitational potential energy. The water level, acted on by gravity, is pulled down, and this forces the object up, providing that it is lighter than the water it displaces, this is the origin of buoyancy. Secondly yes, the water above an object does provide pressure in the same way air above us does, but it does not pin us down does it? gravity does that, and in the same respect, objects underwater are not 'pinned down' by the weight of the water above them.An object that floats placed at a greater depth will in fact experience a greater up thrust, as the higher pressure means the weight of the water it is displacing is greater. Philc 0780 11:28, 4 August 2008 (UTC)
A correction - The weight of the water above a submerged object is realized as pressure, which acts equally in all directions on an object, thus canceling out, so the only relevant forces are the weight of the object and the buoyant force. Also, the thrust does not increase with depth because the weight of displaced water does not increase with depth, because water is incompressible. To answer the other questions, you can say air pressure drops as you climb in altitude for the same reason water pressure drops as you approach the surface - just less weight piled on top - but air pressure also drops because its weight decreases with height due to the declining pull of Earth's gravity (relevant well above the troposphere). The weight of the atmosphere is certainly significant as atmospheric pressure. Buoyancy, in a sense, is responsible for convection, but since convection is a well-defined statistical mechanical effect, it might be more accurate to say that convection is responsible for buoyancy (but I'm not sure on this). SamuelRiv (talk) 13:54, 4 August 2008 (UTC)[reply]
I think this is splitting hairs, but yes, I would prefer to say convection causes buoyancy. Buoyancy is the simple approximation of the net fluid motion effects acting as a single equivalent upward force; it also assumes the object is sufficiently small that there is not a significant pressure gradient between its top and bottom parts, etc. The more rigorous treatment yields a similar effect via much more horrible integral equations. Nimur (talk) 15:50, 4 August 2008 (UTC)[reply]
Thanks a lot guys, really. But sorry, just one last thing, why is the pressure increase caused by the weight of water/air above us distributed evenly?
Because in the case of a liquid or gas, the molecules are free to move in all directions. They are under pressure (the weight of all the molecules above them) so they will push whichever way they can go to escape the pressure. If they can get a little more room by pushing your body inwards a little bit, that's what they'll do. They're pushing back just as hard on the molecules above them, beside them, and below them. (That is the highly unscientific explanation, but it works for me :) Franamax (talk) 02:14, 5 August 2008 (UTC)[reply]
"it also assumes the object is sufficiently small that there is not a significant pressure gradient between its top and bottom parts, etc" Buoyancy is caused by the pressure gradient between an object's top and bottom. The pressure increases as depth increases, so it pushes on an object's top with less force than on the object's bottom. The buoyancy article elegantly shows why this pressure difference is equal to the object's weight in water. A volume of water of any shape is not moving, so water must be pushing it upwards with a force equal to the water's weight. When the water is replaced by a solid, this force remains.
Convection is caused by buoyancy, which is in this case the pressure difference between a mass of air's "top" and "bottom". --Bowlhover (talk) 07:58, 9 August 2008 (UTC)[reply]

I was washing dishes...

And I noticed two things happen that I can't fully explain. First, if I were to plunge a glass into the water (with the open side facing it), the inside won't get wet. I'm not sure why this, and my only explanation is that the air pressure inside the glass is stopping it from entering (and if the water level were to rise, the pressure would increase and stop it eventually). Second, if I quickely remove the drain, a huge bubble will erupt. I not more or less why, but the details seem to be lacking. The water enters some sort of secondary container, and the air needs to escape. But what gives the column of air so much force? Thanks. —Preceding unsigned comment added by 76.68.246.7 (talk) 09:21, 4 August 2008 (UTC)[reply]

In response to the first, the air in the glass takes up the volume of the glass. We'd normally say that the glass is empty but it's not. It's full of air. The air would rise out of the glass if there were a hole in the bottom (now the top) of the glass. This would of course cause the glass to fill with water. You can demonstrate this with a straw though it may be hard to see. Plug one end of the straw with your finger and plunge it into water the same way as your glass into the sink. Then remove your finger. The water will push the air up the straw and out the top. Dismas|(talk) 10:05, 4 August 2008 (UTC)[reply]
To answer your second question, the air that is in the drain beneath the plug has to go somewhere when the water in the sink tries to go down. It can either follow the pipe or it can attempt to rise through the water. Water will attempt to fall because gravity is pulling it into the drain, forcing the air to attempt to rise because it will be displaced by the falling water. This the bubble you see. The reason the air doesn't just move down the drain is because the water falling into the sink does not form a solid uniform wall to prevent the air from escaping. EagleFalconn (talk) 15:38, 4 August 2008 (UTC)[reply]
This is more or less what I thought, but when I say huge, I really mean it i.e. water went everywhere.
If you release it quickly, there is a large amount of air that is rising. It has a bit of momentum when it reaches the water's surface and therefore lots of water gets spritzed everywhere. Dismas|(talk) 20:29, 4 August 2008 (UTC)[reply]
If you ever go SCUBA diving, you can take this experience to a whole new level. Just under the surface the water level in the up-ended glass will be near the lip. As you descend and pressure builds the water will force it's way further up into the glass. When you hit 33 feet under, the water will be exactly (by volume) half-way up the glass. At 66 feet it will be 3/4 and at 99feet it would be at 7/8. As you ascend, it will slowly "empty" in exactly the same manner. ---J.S (T/C/WRE) 23:44, 4 August 2008 (UTC)[reply]

Drinking water vs. pouring it over your head to cool down

If you had a small amount of room-temperature water, say 30 cL, on a hot, dry summer day (say 28 °C), would it help cool you down more/faster to drink the water, or to pour it over your head? What if it was hot or cold water? Or what if it was a humid day? --58.37.187.141 (talk) 10:11, 4 August 2008 (UTC)[reply]

I don't expect drinking the water will actually help to cool you down directly much, but it will help recover a small amount of the lost fluids from sweating. It doesn't matter much what the temperature of the water is (just don't burn your mouth!). Pouring it over you, provided the water is at a lower temperature then your surface temperature is obviously going to cool you down somewhat but it won't help you with the water lose due to sweating and given the small amount, isn't going to help that much. Obviously the colder the water, the more effective. However particularly if you have fairly cold water, I would suggest pouring it on a cloth and holding it over your head (or wherever you want to cool down) will be much more effective. Also, while 28 °C isn't exactly an extremely hot day whatever the case, you may feel there is greater benefit in humid conditions to using the water to cool you down as your sweating is less effective Nil Einne (talk) 13:15, 4 August 2008 (UTC)[reply]
If you drink it, 100% of the temp of the water will go into changing the temp of your body, while if you pour it on yourself, only a portion will, and the rest will go into changing the temp of the air. However, there is no evaporative cooling if you drink it, while there is if you pour it on yourself. Evaporative cooling works best on hot, dry, windy days. If the water is hotter than the air and you, I would expect it would be best to leave it to cool to the air temp before using it for cooling. Also, if you are already sweaty, pouring water on yourself won't help much, as you already have the benefit of evaporative cooling from the sweat. StuRat (talk) 13:23, 4 August 2008 (UTC)[reply]
I've heard that drinking 500 ml cold water will make your body burn 100 kcal to get back at its normal temperature. Regardless of the correctness of it, it implies that any cooling of the body is useless in itself unless you have elevated temperature because cooling when having normal temperature will be reversed by your body. So, the correct answer would be; it depends on the body's temperature. --Ayacop (talk) 13:32, 4 August 2008 (UTC)[reply]
The math: 1 kcal raises a L of water by 1C. 0.5L of cold water raised by 35C (that's from near-freezing to body temp) = 17.5kcal.
Whether the water is cold or warm will certainly affect the comfort of drinking or wearing it. If it's too warm, it could exacerbate the problem of overheating...humidity will reduce the ability of evaporative cooling to effectively remove heat from the body by sweating or evaporating doused water.
Furthermore...one may assume that drinking the water allows one to capture 100% of the coldness and it will be used for further cooling as sweat. Pouring the water on one's self will waste some of it as it falls off the body. In general, drinking the water would be more effective as a cooling mechanism, and to stay properly hydrated. — Scientizzle 15:59, 4 August 2008 (UTC)[reply]
Extra drinking water will only lead to extra sweating if it's hot and you were in a dehydrated state to beign with. Otherwise, water you consume will just be eliminated as urine. StuRat (talk) 20:21, 4 August 2008 (UTC)[reply]
Let's do the math a bit differently. Imagine that you drink 1% of your body mass as water which is 60°F cooler than your body temp. That would result in lowering your body temp by about 60/100 or 0.6°F. That might not seem like much, but the body temp is very closely regulated, and that can be the diff between sweating profusely and being quite comfortable. (And, of course, if that takes your body temp too low, your body would react and burn some calories to bring it back up.) StuRat (talk) 20:28, 4 August 2008 (UTC)[reply]
Intuitively, it seems to me that if you only have a small amount of water you should drink it, assuming it is cooler than your body temperature. As noted above, this way you capture 100% of the coldness, and it helps supply your body with water for sweating. If you have plenty of water, you should drink enough to become well hydrated, and then pour some over yourself.
Partly, this question is hard because you've asked the wrong question. Body temperature is tightly controlled, so unless you are experiencing hyperthermia you can't actually "cool yourself down" with water. What you really mean is "which will make you feel cooler?" There, the answer is less clear, because it involves questions about how the rate of sweating and the sensation of being cool depend on your state of hydration.--Srleffler (talk) 02:46, 5 August 2008 (UTC)[reply]
Well, body temp is controlled, but it's not constant, it varies a bit. And even a few tenths of a degree in body temp is what makes the diff between you feeling hot or cold, so actually the question is dead on, as lowering body temp (a tiny bit) will also make you feel cool. StuRat (talk) 03:20, 5 August 2008 (UTC)[reply]
See [1] for previous responses to the same question.
The rate of cooling determines apparent temperature, not the amount of heat removed. Although drinking water removes more heat than pouring it on the skin, the latter feels much more effective in the short term because a thin film of water heats and evaporates very quickly. --Bowlhover (talk) 07:39, 5 August 2008 (UTC)[reply]
But, as already stated, pouring water on yourself will only help by evaporative cooling if it's hot and dry out, and yet you aren't already sweating, which is an unusual combo. One reason for this might be dehydration, in which case drinking the water becomes more important. Also, if you lower your temp too quickly then the body will react by generating heat, via shivering and other means. So, lowering body temp steadily may be the best way to go, especially if the effect lasts longer. StuRat (talk) 17:15, 6 August 2008 (UTC)[reply]
Evaporative cooling helps whenever the relative humidity is less than 100%. Poured water cools the body via conduction while itself heating up, and after reaching body temperature, the water would cool the body as sweat would do. As for reflexes, shivering and other reactions to cooling are trigged when the core body temperature drops; a drop in skin temperature would not decrease the core temperature because the body is already generating too much heat. --Bowlhover (talk) 11:08, 7 August 2008 (UTC)[reply]
Yes, evaporative cooling helps when less than 100% relative humdity, but adding water won't help in that way if the person was already completely coated with sweat. And cooling from conduction is never as effective as if the person drinks the water, as a portion of the coolness of the water goes into cooling the air, and much of the water drops off the person before it can cool anything down. I don't agree on shivering, however, as people shiver instantly when exposed to something cold, like diving into cold water, long before the core body temp could have had time to drop significantly. StuRat (talk) 14:58, 7 August 2008 (UTC)[reply]

Existance of God

Is there some connection between finding life on Mars and the existence of God? Does finding life on Mars refute the existence of God or prove it? —Preceding unsigned comment added by 71.100.5.89 (talk) 12:23, 4 August 2008 (UTC)[reply]

No. Algebraist 12:24, 4 August 2008 (UTC)[reply]
At worst, it would imply that the Judeo-Christian account of the creation of life was leaving out a few things. (But it's not like that was exactly on the line from a scientific point of view.) --98.217.8.46 (talk) 12:26, 4 August 2008 (UTC)[reply]
If you carefully read the Bible's account of the creation of extraterrestrial bodies, you'll notice that only "greater light", "lesser light", and "stars" are mentioned as being created: "sun" and "moon" are not used, and nothing is said specifically about the planets at all. I doubt anyone believes that this account includes everything. Nyttend (talk) 13:44, 4 August 2008 (UTC)[reply]
The Bible was written from the understanding of the universe they had at that time. Similar events occur in science where completely new ways of thinking or seeing things develop. These are called paradigm shifts. --Russoc4 (talk) 13:47, 4 August 2008 (UTC)[reply]
Neither. If/when we start discovering life on other planets, for the religious it will simply be a matter of saying "well, God created the entire universe and *all* the life in it". As far as I am aware, the Christian religion (I'll assume for the sake of discussion that you are referring to this) has never claimed that their God is only the god of *our* planet. Nothing will really change at all. It will still be impossible to prove or disprove the existence of an 'entity' which presumably would exist (if he/she/it exists) somewhere beyond/outside the universe as we know it. --Kurt Shaped Box (talk) 14:24, 4 August 2008 (UTC)[reply]
Quite the opposite, really. Christians believe God created everything, not just the stuff on Earth. —CycloneNimrodTalk? 16:01, 4 August 2008 (UTC)[reply]
That was the point I was making in the first place. Or at least I was trying to... ;) --Kurt Shaped Box (talk) 22:59, 4 August 2008 (UTC)[reply]
The existence of God might be strengthened by similar life on different planets (i.e. similar species). Life in itself doesn't really say much, it could have formed the same way we think it did here. —CycloneNimrodTalk? 15:26, 4 August 2008 (UTC)[reply]
The discovery of intelligent extraterrestrial life with very similar religious beliefs to those of humans would certainly provoke heated debate. I don't know how much the discovery of (for example) a fish-shaped creature in an extraterrestrial ocean really would prove, other than that that a fish shape is a good shape for swimming in the ocean and that convergent evolution works. --Kurt Shaped Box (talk) 15:56, 4 August 2008 (UTC)[reply]

Whew... I have to say that is certainly a relief. I would call this consensus that the discovery of life on Mars would not have any impact on the question of whether God exists or not so there is no reason for believers to panic or for non-believers to put them down. —Preceding unsigned comment added by 71.100.7.91 (talk) 19:17, 4 August 2008 (UTC)[reply]

There are always the same reasons as before ;) —CycloneNimrodT@lk? 19:48, 4 August 2008 (UTC)[reply]
God of the gaps is an article generally related to this topic. It discusses the interaction of theism and science over the long-term. ---J.S (T/C/WRE) 23:30, 4 August 2008 (UTC)[reply]
The question of whether or not there is a God in the abstract sense cannot and will not be verified or disproved by science. That is fairly clear. The question of specific religious ideas though can vary—if religions make concrete claims about the natural order of the world, then they can (and do) easily overlap into the domain of science, and conflict can (and does) occur. But if things are discussed in abstract, non-testable, entirely "unnatural" (that is, not pertaining to operations of the natural world) ideas, then there is not even the possibility of conflict. --98.217.8.46 (talk) 03:00, 5 August 2008 (UTC)[reply]
The abstract and the concrete, however, are quickly proving themselves to be reliant upon each other and inseparable for that reason and being only different aspects of the same thing. For instance, I can have an abstract concept which a computer program can emulate such as logical equation reduction. Although an abstract concept the results can be implemented directly by computer hardware, just as people can think and believe a certain abstract thing and control their actions accordingly to comply with or to implement the abstraction they are thinking and believe. Conversely, they can adapt their abstract thoughts and beliefs to accommodate feedback they get from the concrete world just a computer sensors can collect information from the concrete world which software can use to modify or to update an abstraction. In other words if you belief that God did not create life on any other planet and life was discovered on another planet then you would have to update your abstract belief with the concrete information or suffer abstract invalidation. —Preceding unsigned comment added by 71.100.7.91 (talk) 07:40, 5 August 2008 (UTC)[reply]

Calculation of pipe dia. Formula Needed?

Flow of water at standard pressure of 3bar, Calculation of pipe dia. to be used? When water needs to be pumped for a distance of 3 kilometers, and a constant pressure of 3 bar is supplied what is the optimal diameter(mm) of pipe that can be used? What formula can be used to calculate the diameter of pipe to be used when the pressure ,substance (water) and distance are known. Can anyone provide me with information? Arjaa (talk) 12:53, 4 August 2008 (UTC)[reply]

There's some missing info:
1) What is the desired flow rate through the pipe ?
2) Is the pipe straight and level or some other configuration ?
3) Is the pressure at the far end important ? StuRat (talk) 13:28, 4 August 2008 (UTC)[reply]


If the pipe is horizontal, and without leaks or taps, there should be no loss of pressure, and the diameter will only affect the flow rate. You can calculate the flow rate from the exit-velocity; density times velocity times flow area will give you mass flux. Mass flow rate has some equations that may put you in the right direction. If you use a pump or something else to induce a pressure differential across the pipe, you can use that to calculate the fluid velocity. Nimur (talk) 16:52, 4 August 2008 (UTC)[reply]
Is your first statement actually true in a real pipe? I would have thought that in an actual pipe there would be a pressure drop with distance due to the drag of the fluid on the walls of the pipe. The pressure drop should decrease with increasing pipe diameter (since there is less wall per unit volume), and also should increase with increasing flow velocity.--Srleffler (talk) 02:58, 5 August 2008 (UTC)[reply]
I'd say it's a suitable approximation; it would really depend on the fluid velocity (a stationary fluid would exhibit no drag!) The discharge coefficient is the quantity which would measure such a loss, but that's not typically applied to pipes, it's usually applied to the entry or exit orifice. In a real pipe, that quantity should be very close to 1.0. Nimur (talk) 04:48, 5 August 2008 (UTC)[reply]

Medical appliances (?), early/mid-20th C.

This picture shows prostheses and similar devices taken from victims of the Auschwitz II (Birkenau) camp and exhibited in a museum on the grounds of the Auschwitz camp. I haven't succeeded in identifying these other objects and need to know how to describe or otherwise refer to them. Would "medical appliances" be appropriate, or is there a more suitable term you'd suggest? -- Thanks, Deborahjay (talk) 13:50, 4 August 2008 (UTC)[reply]

My understanding, based on many years as registered nurse in the UK is that 'prosthetic' refers to some sort of replacement of a body part, sometimes to restore function sometimes for aesthetic reasons. Examples would be a glass eye or false hand for aesthetic reasons and wooden leg as a functional prosthesis. There are nowadays many more much more complex prostheses available. A 'medical appliance' I have always understood to mean a contraption, device or gizmo to assist some failing part of the body such as a spinal support for someone with scoliosis, a metal sprung calliper for drop-foot, a hearing aid for deafness or the good ol' truss for a hernia, as three examples. I hope that helps. Richard Avery (talk) 14:23, 4 August 2008 (UTC)[reply]
"Medical appliance" is quite vague, and includes such things as catheters and stoma appliances. The picture appears to be of orthopaedic-type equipment. It would perhaps be appropriate to refer to them as prostheses and orthoses, or prosthetic and orthotic equipment. Gwinva (talk) 22:14, 4 August 2008 (UTC)[reply]
I have two hesitations in accepting this solution en bloc: (a) some may indeed be hernia trusses (not clear in the photo, but used in this population) which are not orthopedic, and (b) the readership is mainstream (including non-native speakers and "orthoses/orthotic" are probably unfamiliar. Is there another comprehensive term rather than "medical appliance"?-- Deborahjay (talk) 06:39, 5 August 2008 (UTC)[reply]
If you're writing for a general readership, then "appliance" might be misunderstood; most people probably imagine an appliance is something you plug in. (WP doesn't even have an article for "medical appliance", although appliance mentions the concept briefly.) How about "prosthetics and other medical equipment" or "medical/health care aids"? Gwinva (talk) 22:52, 6 August 2008 (UTC)[reply]

Post thunderstorm ionization

Is it true that after thunderstorms people and animals feel better or get a 'high' from the change in ions in the air? Is this ionization? —Preceding unsigned comment added by 72.171.0.146 (talk) 14:42, 4 August 2008 (UTC)[reply]

No. In fact, some evidence[2] suggests the opposite, due to ozone production from the lightning. You may want to read ozone, though. Nimur (talk) 15:54, 4 August 2008 (UTC)[reply]

Tree of life

Does the DNA of currently-living organisms provide enough information to construct a full phylogenetic tree back to the most recent common ancestor of all such life? Are we likely to have such a tree in the next 50 years? Thanks. --Sean 14:51, 4 August 2008 (UTC)[reply]

No. We can make educated guesses and probably create a pretty good approximation, but we will never have a complete tree free from error. Several problems prevent this. First, many ancestral species no longer exist, which means one has to estimate the ancestral relationships between modern species. This can be done fairly well for recent speciation events, but as one looks further back in time, these ancestral relationship become far more speculative and there isn't enough information in currently existing DNA to be definitive. Keep in mind, more than 99% of all species that have ever lived are extinct. Secondly, with bacteria (and very infrequently some higher species) lateral gene transfer violates the assumptions generally used to construct such trees. As one gets closer to the most recent common ancestor, the more likely it becomes that non-descendent mixing will make it impossible confidently define descendent relationships. Dragons flight (talk) 15:42, 4 August 2008 (UTC)[reply]
But you might be interested in Richard Dawkins' book The Ancestor's Tale. It follows human evolution back and discusses the splits (or, in that direction, merges) encountered. He describes about 35 or so common ancestors, and also makes clear how much is hypothesis and how much is fairly solid. I found it in an airport bookshop on sale before I ever heard of Dawkins, and I was hooked by the beautiful writing and clear but not oversimplified exposition. --Stephan Schulz (talk) 00:23, 5 August 2008 (UTC)[reply]
Some clues to ancient life may be found in meteorites on the moon, or floating in the asteroid belt. Some would be derived from the earth, particularly in it's early stages. It is possible that vacuum dried bacteria may still be present. This is not a whole tree, just another node preserved from the past. This sort of thing may be found in the next 50 years. Graeme Bartlett (talk) 00:46, 5 August 2008 (UTC)[reply]

Blood plasma

If you take blood and just centrifuge it you get blood plasma. Plasma still has the clotting factors in it (as opposed to blood serum). My question is will blood plasma clot or does clotting only work when the cells and or platelets are in there? ike9898 (talk) 16:09, 4 August 2008 (UTC)[reply]

Yes, you need the platelets (and some factor proteins, read coagulation) which are in the heavy part of the centrifuged blood. --Ayacop (talk) 17:59, 4 August 2008 (UTC)[reply]

Minor correction; If you simply centrifuge whole blood you'll end up with a pellet of cells at the bottom and serum on top; If you centrifuge blood treated with anticoagulant you'll have a pellet of cells with plasma on top. When you have a blood sample taken you may notice that different colour tubes are used. The colours indicate the additive in the tube. If you put blood into a blue (Lithium citrate) tube the coagulation cascade is inhibited such that after centrifuging the coagulation proteins are still present in solution and the yellow fluid on top of the pellet is Plasma(or strictly speaking Plasma+Artificial anticoagulant). Various diagnostic tests/assays test different blood components, hence the need for different tubes. [3]Od6600 (talk) 16:56, 6 August 2008 (UTC)[reply]

Help diagnose my ear deficiency!

This question has been removed. Per the reference desk guidelines, the reference desk is not an appropriate place to request medical, legal or other professional advice, including any kind of medical diagnosis, prognosis, or treatment recommendations. For such advice, please see a qualified professional. If you don't believe this is such a request, please explain what you meant to ask, either here or on the Reference Desk's talk page.
This question has been removed. Per the reference desk guidelines, the reference desk is not an appropriate place to request medical, legal or other professional advice, including any kind of medical diagnosis or prognosis, or treatment recommendations. For such advice, please see a qualified professional. If you don't believe this is such a request, please explain what you meant to ask, either here or on the Reference Desk's talk page. --~~~~

Earths core

This is a schematic diagram of earth's interior. Nimur (talk) 16:57, 4 August 2008 (UTC)[reply]

How do we know the core of the earth is molten? Has someone drilled right down. Is it possible there could be another planet at the center like in journey to the center of the earth? —Preceding unsigned comment added by 79.76.204.221 (talk) 16:47, 4 August 2008 (UTC)[reply]

This is what we use to determine the above layers. Nimur (talk) 16:57, 4 August 2008 (UTC)[reply]
We measure earthquakes and compare the data from many different places on the planet. Then we use that data to solve the wave equation to estimate what kind of material the wave traveled through. In some cases the waves appear to travel through solid rock, and other cases the wave appears to have traveled through liquid rock. Close data analysis lets us estimate the density, and thus the temperature and pressure, and other parameters. The deepest drilling humans have completed is probably deep petroleum exploration, which may go as far as 10 or 12 kilometers in some extreme cases; this does not even penetrate the crust. It is very unlikely there is another planet or anything like Journey to the Center of the Earth speculates about. Nimur (talk) 16:58, 4 August 2008 (UTC)[reply]
The Bertha Rogers well was the deepest petroleum exploratory well and reached 31,441 feet (9,583 m) encountering pressures approaching 25,000 psi (172,369 kPa). In 1974 the hole was abandoned after encountering molten sulpher that melted the drill pipe!
The Kola Superdeep Borehole made it to 12,262 metres (40,230 ft or 7.62 mi) deep in 1989. Saintrain (talk) 23:01, 5 August 2008 (UTC)[reply]
(edit conflict) We know because earthquakes travel differently through the different layers, and refract at the boundaries. As for drilling, see Mantle_(geology)#Exploration. Note that that's for drilling to the mantle, not the outer core (the liquid layer). Anything in the center of the earth wouldn't fulfill the definition of a planet. If you're asking about having a gas or vacuum layer (so the next layer down would look like a planet), then no. It wouldn't conduct earthquakes as well as it does. If having a liquid layer with a solid layer under it counts, then it's correct. The outer core is liquid, and the inner core is solid. If you could survive the temperature and pressure, the inner core-outer core discontinuity would seem similar to a liquid covered planet. — DanielLC 17:16, 4 August 2008 (UTC)[reply]
But the inner core would be covered by an ocean of liquid metal, and that would be opaque and probably red-hot. It's not quite the typical "liquid-covered planet" you might envision. Nimur (talk) 17:43, 4 August 2008 (UTC)[reply]
What inner core made of? If iron why isnt it molten? —Preceding unsigned comment added by OpticalSphincter (talkcontribs) 17:48, 4 August 2008 (UTC)[reply]

Who said the core was molten? The core is solid. In fact most of the layers of the Earth are solid. The pressure would force any matter near the core into a solid. Of course if you were to bring that hot matter to the surface, it becomes a viscous liquid. ScienceApe (talk) 19:46, 4 August 2008 (UTC)[reply]

The outer core is liquid. Dragons flight (talk) 19:49, 4 August 2008 (UTC)[reply]
Actually, OpticalSphincter asked why the core isn't molten. It's made of mainly iron and nickel, and is solid because the pressure on it prevents molecules from moving freely. The Sun`s core is even more interesting; it comprises mainly hydrogen and helium at 15 million degrees Celsius, yet the density is 150 g/cm^3. --Bowlhover (talk) 19:52, 4 August 2008 (UTC)[reply]
How do we know that?
Pretty much the same way we know about the Earth's core, but rather than studying earthquakes, you study sunquakes. See Helioseismology. --Tango (talk) 03:41, 5 August 2008 (UTC)[reply]
Fun (aside) fact—in his memoirs, Edward Teller says that working with George Gamov on questions relating to the core of the earth gave him the idea that you could use a solid sphere of plutonium inside an implosion design nuclear weapon, that under the pressures of a few hundred megabars even an apparently dense solid could get a LOT denser than it would be at a single atmosphere. --98.217.8.46 (talk) 02:55, 5 August 2008 (UTC)[reply]

Galactic Center of the Milky Way

Hypothetically, if there was no cosmic dust and such in the way, how bright would our galactic center would be to us earthlings? If anything, how would this extra light or whatever, affect us on earth? --Anilmanohar (talk) 18:57, 4 August 2008 (UTC)[reply]

Well there should be a super massive black hole at the galatic core of our galaxy, and it's surrounded by an immense accretion disk. This disk should be very hot and larger than any other object in our galaxy, so it should be very bright. ScienceApe (talk) 19:52, 4 August 2008 (UTC)[reply]
Actually by itself the black hole, official known as Sagittarius A*, is very dim (as galactic nuclei go). Only about 1000 times the luminosity of the sun, or so. For comparison, the neutron star at the heart of the Crab Nebula and some individual stars, such as Rigel, are at least 10 times more intrinsically luminous than this black hole. Since the black hole is 27000 light years away (give or take), it would not be visible to the naked eye (even if you could distinguish it from all the stars clustered around the galactic center). As a mature black hole Sgr A* has already consumed most everything in its immediate vicinity and so now it accretes matter only very slowly. Young black holes, which have lots of matter to eat, can power active galactic nuclei which are among the brightest objects in the universe, but our black hole thankfully became quiet billions of years ago. Also, the bulk of the accretion disk is only about the size of a solar system. The black hole itself is not huge on astrophysical scales. Dragons flight (talk) 20:40, 4 August 2008 (UTC)[reply]
No not necessarily. I guess you are talking about the black hole at the center of our galaxy. But the it depends on which supermassive black hole we are talking about because they vary in size by quite a lot. The one in our galaxy is pretty small at only about 3.7 million solar masses. But Q0906+6930 is much larger at 16 billion solar masses and has a event horizon volume that's 1000 times our solar system. And that's just the event horizon. The accretion disk is much much larger. ScienceApe (talk) 03:28, 7 August 2008 (UTC)[reply]
(Edit conflict) Okay, let's work through this problem - it's an interesting one that requires a bunch of assumptions and formulas. First, Galactic Center gives us relevant information on the area immediately around the supermassive black hole, and it tells us that within 1 parsec (3.something light years) you "thousands of stars... most of them are old red main sequence stars." Great, so we can ignore most of them, because such stars are very very dim compared to the "more than 100 OB (blue giant)... stars that have been identified so far". We'll use Rigel as a model for this type of star, which is at 40,000 apparent solar luminosity. One more thing: this central parsec is 7600 parsecs from us, which, using trig, gives us 27 seconds of arc, which is about 1/60th the diameter of the moon in the sky, so roughly the angular size of a planet.
So now to crunch numbers to find apparent magnitude of this galactic center from Earth. We can add luminosities since it's just a measure of power and get about 4*10^6 solar luminosities, which is an absolute magnitude of -11.7, which is an apparent magnitude at 7600 parsecs of 2.7, which would make it one of the brighter lights in the sky. With all that done, it would probably be more relevant to get a number for the whole galactic bulge, since the combined luminosity could probably outshine the moon. But I'm tired now. SamuelRiv (talk) 20:16, 4 August 2008 (UTC)[reply]
This webpage has luminosity data on bright galaxies. Taking the spiral galaxies M31, M77, M81, NGC 3521 and averaging the brightnesses of their inner 2 arcminutes, we find that galactic nuclei are around magnitude 8.7 per 4pi square arcminutes or magnitude 20.3 per square arcsecond. This corresponds to class 5 on the Bortle dark sky scale, described as "Milkyway washed out at zenith and invisible at horizon. Many light domes. Clouds are brighter than sky. M31 easily visible. Limiting magnitude about 5.6 to 5.9." The galactic nuclei would be conspicuous unless light pollution is heavy. --Bowlhover (talk) 10:28, 5 August 2008 (UTC)[reply]

No one ha explained how this will appear to us earthlings or the effect it would have on earth...... --Anilmanohar (talk) 14:06, 5 August 2008 (UTC)[reply]

Using SamuelRiv's calculations, we would see something about the same size as a planet (i.e. showing features when viewed through binoculars or a telescope) and just about bright enough to be seen in an urban night sky. Gandalf61 (talk) 14:24, 5 August 2008 (UTC)[reply]
SamuelRiv only calculated the size of the parsec around the supermassive black hole; his purpose was to give an indication of brightness. Anilmanohar: see this excellent photo of the Milky Way. Look at the bright region on the right and imagine the dust lane does not exist. The galaxy would continue to get brighter as one approaches the center of where the dust lane is now, and the region around Sagittarius A*, as SamuelRiv calculated, would be magnitude brighter than magnitude 2.7. On average, the bright region would be as bright as a suburban night sky, but it would be extremely luminous at the center and comparatively extremely dim at the edges. --Bowlhover (talk) 20:56, 5 August 2008 (UTC)[reply]
So, in summary, an unobscured galactic centre would not be at all spectacular when viewed from Earth - we are just too far out in the galactic 'burbs. And, to answer the other part of the original question, I don't see how it would have any effect on life on Earth. Gandalf61 (talk) 09:00, 6 August 2008 (UTC)[reply]
It wouldn't be particularly bright, but I still think it would be quite spectacular - it's pretty impressive with the dust cloud in the way, without it it would be even better! --Tango (talk) 09:12, 6 August 2008 (UTC)[reply]

Why can't bullets fly ?

(fly in the sense that its form causes a little lift to compensate the free fall) —Preceding unsigned comment added by 77.224.230.72 (talk) 19:47, 4 August 2008 (UTC)[reply]

It does. A little. It would need constant thrust like a rocket though. ScienceApe (talk) 19:53, 4 August 2008 (UTC)[reply]

A form that would create lift would also increase drag, decreasing the effective range of the weapon. You also have to consider stabilisation; how would a lift-creating shape work with the spinning motion that most modern firearms use to increase accuracy? FiggyBee (talk) 20:02, 4 August 2008 (UTC)[reply]
Back in the olden days of SciRefDesk, you could get a whole treatise about how angle of attack, not asymmetric air-foil shape, can be key to lift. So have its center of mass displaced relative to the center of lift: essentially weight down the rear, increasing the angle of attack. The bullet flies rotating on its axis (spin-stabilized) but the axis is pointed nose-up. And has enough surface area on the "forward-facing bottom" so that the air pushes the bullet up, but not so much that you lose all your forward motion. And still maintain stability in the forward (as well as axial) direction without starting to tumble. Pretty soon you're just firing a normal rifle nearly straight up:) DMacks (talk) 21:50, 4 August 2008 (UTC)[reply]
Now that sounds interesting. Can you point us to an article that says all that/ —Preceding unsigned comment added by MisterRSole (talkcontribs) 01:10, 5 August 2008 (UTC)[reply]
Indeed, I would like to see that treatise. It seems to me that after the bullet exits the barrel in a straight line, the extra weight at the rear would cause the bullet to (gyrate? pronate? precess?) go into a speed wobble. Franamax (talk) 01:54, 5 August 2008 (UTC)[reply]
I had the same reaction. Weighting the rear would precess the spinning bullet and it would wobble its way dramatically off course. I'm skeptical of any angle-of-attack-related lift for a spinning, axially-symmetric shape. Nimur (talk) 04:53, 5 August 2008 (UTC)[reply]
Bullets can and do wobble to some degree—a really wobbly bullet produces horrible forensic wounds if it hits someone. I believe the degree to wobbling depends on how the weight is distributed in the bullet itself. --98.217.8.46 (talk) 02:49, 5 August 2008 (UTC)[reply]
Note my conclusion: "Pretty soon you're just firing a normal rifle nearly straight up". I'm completely doing thought experiments, speaking from first-principles physics and not any specific training in balistics design. The angle-of-attack issue is real for airplane wings though. But back to the bullet...by the time you fix all the problems and have enough tilt and enough spin to maintain stability and a steep enough angle of attack, your gonna tumble or something else rather than "fly". I don't think you could get a stable design that gave enough push "up" enough to generate real lift without pushing "up and over backwards" and creating back-spin. Wobble really does occur (and pretty much makes hamburger of whatever it hits). An easy way to get it is to have the weight distributed off-center from the axis...rotation about the center of mass thus has the bullet itself spinning off-center. Dunno how badly pushing center of gravity forward or backward would lead to wobble (lift on one end of the axis of rotation would produce torque at right angles, sending the bullet wobbling or curving sideways?). Maybe could also off-axis weight to counteract it? DMacks (talk) 04:25, 5 August 2008 (UTC)[reply]
Yeah, if I remember my gyroscopes right (which I don't) changing the direction of the axis of rotation produces precession (hah, I got to use the word :). Since the direction of flight isn't changing, that would add up to a wobble. Now I don't think adding an off-axis weight could counteract that, because - where would you put the weight? The bullet is spinning, so the off-axis weight is spinning around too, seems to me you could only increase the wobble. And now to complicate things, bullets are pointy-shaped, so aren't they already heavier at the back end? Franamax (talk) 04:57, 5 August 2008 (UTC)[reply]
It's not "heavier at the back" (CoG relative to object's overall form) but (dim memory) where the CoG is compared to center of lift or something like that. There was something in an Estes catalog about finding CoG of a model rocket, then tying a string there (wherever it was front-to-back) and swinging it around. If it flew level (not pitching up or down) then you were all set and you wouldn't wind up launching at through the neighbor's window by accident. Well I'll be damned, I managed to learn some science from the toys I played with as a kid! (learned about torque from Capsela too:) DMacks (talk) 05:50, 5 August 2008 (UTC)[reply]
OK, I think I understand you, the rocket on a string analogy is good, I thought they were only good for poking your eye out. Now the rocket over its various shape and contours will have areas of greater or lesser positive and negative lift. Adding up these vectors will give you the centre of lift. But how do you do that with the spinning bullet? If a patch of the surface has positive lift, it will have negative lift in one-half a revolution. Spinning the bullet will put one centroid of lift onto the rotation axis, now how do we move the fore-aft centroid away from the CG? Add aerodynamic drag at the back end of the bullet with little extra furrows? Would that add net negative lift at the back? Franamax (talk) 06:38, 5 August 2008 (UTC)[reply]
If one could stabilize the spinning bullet with an angle of attack, the Coanda effect would cause it to curve way off course (see curveball).
A (perfect) flying bullet would have about the same range as a ballistic one but, when it got to that range, it would just drop out of the air with no impact speed, all its umph having been used up as drag.
(Side note:if you fire a bullet horizontally and drop one from the same height they'll both hit the ground at the same time! (Unless one of them is "flying" :-) How cool is that!?) Saintrain (talk) 22:23, 5 August 2008 (UTC)[reply]
  • 'Cause the distance the air has to travel on the top of the bullet is the same as the distance the air has to travel on the bottom of the bullet. Hence, no vacuum. For an airplane wing, it is curved on the top and straight on the bottom. Comparted to the wing bottom length, the longer air travel distance on the top results in vacuum, which sucks the airplane right off the ground. Of course, if you laid a bullet on the ground and put the end of a vacuum nozzle over it, the bullet would fly upward. And if you shoot a small airplane wing out of a gun, you might miss what ever you are aiming at. Suntag (talk) 17:28, 7 August 2008 (UTC)[reply]

Plant identification: red berries

I saw these berries in High Park Toronto yesterday while I was out for a walk. I was wondering what kind of berries they are and if they are edible. They looked like red currants, but I'm not sure if that's what they are. Please see the photo I've linked bellow.

http://img.photobucket.com/albums/v244/shigil/berries.jpg —Preceding unsigned comment added by 99.254.23.238 (talk) 21:31, 4 August 2008 (UTC)[reply]

I don't live in Ontario anymore, otherwise I'd be running out to my little backyard paradise to look at the Honeysuckle bushes I planted. That was my immediate thought when I brought up the photo you link. Unfortunately, I no longer have the record of the exact species and my Audubon guide is not helping out, so maybe someone else can chime in here.
Our red currant article states "3-10 berries on each raceme" and I can't quite get to three when I count from your pic, so I'd say red currant is unlikely.
And of course, unless you're sure, assume they're not edible!
And looking a little more, a lot of people love High Park [4] [5], so I'm sure you can find some locals to ask if we can't pin it down. Franamax (talk) 01:35, 5 August 2008 (UTC)[reply]
They give me the impression of being from the dogwood family. This family is quite large and more expert knowledge will be needed to make an accurate identification. I certainly wouldn't put them on my pizza. Richard Avery (talk) 07:32, 5 August 2008 (UTC)[reply]
Lonicera morrowii I think. See http://www.ppws.vt.edu/scott/weed_id/lonmo.htm and http://www.illinoiswildflowers.info/weeds/plants/morrow_hs.htm William Avery (talk) 19:02, 6 August 2008 (UTC)[reply]
Aw crap - looks like I'm right about honeysuckle, but I always (tried) to take care to only plant native species in my yard. Maybe I wasn't clear with Dan at the garden centre that year. Any chance there's an equivalent native Ontario species? (I agree with your diagnosis, just trying to salvage my principles. You may have nailed the OP's plant right, it's my own I'm worried about now :) Franamax (talk) 09:41, 7 August 2008 (UTC)[reply]

Why did Newton think light speeded up in glass?

The Principle of least action was already a well known principle that explained Snell's law when Newton was investigating optics, so why did he come to the conclusion that light went faster when it entered glass rather than slower? —Preceding unsigned comment added by Dmcq (talkcontribs) 21:42, 4 August 2008 (UTC)[reply]

Did he? I hadn't heard of that, but it's a reasonable layman's interpretation to infer from observation that sound waves travel faster in denser materials, so perhaps electromagnetic waves would do the same. It's wrong, but understandable. ~Amatulić (talk) 22:29, 4 August 2008 (UTC)[reply]
Yes, it's how he explained refraction (thus accurate measurements of the speed of light in different materials were the first good evidence against Newton's ideas). I don't know how he came to this position, but it's probably in his Opticks. Algebraist 23:20, 4 August 2008 (UTC)[reply]
It is explained in [6]. Newton imagined that as light moved through air, the net attractive force on it from all air particles equaled zero. But at the interface between air and water, there was a net force pulling it towards the water, speeding it up. As its velocity along the surface remained unchanged, light would bend. this was Newton's explanation of snell's law and he was quite happy about it. :-) The different refractions of different colours were explained by the different colours being of different masses, so that the same force gave different changes in velocity. It wasn't until the 1850's that Léon Foucault got to compare the speed of light in air and water and discovered that light slows down in water.
As an apropos, [7] tries to speculate on what Newton would have made of simple experiments where the refraction of colours are opposite to the usual. EverGreg (talk) 07:12, 5 August 2008 (UTC)[reply]

Thanks very much. Very interesting. I must say I find his argument a bit unconvincing though and can see why he would be worried about Hooke! By his argument light of a single colour even if all the corpuscles were going a the same speed would have a different speed after entering glass depending on the angle they entered it at. But all the different speeds would have their vertical speeds leaving the glass changed by exactly the same ratio, that's a very strange idea for a particle theory - you'd expect the slower ones to change more if anything. I can see why he'd want a corpuscular theory with different shaped particles to try and explain polarization though. Dmcq (talk) 07:56, 5 August 2008 (UTC)[reply]

Sorry I misunderstood. He said the vertical attraction would change the velocity by the ratio rather than the vertical component. That makes a lot more sense. I guess he might have modeled it as two flat surfaces separated by a slope and the light particle going faster after it goes down the slope, the speed change wouldn't depend on the angle the slope was approached at. If the particles had differing speeds though they would bend differently so I guess he thought all light of a particular colour was going at the same speed - the obvious way then would be to have the different colours going at different speeds rather than have different masses. I must check if that is what he thought. Also I guess he liked having different particles for the different colours because Fourier was far in the future - who would have though different wavelengths could practically ignore each other like particles. Dmcq (talk) 18:27, 5 August 2008 (UTC)[reply]

Nope it looks like Newton believed all light moved at the same speed in space and he actually had quit a good estimate. I guess his reasonng was different speeds for different colours would give funny effects when looking at the moons of Jupiter. So he thought different colours were different masses. All a bit strange but it loks like he wasn't altogether sure of his theory himself because of Newton's rings Dmcq (talk) 19:13, 5 August 2008 (UTC)[reply]

Gas

Let's say I had a closed vessel containing only two liquid chemicals (no air), and this vessel had a known weight. If the two chemicals started a reaction which produced a gas, and I were to weigh the vessel afterwards, would the recorded weight constant (i.e. can a balance record the mass of a gas)? A second and unrelated question, how does water behave in a vacuum? —Preceding unsigned comment added by 76.68.246.7 (talk) 22:12, 4 August 2008 (UTC)[reply]

Assuming the vessel itself has mass, you can certainly weigh it "full of" something (liquid, gas, whatever), and then subtract the weight of the vessel itself to see what the effect of its contents are. If your reaction produces a gas, remember gas is still matter, and that matter has to come from somewhere--the liquid! See Conservation of matter. DMacks (talk) 22:15, 4 August 2008 (UTC)[reply]
As to how water behaves in a vacuum, the phase diagram for water shows that at zero pressure and some positive temperature, water will be in a gaseous state. The phase diagram for water also shows an anomaly in that for a certain range of temperatures, increasing the pressure will cause a transition from gas to solid, and then to liquid. Most other substances don't behave that way. ~Amatulić (talk) 22:27, 4 August 2008 (UTC)[reply]
The mass of the liquid, solid and gaseous contents of a sealed container would not change measurable as it went through chemical reactions or phase changes. In theory, it should weigh a tiny bit less (too small a change to detect on existing scales) if it emitted energy by heat or by radiation, and it should weigh more if it absorbed energy through the container, due to E=MC2. Edison (talk) 23:57, 4 August 2008 (UTC)[reply]
You'd need a very accurate scale to measure that! Even nuclear fissions yield mass discrepancies of tiny fractions of an atomic mass unit (relatively, on the order of hundredths of a percent). A chemical reaction has a far smaller energy change; as Edison said, no conventional mass measuring equipment could detect it (I am imagining some sort of emission spectrum fine structure analysis might reveal some indirect measurement which could reveal mass changes on this order of magnitude, but I've never seen anything like that in any lab I've been around). Nimur (talk) 05:01, 5 August 2008 (UTC)[reply]
I think you misunderstood my question. I know the total mass in the vessel will remain the same. I was wondering if a balance can detect the weight of a gas (afterall, it's not really touching the balance).
Yes, a balance can weigh a gas. Ultimately, the balance is reacting to pressure exerted, rather than "touch". Similarly, a truck full of birds weighs the same if all the birds begin flying within the truck. — Lomn 18:17, 5 August 2008 (UTC)[reply]
Whether the gas touches the balance or not, gravity is still acting on it, and it in turn presses down on the other substances in the vessel. If you stand on a scale, carrying a backpack would increase the weight reading even though the backpack is not in contact with the scale.
Yes, but if you let go of the backback (i.e. it's in freefall), then the scale won't take it into account. From my knowledge of gasses, molecules are touching each other and thus spend a considerable portion of their time in freefall, and so the balance shouldn't be able to detect their weight. So wouldn't the total weight measured be slightly less, because some of the atoms of gas are in freefall? —Preceding unsigned comment added by 76.68.246.7 (talk) 13:04, 7 August 2008 (UTC)[reply]
If you're trying to actually weigh a gas, however, you must take buoyancy into account. It has a significant effect on apparent weight when dealing such low-density substances. --Bowlhover (talk) 20:29, 5 August 2008 (UTC)[reply]
Buoyancy shouldn't have a meaningful impact on this example, though, with the gas in question enclosed in a vessel. — Lomn 23:20, 5 August 2008 (UTC)[reply]

Viscious liquid

i read lot of science pages. I notice lot of mention of vicious liquid. Why is it vicious. will it attack me/ —Preceding unsigned comment added by MisterRSole (talkcontribs) 23:07, 4 August 2008 (UTC)[reply]

Yes. It is a a very dangerous liquid. Simply vicious. It will tear your heart out if you give it half a chance. On the other hand a viscous liquid is a very thick, slow flowing liquid, such as maple syrup. Though they may lead to sticky situations, most viscous liquids are not plotting to kill you. Dragons flight (talk) 23:16, 4 August 2008 (UTC)[reply]

Viscosity is a measure of the resistance of a fluid which is being deformed by either shear stress or extensional stress. In general terms it is the resistance of a liquid to flow, or its "thickness". - Viscosity

(vicious edit conflictx2)Does that help? ---J.S (T/C/WRE) 23:17, 4 August 2008 (UTC)[reply]
Of course, viscosity is definitely at its finest with Non-Newtonian fluids. -- Captain Disdain (talk) 01:38, 5 August 2008 (UTC)[reply]
...Funny you should say that, because one of the definitions of a non-Newtonian fluid is one that doesn't have a single, well-defined viscosity. —Keenan Pepper 02:15, 5 August 2008 (UTC)[reply]
Yeah, and that's what makes them cool. -- Captain Disdain (talk) 10:02, 5 August 2008 (UTC)[reply]
Like cold custard? —Preceding unsigned comment added by 79.76.159.153 (talk) 20:47, 5 August 2008 (UTC)[reply]
According to this[8] the danger is the sharks. Julia Rossi (talk) 02:16, 6 August 2008 (UTC)[reply]

Expansion on shaking container of hot water

If I take a plastic food storage container and partially fill it with cold water, snap the lid on and shake it, nothing happens. If I repeat the procedure with hot water, very shortly after I begin shaking, the lid will vigourously pop off due to pressure buildup inside the container. If I do the experiment with a mason jar, there's a "Pfff!" of escaping air if I loosen the lid after shaking, but again, only with hot water. Why? The addition of dishwashing liquid seems, if anything, to amplify the effect, though I'm guessing this is due to the volume of suds produced. —Scheinwerfermann (talk) 23:20, 4 August 2008 (UTC)[reply]

Hot water heats the remaining air in the container. Warm air takes up more space than cold air. Shaking the container mixes the water with the air, warming it more quickly. Pressure builds up until the pressure difference between the contained warm air inside and the free, cool air outside is enough to pop the lid. 79.66.32.107 (talk) 23:30, 4 August 2008 (UTC)[reply]
Also, if the air is quite warm and the water is cold, you might notice the container collapse a bit. ---J.S (T/C/WRE) 23:45, 4 August 2008 (UTC)[reply]
"As the temperature is raised, gases usually become less soluble in water" <-- Maybe the heated water is releasing dissolved air. --JWSchmidt (talk) 03:55, 5 August 2008 (UTC)[reply]
Today's safety tip: Be very careful when blending hot liquids, basically for the reasons 79 gave above. -- Coneslayer (talk) 12:10, 5 August 2008 (UTC)[reply]
@JWSchmidt: Not likely, for two reasons. One; the water isn't being heated in the container and two; the dissolved gasses take up very little volume. ---J.S (T/C/WRE) 17:27, 5 August 2008 (UTC)[reply]

Here's a nice simple experiment to demonstrate the expansion of air with heat. All you need is a sink with hot water and an empty lightweight plastic bottle (like a big soda pop bottle). If your hot tap water is not very hot, you might start by heating some with a stove, microwave oven, or electric kettle instead of using it straight from the tap, in which case a funnel might be handy for pouring safely. Okay, first fill the bottle with hot tap water to warm the bottle, then dump out the water. Next put in just a little fresh hot water from your hottest supply in the bottom. Cap the bottle and shake it vigorously so the water heats the air, then uncap it and drain the water. Repeat with a little fresh hot water again, to make sure the air is really hot. After draining the water this time, quickly recap the bottle and close tightly. The heated air will now cool to room temperature and you will see how much it shrinks. --Anonymous, 18:36, August 5, 2008.


August 5

Apparent Incandescence of Planets

When viewed from earth, why do the planets of the solar system appear to be incandescent, as if they were stars? [This paragraph has been corrected by the originator. Sorry, folks, I didn't express myself right.]

We know that all the planets are below the temperature of incandescence.

Similarly, the earth's moon has a glowing appearance, rather like a fluorescent lamp. But men have walked on the moon and found it did not glow when the observer is close to it.

Things should get dimmer as they become farther away, not brighter. We know that from observation of things on earth. That also conforms with the principle that radiant energy decreases with the square of the distance.

What is the solution to this paradox? —Preceding unsigned comment added by Andme2 (talkcontribs) 04:26, 5 August 2008 (UTC)[reply]

Planets and moons shine because they reflect the light of the Sun, in varying amounts according to their albedo. Watch the moon throughout a month though, you will see large parts of it that don't glow brightly - those are the parts turned away from the Sun. (Oh and they're not the Earth's planets, they revolve around the Sun) Franamax (talk) 05:01, 5 August 2008 (UTC)[reply]
Are you suggesting that we don't own the entire solar system? I find it strange that your definition of ownership would depend so heavily on gravitational attraction. Does that mean that the Earth owns your car and your house? Human-made paperwork notwithstanding, Earth is certainly outcompeting you on net induced gravitational force acting on everything you think you possess. Or, you can adopt a philosophy that ownership transcends gravitational attraction... Nimur (talk) 05:06, 5 August 2008 (UTC)[reply]
Well on the few occasions when I've taken my car off the Earth's surface, Earth has exercised its prior claim to ownership pretty darn fast. I haven't tried launching a house yet, for one thing if I didn't have gravity, I would have wasted all that money on floors. Not to mention toilets with pipes that go downwards... Franamax (talk) 05:57, 5 August 2008 (UTC)[reply]
And since there is a chance you're serious: 1) ownership implies control, so Earth tells the Moon what to do, but it sure doesn't tell Ganymede when it's bedtime; 2) ownership implies relevance, if my car doesn't have me around it's not much good (since I'm the only one who has the keys), whereas if Earth's not around, Jupiter still works just fine; and 3) philosophize all you want, the Earth is what you're made from and the Earth is where you're gonna end up, all nice and molecule-like. Write your paperwork up, it's gonna end up as dirt too eventually. Of course, there will always be a Wikipedia :) Franamax (talk) 06:14, 5 August 2008 (UTC)[reply]
Wait, wait a second... How can you be so sure we came from Earth when even NASA is trying to find out if we really came from Mars? —Preceding unsigned comment added by 71.100.162.249 (talk) 20:58, 5 August 2008 (UTC)[reply]
They seem bright because you are looking at them contrasting with a black night sky. If you look at the moon in the day time (easy) or at the planets in day time - you need to have binoculars at least and know where to look you will find that the moon and planets are much dimmer looking. Only Venus is really bright, and that is because it is very white and closer to the sun. Graeme Bartlett (talk) 06:50, 5 August 2008 (UTC)[reply]
"But men have walked on the moon and found it did not glow when the observer is close to it." Actually, the Moon's surface reflects about 12% of incident light, giving it an albedo similar to that of worn asphalt.
Anyone who has looked through a telescope knows that the planets are far, far dimmer in terms of surface brightness than any star. No amateur telescope can show the disc of any star except the Sun; hence, stars appear to have about the same surface brightness regardless of magnification. Planets rapidly dim when the magnification is increased. --Bowlhover (talk) 07:23, 5 August 2008 (UTC)[reply]
First, each point on a planet or Moon does get dimmer as distance increases. However, you also can see more of the surface of such an object as you back off, up to the point where it all fits in your view. Backing up beyond that makes the object seem smaller, but this essentially squeezes all the reflected light down to a smaller and smaller area, such that the total amount of light that hits one rod or cone in your eye, in theory, remains the same, if there's no dust in between to absorb the light. This continues until the light from the planet or Moon only hits one rod or cone. From that point on the total amount of light hitting that point in your eye decreases with distance. However, the nearby planets and our Moon aren't far enough away for that to happen, while all stars except the Sun are. StuRat (talk) 12:18, 5 August 2008 (UTC)[reply]
"However, the nearby planets and our Moon aren't far enough away for that to happen" Just to clarify, all planets except Earth are seen as points with the naked eye. There are exceptions, but those are rare. --Bowlhover (talk) 21:13, 5 August 2008 (UTC)[reply]
Using my def of a point light source as one where all the light hits a single rod or cone in the eye, the nearest planets don't qualify. StuRat (talk) 17:09, 6 August 2008 (UTC)[reply]
By the def, not even the stars are point sources. The lens can't focus light perfectly, but even if they could, the Airy disk would spread out the light. These two factors are the reason bright stars like Sirius appear larger than dim ones like Gamma Trianguli. It's true that planets should appear much larger than stars, but only the actual appearance of stars and planets is relevant to the question. --Bowlhover (talk) 20:28, 6 August 2008 (UTC)[reply]
Another effect to consider is that our eyes dilate or contract in an attempt to make everything appear equally as bright, so that something like the Moon, which isn't very bright at all, seems bright when viewed at night. When you see the Moon during the day, you can really compare it with the Sun, and see how dim it really is. StuRat (talk) 12:22, 5 August 2008 (UTC)[reply]
Yet another effect is the night vision that only becomes fully effective after 30 minutes in the dark. Judging from http://webvision.med.utah.edu/imageswv/KallDark1.jpg, the eye's sensitivity threshold decreases by a factor of 60 000 after half an hour. Pupil dilation has a significant effect only in the first few minutes of dark adaptation, in which it allows 4-16 times the light-adapted amount of light to pass through (http://sightresearch.net/files/adaptation.htm). --Bowlhover (talk) 20:07, 5 August 2008 (UTC)[reply]

Height of mountains

how are the height of mountains calculated? how accurate are these?Shraktu (talk) 04:39, 5 August 2008 (UTC)[reply]

It could be done by dead reckoning, by pressure altimeters, or by triangulation. In modern times, GPS and RADAR are some other options. Nimur (talk) 05:03, 5 August 2008 (UTC)[reply]
Here's an article from Slate, but it's not actually all that informative... Nimur (talk) 05:23, 5 August 2008 (UTC)[reply]
When I was in high school I was taught that 0 m starts at the geoid level. --Kjoonlee 06:03, 5 August 2008 (UTC)[reply]
Yes it is not that easy to calculate exactly where the geoid is for a mountain a long way inland. This has been a diffulcult job for surveyors. It will involve taking precise measures of the vertical and comparing it with the known 0m datum at many points on a path to the top of the mountain. See if geodesy#Heights has anything. Graeme Bartlett (talk) 06:44, 5 August 2008 (UTC)[reply]
You may want to read Mount Everest for some info on the height and various ways it was measured Nil Einne (talk) 08:52, 5 August 2008 (UTC)[reply]
I believe triangulation is the most frequently used method. It has the advantage of not requiring a physical presence on the top of the mountain - don't forget that even today some mountains have not yet been climbed. Professional triangulation can also be amazingly accurate - the Great Trigonometric Survey of India in the 19th century measured the height of Mount Everest to be 8,840m, even though they were measuring from the border of India, about 150 miles away. This is within 0.1% of the current official height of 8,848m. Gandalf61 (talk) 08:58, 5 August 2008 (UTC)[reply]
You need some kind of zero point base to measure from, usually "sea level", which is a vague term at best. See Datum (geodesy)#Vertical datums. Pfly (talk) 09:18, 5 August 2008 (UTC)[reply]
Well, yes, but that is true of all the methods mentioned - they only actually give you a height relative to a base line. You then need some convention for defining the elevation of the base line - so you use a geodetic system. In the 19th century you put your base line near the coast (the Great Trigonometric Survye of India established its base line at Chennai) and measured its elevation relative to local mean sea level, which you averaged over several years. A modern survey would use the World Geodetic System. Gandalf61 (talk) 09:40, 5 August 2008 (UTC)[reply]

The height of mountains being above sea level means that on Mars or the Moon the mountain hieghts would be undefined, and that on Earth, as the hypothesized sea level rise progresses, all mountains would need periodic adjustments down in their reported heights. Seems odd. Edison (talk) 13:40, 5 August 2008 (UTC)[reply]

Yea it is non-ideal. Another thing is that a mountain like Mount Hood seen from Portland looks much taller than Pikes Peak seen from Colorado Springs, even though Pikes Peak is actually higher. But Portland is nearly at sea level and C-Springs is something like a mile high. But then, measuring elevation from sea level makes sense given human history and culture. Measuring from the center of the Earth might be more... scientific? Still, it's more useful to know how far above sea level your house is as a hurricane approaches, for example, than how far your house is from the center of the Earth. I agree it is odd though. "Sea level" is not the most stable and well-defined baselines. On the other hand, mountains are not exactly stable either, with their height changing not only from erosion but from larger scale uplift and sinking of land. This kind of height change may usually be slower than sea level change, but in some cases it happens catatrophically fast! Pfly (talk) 16:22, 5 August 2008 (UTC)[reply]

Semiconductor Resistivity

What is the "Four Point Probe Method" in measurement of Semiconductor Resistivity?Shraktu (talk) 06:27, 5 August 2008 (UTC)[reply]

I think you are looking for Four-terminal sensing --Dr Dima (talk) 06:33, 5 August 2008 (UTC)[reply]

Applications of Chem kinetics

do you know any websites on the applications of chem kinetics? Jaydilite2008 (talk) 12:10, 5 August 2008 (UTC)[reply]

Chemical kinetics would be a good place to start. EagleFalconn (talk) 13:18, 5 August 2008 (UTC)[reply]

Kissing scenes in movies - health risks?

In movie production, how do they prevent diseases from being passed from one actor/actress to another when they kiss? It doesn't look like there a lot that can be done when you see open mouths touching each other. On the other hand, if you kiss enough random people, sooner or later you'll catch something. Are actors and actresses really risking their health when shooting kissing scenes or do they have some kind of high-tech tricks not visible to the viewer that prevent disease transmission? —Preceding unsigned comment added by 71.185.73.240 (talk) 14:26, 5 August 2008 (UTC)[reply]

They are really kissing, if that's what you're asking. They probably do occasionally pass colds on to one another that way. But it's not much of an epidemic—the sum total of people kissing in movies per year is really just a handful. --98.217.8.46 (talk) 15:06, 5 August 2008 (UTC)[reply]
Plus during the filming of any one movie an every-day character I wouldn't expect will kiss more than 5 or 10 of their fellow actors on the movie. Given that shooting a movie can take anything from weeks to years it's hardly like the actor will be kissing significant numbers of individuals so I doubt the risk will be notably higher than the general population. 194.221.133.226 (talk) 15:14, 5 August 2008 (UTC)[reply]
Yeah, kissing is not risk-free, but it's not much different than shaking hands or other forms of close contact. Kissing#Disease_transmission is pretty scant but touches on the health aspects. Friday (talk) 15:15, 5 August 2008 (UTC)[reply]
The diseases spread via kissing are usually upper respiratory tract infections such as the common cold and influenza. Most people in good shape will recover from these within a week or two. Slightly more rare are the diseases such as infectious mononucleosis (kissing disease, mono etc) which are still usually non-fatal but take longer to recover. The risk is there but I suppose it's part of the job. —CyclonenimT@lk? 17:12, 5 August 2008 (UTC)[reply]
Lots of movies have prolonged tougue-wrestling scenes. There are maladies more serious and persistent than colds and flu which could be spread by such contact. I wonder if standard contracts at mainstream studios require medical certification that the co-star is free of herpes, Infectious mononucleosis, Hepatitis B and venereal disease in general? The the Columbia University health information site "Go Ask Alice" says these could be transmitted by kissing [9], although they call it "highly unlikely." Edison (talk) 19:19, 5 August 2008 (UTC)[reply]
How do they prevent that? For the most part, they don't. I mean, if someone has a terrible cold, I'm sure they avoid filming the kissing scenes to try and ensure they don't have two sick actors on the set, which'd be a waste of time and money. Likewise, if someone has a herpes flare-up, I'm sure they avoid kissing for obvious reasons. And of course, actors and directors tend to get medical check-ups before filming begins (for insurance purposes), so if they have any even remotely serious communicable diseases, chances are that they're known and being dealt with. But apart from situations like this, I don't think it's a major concern, any more than kissing someone on a date is. I mean, I'm sure some people just don't kiss anyone if they can avoid it, but for most people -- whether they're in front of a camera or not -- kissing is not first and foremost thought of as a health risk. I mean, if you're going to be worried about that kind of thing, chances are that you'll also want to avoid shaking hands with people, eating out, engaging in many sports, etc. -- at which point one kind of has to wonder if you're suffering from mysophobia or something. It's not really a very reasonable concern, all things considered. -- Captain Disdain (talk) 00:41, 6 August 2008 (UTC)[reply]

Do newspaper inks contain cyanide?

Do the inks commonly used to print newspaper and newspaper inserts contain cyanide? —Preceding unsigned comment added by 71.185.73.240 (talk) 16:18, 5 August 2008 (UTC)[reply]

This may help you. —CyclonenimT@lk? 17:09, 5 August 2008 (UTC)[reply]
Cyanide is a very dangerous chemical. I wasn't able to find any reference to it on any website discussing the dangers of newspapers/inks/etc. On the other hand, I did not find an itemized list of the chemicals in ink. ---J.S (T/C/WRE) 17:47, 5 August 2008 (UTC)[reply]
According to the link I just gave (albeit not in the free abstract), ink used in newspapers in the U.K. contains acetonitrile which is a cyanide.
Saying that acetonitrile contains cyanide is like the old trope about salt being made up of sodium, an caustic and explosive metal, and chlorine, a poisonous gas. While true in the sense that salt contains sodium and chlorine atoms, they exist in salt as (mostly) harmless ions, not as the poisonous elemental forms. Likewise with acetonitrile. Though it contains a cyano group, which is like the cyanide ion, the cyano group is covalently bonded to another carbon atom, which completely changes its reactivity versus the cyanide ion. While not completely harmless, acetonitrile is very different from the cyanide ion in the way it behaves in your body. It's like the difference between putting a cotton ball (cellulose) into your coffee and putting a teaspoon of sugar (sucrose). They may both be carbohydrates containing glucose groups, but what those glucose groups are attached to and the way they are attached makes a world of difference. -- 128.104.112.147 (talk) 20:49, 5 August 2008 (UTC)[reply]
Is it possible that Potassium ferricyanide or Prussian blue are used?87.102.5.5 (talk) 17:52, 5 August 2008 (UTC)[reply]

is this snake poisons —Preceding unsigned comment added by 71.3.150.136 (talk) 18:40, 5 August 2008 (UTC)[reply]

Strictly speaking, yeah, I guess its possible. However, acetonitrile and cyanide should not be confused. Acetonitrile is a very common organic solvent, cyanide is an ion. Acetonitrile has been known to breakdown into cyanide in human metabolic processes, however, and thats likely where any toxicity from newspaper ink would come from. EagleFalconn (talk) 20:00, 5 August 2008 (UTC)[reply]

Seems that nearly all the solvent from the ink would evaporate (otherwise the ink would never dry!), and that toxicity from the ink could be due to all the pigments too. Even the cited article above talks about a very different toxic chemical than acetonitrile. DMacks (talk) 20:58, 5 August 2008 (UTC)[reply]

The answer may be YES - see http://holmfirthtypo.blogspot.com/2006/04/complete-history-of-typography-part-3.html 87.102.5.5 (talk) 21:54, 5 August 2008 (UTC)[reply]

Pigments that may test positive for “free cyanide” under some test protocols include CI Pigment Blue 27, CI Pigment Red 169, as well as PMTA green and violet pigments

I'll leave the rest to you.87.102.5.5 (talk) 21:55, 5 August 2008 (UTC)[reply]


In my completely unprofessional, non-warrantied opinion: Don't get too worked up about tiny doses of cyanide. Cyanide is everywhere; the body has ways to deal with it. It's very toxic in the sense that it doesn't take much to kill you, but in the sense of nasty things happening to you from doses that don't cause symptoms, I'd be much more worried about benzo(a)pyrene, which one of the links suggests is in the ink. --Trovatore (talk) 22:03, 5 August 2008 (UTC)[reply]

I really should emphasize here that I'm not at all sure that what I write above is true -- this is a vague impression not based on anything solid. If anyone actually knows something about it I'd be interested to hear. --Trovatore (talk) 22:51, 5 August 2008 (UTC)[reply]
Toxicity from newspaper ink has been related to Antimony -- whether it still is, don't know and the article doesn't say. Julia Rossi (talk) 02:09, 6 August 2008 (UTC)[reply]
I think that must be slight wrong - antimony can be toxic - but surely would only affect those that worked with it ie the printers/typsetters/typemakers - occupational hazards.
I doubt there is (or was) any risk from antimony to the average reader.
But I could well be wrong. —Preceding unsigned comment added by 87.102.5.5 (talk) 02:27, 6 August 2008 (UTC)[reply]
Not the reader, but in other ways, so not for wrapping fish and chips or using on the body because it can be metabolised. Julia Rossi (talk) 06:07, 6 August 2008 (UTC)[reply]
Well, here's what I meant about cyanide. If someone forced me to make a choice between taking 100 mg of cyanide, or 100 mg of benzo(a)pyrene, I'd take the latter. It would quite possibly (probably? certainly?) give me cancer at some later time, but the cyanide would be certain death immediately. On the other hand, if the choice were between 1 mg of cyanide or 1 mg of benzo(a)pyrene, based on my current state of knowledge and belief, I'd take the cyanide, expecting it to be essentially harmless, whereas the hydrocarbon could still give me cancer even in the much lower dose.
So what I'm kind of curious about is, am I right about that? Does anyone really know? --Trovatore (talk) 08:21, 6 August 2008 (UTC)[reply]
It seems you want to avoid cancer, but not as much as instant death - I think your analysis is correct - though of course nobody really knows how much an increased chance of cancer from eating 1mg of benzopyrene - but 1mg of cyanide is not lethal (Cyanide poisoning). Actually you quite a good chance of surviving 100mg of cyanide - but not guaranteed. It's a probability choice and science can only help so much.87.102.5.5 (talk) 10:29, 6 August 2008 (UTC)[reply]

Logical human thought

I have a computer program which reduces multiple state equations to minimum form. When I record sensor data into a table as states of an independent variable for each sensor concurrent with some event and use the program to create a rule from the data, in some cases the rule that is created is very easy for a human to comprehend. I consider this process, therefore a form of simulated [logical] human thought. Is there any published objection to defining this process in that way? —Preceding unsigned comment added by 71.100.162.249 (talk) 19:27, 5 August 2008 (UTC)[reply]

The Chinese room argument? --Bowlhover (talk) 21:21, 5 August 2008 (UTC)[reply]
All of what I have read and skimmed through so far suggests Searle knew nothing of the process of logical equation reduction or that the human mind can apply it to memory of past events to formulate rules. Thanks for the reference. The student I am dealing with is at that age where whatever is read is accepted on face value without question as to whether it is true. Thanks. —Preceding unsigned comment added by 71.100.162.249 (talk) 22:12, 5 August 2008 (UTC)[reply]
True, but Searle used speaking Chinese as a random example of a human activity that computers may eventually do following code. Speaking English or reducing equations would have worked just as well. --Bowlhover (talk) 22:25, 5 August 2008 (UTC)[reply]
But conversely Searle's argument makes a different point: What might a human do in such a situation to prove they are human and not computer? "For if we believe that Searle has proven that the man in his room (or more precisely, the system of the "rulebook-plus-man") doesn't understand Chinese, then Searle has also proven that a Chinese person doesn't understand Chinese. After all, no individual cell in a Chinese person's nervous system understands Chinese, and all of these cells obey the "blind" laws of biology (This argument doesn't work so well against dualists, but Searle, and many of his fans, claim not to be dualists.), just as surely as the man in Searle's room blindly follows the rulebook." -- A Refutation of John Searle's "Chinese Room Argument" by Bob Murphy. Murphy's refutation of Searle's argument applies regardless of the human activity. —Preceding unsigned comment added by 71.100.162.249 (talk) 06:31, 6 August 2008 (UTC)[reply]

electro etching of copper with copper suphate soln

can it be done? —Preceding unsigned comment added by 79.76.159.153 (talk) 20:50, 5 August 2008 (UTC)[reply]

With a current and an electrode most any conductive solution can be used to effect electro etching. Ferric hydrochloride, however, works without any current at all. —Preceding unsigned comment added by 71.100.162.249 (talk) 21:03, 5 August 2008 (UTC)[reply]

Rise time

What is the shortest mans rise time recorded? —Preceding unsigned comment added by 79.76.159.153 (talk) 21:24, 5 August 2008 (UTC)[reply]

Did you mean 'mains rise time' or 'shortest man's rise time'?87.102.5.5 (talk) 21:35, 5 August 2008 (UTC)[reply]
Haha, there are so many things this could mean! Is it about climbing something? Springing to one's feet? Getting an erection? We may never know... —Keenan Pepper 00:46, 6 August 2008 (UTC)[reply]
Even the earliest ever time the world's shortest man got out of bed...  ; )) Julia Rossi (talk) 01:48, 6 August 2008 (UTC)[reply]
Maybe the OT has drowned someone and fears early detection? --Ayacop (talk) 08:45, 6 August 2008 (UTC)[reply]
Mr. He Pingping claims [10] to be the world's "shortest man" at 2.4 feet (73 cm). How quickly could one get a rise out of him? Perhaps by pointing put that Mr. Lin Yik-chic is only 67.5 cm tall [11]. Edison (talk) 14:08, 6 August 2008 (UTC)[reply]

No no. I mean shortest time for man to get eerection —Preceding unsigned comment added by 79.76.167.200 (talk) 23:31, 6 August 2008 (UTC)[reply]

One of Kinsey's studies probably has the answer.[12]. Suntag (talk) 17:21, 7 August 2008 (UTC)[reply]
What are you counting as the starting point? Beginning of stimulation? In which case, it's zero, since it's not uncommon to get erections without any stimulation at all (especially during puberty). If you mean from the penis starting to become erect and becoming fully erect, it's probably difficult to define exactly. --Tango (talk) 02:40, 10 August 2008 (UTC)[reply]

Question of utmost seriousness

Scenario: You take out a 500ml can of beer from the fridge where it has been sitting for the last 24 hours. You notice that its temperature is exactly +2 deg C. Unfortunately, because of the condensation on the can, it slips from your hand and falls a distance of exactly 1m to the floor. Miraculously, the can does not burst (although it may be dented), and all seems well. However, you quickly realize that if you try to open the can immediately, it will almost certainly mostly be lost in froth due to out gassing of the CO2.

Now the Q: If you were to return the can in question to the aforementioned fridge (with a temp of +2 deg C), how long would the can need to remain there before you could once again get it out and open it without any frothing and enjoy it at its best?

BTW its not homework-- I just done it ! :( —Preceding unsigned comment added by 79.76.159.153 (talk) 21:42, 5 August 2008 (UTC)[reply]

I'm guessing at this but I believe it is the same problem that requires cooling in the first place to remove the heat (in this case caused by the fall to the floor) it has acquired. Once the heat is lost there may be an additional time period for the molecules to forget their trauma and to go back to sleep since some of the energy from the fall was not turned into heat but into "nervous" molecules. I'd say give them another week. —Preceding unsigned comment added by 71.100.162.249 (talk) 22:20, 5 August 2008 (UTC)[reply]
A week? That's pessimistic! It's not really to do with heat or energy, it's just the dissolved gas coming out of solution, you need to give it time to dissolve again. I'm sure someone has done a detailed study on the problem, but in my experience about 20 minutes should do it. If it's been really shaken up, then maybe a little longer, but a simple fall wouldn't normally shake it up much. --Tango (talk) 22:54, 5 August 2008 (UTC)[reply]
I have spent many years pondering this issue. The first question is why shaking (agitating) the can makes it froth. The increase in temperature is a red herring (the increase in temp from dropping it is orders of magnitude less than the variability in the fridge). Frothing requires nucleating agents: in this case, this is small bits of dust etc that are present inthe liquid. Under calm circumstances, these particles adhere to the sides of the can; but if you shake or drop the can they become suspended and become much more effective nucleating agents: FROTH! Anyone got any comments? Robinh (talk) 07:15, 6 August 2008 (UTC)[reply]
Or, could it be that shaking it causes bubbles of air to get mixed into the liquid and those act as nucleation sites? --Tango (talk) 08:00, 6 August 2008 (UTC)[reply]
Both small bubbles and particulate matter can act as nucleation sites. Most commercial mass-market beers are filtered to remove the vast bulk of particulate matter. (Exceptions exist, of course. Trappist ales are perhaps the most dramatic example, though you'll only find those in glass bottles, not cans.) That leaves bubbles. Most will rise to the top of the beer after the can is allowed to stand undisturbed for a few minutes. I would also recommend flicking the can sharply a few times around its circumference with your finger; this will dislodge some of the bubbles which would otherwise remain stuck to the side of the can. TenOfAllTrades(talk) 13:17, 6 August 2008 (UTC)[reply]

From my experience, I'd think a few hours should be more than enough. --Kjoonlee 11:19, 6 August 2008 (UTC)[reply]

From my experience, holding your mouth over the opening as you pull the tab stops waste and saves time. DuncanHill (talk) 11:36, 6 August 2008 (UTC)[reply]

I don't have a clue how but tapping the beer (if in a can) several times on the top seems to prevent fizzing up by a large degree. I'd love to know the science to that, actually, but it always works for me even if I shake it up loads. —CyclonenimT@lk? 14:05, 6 August 2008 (UTC)[reply]

I used the "tap the can" method as the basis for a high-school science experiment. I did an exhaustive study with different types of taps, different lengths, different amounts of shaking (I even built a shaker to ensure all cans were shaken just as much) and I had controls that were not shaken, but sat there for the same length of time as those getting tapped. Just out of a pure guess, I also used a "turn the can upside down" think my grandmother used. I measured spray by opening the cans in a small fishtank with a lid (I cut holes in the plastic lid and used rubber gloves - so I could trap all the spray). In the end, I found that tapping the can for about 5 seconds stopped most of the spray. Also, turning the can upside down slowly and then upright again had the same effect. Finally, leaving the can sitting there for about 5 seconds had an equivalent effect. So, my conclusion was that it takes about 5 seconds for much of the fizzing to cease inside the can. If you like, you can spin around three times on one foot while singing Old McDonald. What you do during those few seconds isn't the key. It is merely the length of time. In the end - I got second place. I'm still a bit pissed about it. I was told that I got second place because the science teacher disagreed with my conclusion. She was certain that tapping the can had some sort of physical effect on causing the bubbles inside to burst and reduce fizzing. -- kainaw 17:20, 6 August 2008 (UTC)[reply]
[unsure of etiquette WRT editing out-of-chronological order]. Anyway, see Mpemba effect for a similar phenomenon of teachers not believing a student. Robinh (talk) 08:24, 8 August 2008 (UTC)[reply]
Ahahahaha! Welcome to the world of research :) Now you just need to replicate your results, publish your research, wait for (or encourage) others to independently replicate your results, and then you're home dry. But your teacher still probably won't believe you for another 20 years. 79.66.38.215 (talk) 23:25, 7 August 2008 (UTC)[reply]

OK AS the OP, i have to say that none of the above time estimates seem to concur with my experience. I have found that if I drop a can of lager from 1m or less, I have to leave it in the fridge for a considerable number of hours (may be 5 hours) before I can open it without frothing. Is lager different from other beers in this respect? —Preceding unsigned comment added by 79.76.225.183 (talk) 01:00, 8 August 2008 (UTC)[reply]

Lager is usually fizzier than ale, so I would expect it to need longer. --Tango (talk) 02:43, 10 August 2008 (UTC)[reply]

Harmful bacteria

I heard it said recently that the human mouth contains more harmful bacteria then the rectum or colon. Is that true or false? —Preceding unsigned comment added by 79.76.159.153 (talk) 23:16, 5 August 2008 (UTC)[reply]

Define harmful. There are normally orders-of-magnitude more bacteria near the exit of your alimentary canal than near the entrance, but I do not know how you would like to determine which are harmful and which are not. You would not want any of them in your blood stream, that's for sure. It depends on your immune system just how much harm any of them would do when given the opportunity. --Dr Dima (talk) 00:18, 6 August 2008 (UTC)[reply]
Well, oral bacteria in the bloodstream can possibly cause myocardial infarction. How much shit would you need to eat cause that? —Preceding unsigned comment added by 79.76.159.153 (talk) 00:31, 6 August 2008
Eating something does not automatically put it into your bloodstream. For a fair comparison you'd need to be injecting it. Probably a bad idea. APL (talk) 02:05, 6 August 2008 (UTC)[reply]
I'm not sure. I've been told that there are more bacteria per square centimeter on one's tongue and teeth than anywhere else in a healthy human body. Maybe that's totally wrong, but it is widely circulating idea at least. Dragons flight (talk) 00:29, 6 August 2008 (UTC)[reply]
It all depends whether you mean "(more harmful) bacteria" (i.e. bacteria that are more harmful than others), or "more (harmful bacteria)" (i.e. a greater number of bacteria that are harmful at all, even if only slightly). —Keenan Pepper 00:42, 6 August 2008 (UTC)[reply]
To 79.76: How much shit you would need to eat depends on how deep a shit you are in right now. Please use more appropriate terminology next time. To other readers: you may want to read Gut flora article. It clearly states the usual numbers of bacteria in human gut, as well as the more common genera of bacteria found normally in the gut. Clostridium, in particular, is a well known human pathogen, and many others are opportunistic pathogens. So, colon wins! --Dr Dima (talk) 01:13, 6 August 2008 (UTC)[reply]
Clostridium species, especially the dangerous ones like C. botulinum and C. perfringens, are pretty much everywhere in nature, though. They aren't harmful without special circumstances, though in those situations they're quite lethal. SDY (talk) 02:04, 6 August 2008 (UTC)[reply]

The gut flora article is missing all the plant viruses see doi:10.1371/journal.pbio.0040015 . And probably bacteriophages as well. --Ayacop (talk) 08:39, 6 August 2008 (UTC)[reply]


August 6

Shoe polish and solubility

Are there any household liquids that can dissolve shoe polish? It's obviously not very soluble in water. SDY (talk) 01:49, 6 August 2008 (UTC)[reply]

Warm water with detergent works just fine. If that didn't help for some reason, try petroleum ether. --Dr Dima (talk) 01:59, 6 August 2008 (UTC)[reply]
Hi, in the article Shoe polish there's this[13]. Being made up of waxes and oils, then a solvent for these would be turps as a cleaner for oils. Another solvent is Kerosine. Julia Rossi (talk) 02:01, 6 August 2008 (UTC)[reply]
Swarfega is fine for removing polish from the hands. Algebraist 12:58, 6 August 2008 (UTC)[reply]
I don't know bout household liquids, perhaps fingernail polish remover? I keep a very inexpensive product called 'Goop' hand cleaner. It was originally used by auto mechanics, but I use it after oil painting. Also, it's great for grease stains if you haven't washed the clothing yet.Kissnmakeup (talk) 04:14, 7 August 2008 (UTC)[reply]

Recycling plastic

Recently read about plastic refuse not only creating an island of junk the size of Texas or New South Wales but breaking down to particles the size of plankton without the eco benefits. I forget the name for them, but they end up eaten, then as part of the food that ends up on our plates. Given this process, rather than just landfill or ocean jetsam, is there an better option to crumble and mix plastics back into semi-permanent materials such as concrete, or road surfaces, or building foundations? Is anyone doing this already? Julia Rossi (talk) 01:53, 6 August 2008 (UTC)[reply]

To some extent. Did you check out plastic recycling?--Shantavira|feed me 06:43, 6 August 2008 (UTC)[reply]
With today's and tomorrow's energy costs, also incineration with energy recovery is one of several waste-to-energy (WtE) technologies. In summary, plastics are like biomass, you can reuse it or burn it. --Ayacop (talk) 08:32, 6 August 2008 (UTC)[reply]
Do you mean the Great Pacific Garbage Patch? Much of it is in tiny pieces, not yet plankton size, but too small to strain back out. Franamax (talk) 09:05, 7 August 2008 (UTC)[reply]
In what I read, it mentioned that as well as the synthetic plankton stuff. Maybe someone will sell real estate on the patch and it will just sail around on the currents something like a floating Howl's castle. I was more interested in what's being done in the gap between the big stuff that's already out there, the stuff that's being recycled up to a point, and the stuff that slips through the system and simply distintegrates without going away. Julia Rossi (talk) 11:42, 7 August 2008 (UTC)[reply]
Well the stuff in the ocean is there mostly because of carelessness. If you throw plastic out the car window and it blows into the ocean, it stays there for (almost)ever. Plastics that can be separated by type can be directly recycled. I don't think there's much that can biologically digest plastic, no matter how small the pieces are, I suspect they just come out the other end. As for the "gap", waste-to-energy is probably the best solution (if the emissions can be controlled). As far as I know, plastic is not a good mix with construction materials as it weakens them too much, but the fly-ash left after combustion is. The big problem here is collecting the materials. Recycling at source works well, if people throw things out the window, into the water, or into general garbage it becomes "energetically unfavourable" to recover the materials. And if manufacturers wouldn't put three layers of plastic around their products and stores wouldn't give out a plastic bag when you buy a pack of chewing gum, we would have less of a problem in the first place. Reduce, reuse, recycle - in that order. Franamax (talk) 20:20, 8 August 2008 (UTC)[reply]
Ah, thanks guys, thanks F'max. Julia Rossi (talk) 10:18, 10 August 2008 (UTC)[reply]

Gasoline Expiration Date?

I own an automobile that hasn't been driven since 2001. It has 80% of a tank of gas left in it from the last time I filled it up, when gas was somewhere around $1.30 a gallon. Now, I'm wondering if I could siphon it to another gas tank and it work just like new, $4 per gallon gasoline. Thanks. Danthemankhan 03:51, 6 August 2008 (UTC)[reply]

According to Gasoline#Stability: "When gasoline is left for a period of time, gums and varnishes may build up and precipitate in the gasoline, causing "stale fuel." This will cause gums to build up in the fuel tank, lines, and carburetor or fuel injection components making it harder to start the engine. Motor gasoline may be stored up to 60 days in an approved container. If it is to be stored for a longer period of time, a fuel stabilizer may be used."
There are many other Internet users with similar questions. Here is an anecdotal account in which 18-month-old cans of gasoline still powered a car perfectly. Also see these Yahoo! Answers responses. The Straight Dope has this article claiming gasoline has a shelf life of months to years, and degradation is due to oxidation, evaporation, and contamination by water. --Bowlhover (talk) 05:00, 6 August 2008 (UTC)[reply]
There are several microorganisms living on fuel, the German language WP has de:Kerosinpilz (kerosene fungus, Amorphotheca resinae, also growing in diesel) and a short mention exists in microbial corrosion#Aviation fuel about some other bacteria. Both types, however, need a certain percentage of water, so they appear only at specific places in the tank, where water condenses or collects on the inside or within the fuel at the bottom. Kerosene fungus is a big problem for the aviation industry. Probably much of what the links above claim as degradation is due to microorganisms. --Ayacop (talk) 08:15, 6 August 2008 (UTC)[reply]
There's also the problem of exactly when you filled it up. If you live somewhere with major seasonal temperature variation, then the winter fuel might not be suitable for summer use. Algebraist 12:55, 6 August 2008 (UTC)[reply]
It is important to know if this is diesel or unleaded. Diesel is considered to have a much shorter useful life than gasoline as diesel has problem with bacteria. Using winter gas in summer might cause vapor lock - but the extra higher volatility compounds may have already evaporated, effectively turning a winter mix into a summer one or worse. Rmhermen (talk) 14:19, 6 August 2008 (UTC)[reply]

I would also have reservations because gasoline is made up of many different components. Some of the hydrocarbons in there are light and could well have evaporated by now, with the heavier ones sticking around. This will make combustion very difficult and could cause the gumming up issues mentioned earlier. In short, as expensive as gasoline is now, given the risk and the conflicting information on the matter, safe is better than sorry. EagleFalconn (talk) 19:08, 6 August 2008 (UTC)[reply]

If you decide to throw it away, don't just pour it on the ground; although its properties of combustion may have degraded, it's still plenty potent enough to contaminate ground water.
Now, I'm sure you knew that, and the only reason I wrote it is to provide another alternative. Siphon it out, if you wish, and dilute it with newer gas, 5:1 or maybe 10:1. OR, use it in a lawnmower -- the gumming-up-the-innards stuff has already been done, and lawnmowers are ridiculously ignorant of octane and fuel quality.
You still have to deal with any potential damage done by the old product in the old vehicle. For that, I strongly recommend replacing the old fuel with new as soon as possible after emptying the tank, the goal being to minimize "drying out" the internal parts. If the car starts, it will probably not run well for a while, but the only way it's going to improve (in the short term) is to run more fuel through it
HTH! --Danh, 63.231.153.235 (talk) 21:10, 6 August 2008 (UTC)[reply]

Genital herpes Outbreaks

I dont have herpes, but was wondering that if someone does, and they take their medication everyday, do they ever have an outbreak again? —Preceding unsigned comment added by 76.168.89.122 (talk) 04:52, 6 August 2008 (UTC)[reply]

You might like to plod through the treatment section of the Herpes article or try google. Julia Rossi (talk) 05:21, 6 August 2008 (UTC)[reply]
(ec) There is more than one strain of the herpes virus, but immunity to even one of them will make a second herpes infection very unlikely (see http://chealth.canoe.ca/channel_section_details.asp?text_id=1370&channel_id=16&relation_id=27918). Infection is still possible as the 1980 study showed. --Bowlhover (talk) 05:27, 6 August 2008 (UTC)[reply]
By my understanding, it's common for a herpes infection to never be completely eradicated and just to lay dormant and then break out again later. That would suggest getting an infection doesn't yield immunity for the future. --Tango (talk) 05:35, 6 August 2008 (UTC)[reply]
Or as Demetri Martin would say - Glitter is the herpes of craft supplies... You think you've got rid it of it then go out into the sun then flare-up. 194.221.133.226 (talk) 08:00, 6 August 2008 (UTC)[reply]

Windowboxes

moved from the misc desk for better information Julia Rossi (talk) 10:03, 6 August 2008 (UTC)[reply]
Which are the best flower plants to use in windowboxes that are in full sun?TerryBohle (talk) 08:03, 6 August 2008 (UTC)[reply]

Now where were we, have you looked at cactus, geraniums, succulents? Julia Rossi (talk) 10:06, 6 August 2008 (UTC)[reply]
Wallflowers could work. DuncanHill (talk) 10:11, 6 August 2008 (UTC)[reply]
This site [14] suggests herbs and vegetables. DuncanHill (talk) 10:20, 6 August 2008 (UTC)[reply]
Other suggestions include miniature roses, dwarf gladioli, sunflowers, and daisies. DuncanHill (talk) 10:22, 6 August 2008 (UTC)[reply]

how to submit a picture with an question?

i need to submit a picture for detail of a question in help desk. how to do it ? —Preceding unsigned comment added by Shamiul (talkcontribs) 10:04, 6 August 2008 (UTC)[reply]

You scroll to the top of this page, click on Help desk on the right hand side under See also, and ask there for how to use wikipedia. Julia Rossi (talk) 10:34, 6 August 2008 (UTC)[reply]
Click on "Upload file" on the left. --Bowlhover (talk) 10:56, 6 August 2008 (UTC)[reply]
This assumes that the pic isn't already on the Internet. If it is, use a link like this: [15]. You can do the same thing for an image you've uploaded. It's also possible to have the pic display directly on this page, instead of by picking a link, but that's generally discouraged, because it slows down the page loading speed to a crawl. StuRat (talk) 13:33, 6 August 2008 (UTC)[reply]

Buoyancy on the ocean floor

If I take a cylinder (with its axis vertical) and immerse it in water, it will experience an upwards force due to buoyancy. As per the article, this is cause by the difference in pressure acting on the top and bottom surfaces of the cylinder (there are also pressures on the vertical walls of the cylinder, but these cancel out, and in any case they only act normal to the surface, ie in a horizontal direction). If I now push the cylinder all the way to the bottom of the body of water (eg the ocean floor), so that it is in contact with the (assumed to be smooth and flat) surface of the ocean floor, will the buoyancy force suddenly reverse direction and point downward?

Experience says no, but from the way I understand buoyancy, if the cylinder is sitting on the ocean floor, there is no more water underneath it exerting an upwards pressure force, so the only pressure forces acting on it are on its top and its sides. Since the sides are vertical and pressure acts normal to the surface, these forces act horizontally, leaving only a downward pressure force from the water on the cylinder's top surface. Therefore, the cylinder on the ocean floor has both its own weight and water pressure pushing it down, hence no more buoyancy, which would mean that even a light, air-filled cylinder would stay on the ocean floor. Where have I gone wrong? — QuantumEleven 12:24, 6 August 2008 (UTC)[reply]

It'd still be less dense than the water it displaces; thus it will still rise a little. --Kjoonlee 12:34, 6 August 2008 (UTC)[reply]
If you create a water-tight seal between the cylinder and the sea-floor and pump out any water in between them, then yes, the cylinder will be held on the sea floor by water pressure - you have, in effect, created an underwater suction cup. Gandalf61 (talk) 12:39, 6 August 2008 (UTC)[reply]
That's true, but it's not what Kjoonlee was asking. If the object is just resting on the ocean floor, and there's no seal (and partial vacuum) to make a suction cup, then it will rise off the floor and water will rush in under it. The reason is that the solid material that forms the ocean floor is also under pressure (the same pressure as the water touching it, because they are in mechanical equilibrium), so the floor itself exerts an upwards force on the object. 128.165.101.105 (talk) 14:03, 6 August 2008 (UTC)[reply]
I'm not sure that's true at all. You're assuming a semi-fluid sea floor. But the cylinder would still float even if it landed on solid rock. The reason is that the cylinder without the seal floats upwards because there is pressurized water underneath it, even if only a tiny bit. APL (talk) 16:34, 6 August 2008 (UTC)[reply]
Kjoonlee, You've just described exactly how suction cups work. The obvious practical problem is making sure that fluid does not get between the sea floor and your cylinder, which is why suction cups are typically made of rubber so that they can deform and create a seal with the surface they're sticking to. APL (talk) 16:34, 6 August 2008 (UTC)[reply]

I think Gandalf is right. In highly skilled metalworking, such as tool and die making, they use Johannsen (spelling?) blocks. These are metal blocks that have an exceedingly smooth and flat surface. If one surface of two of these blocks is made very clean and free of grit, and the two surfaces are then pressed together and rubbed together a bit, it will squeeze out all air between the blocks. One block can then hang below the other solely because of air pressure. The blocks are not magnetized.66.52.8.128 (talk) 16:58, 6 August 2008 (UTC)[reply]

That sounds like a cold weld. StuRat (talk) 17:06, 6 August 2008 (UTC)[reply]

Maybe I'm missing something here, and correct me if I'm wrong, but I think I'm detecting a slight error in logic. Assume the cylinder is less dense than water. If the cylinder is submerged in the middle of the water, the net force due to the pressure of the water existing is going to be zero because the water pressure will be equal in all directions (assuming still water of homogenous temperature etc etc). The model you guys are using for buoyancy only works at the surface. When submerged, the cylinder rises because of a difference in density. If it is more dense than water, it sinks. If it is less dense, it floats. As buoyancy explains (slightly poorly) the issue is weight difference manifesting as density difference. Because an object displaces a volume of water equal to its own volume, if the weight of the object is less than the weight of the water it displaces the water flowing downward decreases the overall potential energy of the system more. Therefore, water goes down, cylinder goes up. In the case where the (perfectly smooth) cylinder is on the (perfectly smooth) bottom, the cylinder will not rise because of the effect that 66.* mentioned. There is no path for the surrounding water to be under the cylinder to push up on it, so it stays put. If neither the cylinder or the bottom are perfectly smooth, water has access to the area underneath so it can push up. (Clarification: Collisions are pushing force only). EagleFalconn (talk) 19:21, 6 August 2008 (UTC)[reply]

The second part of your explanation is fine, but the first part is incorrect. When the cylinder is in mid-water the net force due to water pressure is not zero. It is equal in magnitude to the weight of an equivalent volume of water at that depth, but opposite in direction i.e. upwards - this has to be so, because the water pressure must be sufficient to support an equivalent volume of water in equilibrium. The net upwards pressure force arises because the water pressure at the bottom of the cylinder is greater than the water pressure at the top because pressure increases with depth. And this upwards pressure force is what we call buoyancy. This is explained in the Forces and equilibrium section of our buoyancy article. Gandalf61 (talk) 19:42, 6 August 2008 (UTC)[reply]
My mistake, let me rephrase. I did not mean net force due to water pressure. I meant the net force on the cylinder due to the water molecules colliding with the cylinder. A subtle difference, but my point was that the water colliding with the cylinder has nothing to do with the motion up or down, and that the motion was due entirely to the weight displacement of the water. EagleFalconn (talk) 14:22, 7 August 2008 (UTC)[reply]
Water pressure is due to water molecules colliding with the cylinder - these are just macroscopic and microscopic descriptions of the same phenomenon - see this section of the kinetic theory article. Water molecules colliding is pressure; pressure difference between bottom and top surfaces gives rise to net upward force that we call buoyancy; buoyancy force equals weight of equivalent volume of water. The displacement of the water has nothing to do with buoyancy - when you pour water from a jug into a glass you displace the water, but you don't create mysterious buoyancy forces. Gandalf61 (talk) 14:44, 7 August 2008 (UTC)[reply]

Let us suppose that the cylinder is a cylinder of water enclosed by an infinitely thin weightless skin. Even though the pressure of the surrounding water is greater at the bottom, the cylinder will not rise because its weight is such that the total pressure exerted by the cylinder on the water beneath it is equal to the pressure on it. The difference in pressure between the water under the cylinder and the bottom of the cylinder occurs when the cylinder weighs less than water. Then the total weight of the cylinder and the water above it is not the same as the weight of the water beside it. The difference in pressure is not just caused by the difference in pressure of the water between the top and bottom of the cylinder. It is also caused by the difference in the weight of the cylinder and an equal amount of water. Even when the cylinder sits on the bottom of the body of water, it weighs less than water. The pressure of the bottom of the cylinder on the bottom of the body of water will not be the same as the pressure of the water around it on the bottom of the body of water because it doesn't weigh as much, therefore the weight of the the cylinder and the water above the cylinder is not the same as the weight of the water beside it, regardless of where it is. So picture a free body diagram of the cylinder. The normal force of the bottom on the cylinder, the weight of the cylinder and the weight of all the water above the cylinder are acting on the cylinder. Are there any other forces in the vertical direction? Are these forces equal but opposite? I believe that if I took a wooden block in freshman physics lab and placed it on the bottom of a beaker full of water, its gonna float to the top because it's lighter than the water that it's in. If I take an ice cube and put it at the bottom of a glass of water, it might stick to the bottom of the glass because of dangling surface bonds, but I think it would float to the top. What do you guys think?Kissnmakeup (talk) 03:58, 7 August 2008 (UTC)[reply]

The problem with the idea that buoyancy force = weight of displaced water regardless of whether the object is on the bottom or not is that you then have to invoke some sort of negative pressure theory to explain the action of a suction cup, which will remain on the bottom despite weighing less than an equivalent volume of water. Returning to the block in a glass example, if the bottom of the block and the glass were sufficiently smooth to exclude water molecules from their interface (not going to be possible with a wooden block, but maybe possible with a plastic block ??) then, yes, the block would stay on the bottom, held there by its weight and the unopposed water pressure on its top surface. Our intuition is at fault here because our experience comes from objects that are not sufficiently smooth to observe this phenomenon. Gandalf61 (talk) 09:28, 7 August 2008 (UTC)[reply]
Yes, after further consideration, I agree. By sufficiently smooth, what you mean is that there cannot be even as much as the width of a water molecule of space between the surfaces of the container and cylinder, because if there is even that much space, the pressure will force the water up under the cylinder and there you will have it, via Pascal's Principle, buoyancy.Kissnmakeup (talk) 11:00, 7 August 2008 (UTC)[reply]
That was one hell of an explanation - thank you very much everyone, I think I've understood it now. Kudos! :) — QuantumEleven 11:52, 8 August 2008 (UTC)[reply]

Epileptology and Neurophysiology

A quick question about titles in the UK. Are the above 2 titles (Epileptologist and neurophysiologist) restricted to Medical Doctors only, or can a Clinical Physiologist (Neuro) call themselves either of those. Obviously they'd have to have further study in epilepsy, but if they had a masters in epileptology, would they be allowed to call themselves an epileptologist? Thanks 92.4.122.75 (talk) 14:33, 6 August 2008 (UTC)[reply]

I don't know if epileptology is a term regulated or licensed by a medical board, but it probably varies by region. Nimur (talk) 17:10, 6 August 2008 (UTC)[reply]

Name for when skin hurts

I've heard about something where someone's skin hurts as though it had a sunburn underneath the skin. First it starts around the chest. Each day, it is in a lower position and moves from around the chest, to around the stomach, to around the groin area, then down one or both legs. There are no visible marks, but there is tingling pain. What might something like that be called and does Wikipedia have any articles related to it? Thanks. Suntag (talk) 17:33, 6 August 2008 (UTC)[reply]

It may be an unusual form of paraesthesia. Fribbler (talk) 21:56, 6 August 2008 (UTC)[reply]
For me, when my skin hurts, that's a sure sign I'll be hit with severe flu symptoms a few hours from the time I notice the pain in my skin. Not sure about your localized moving patch, though. ~Amatulić (talk) 00:34, 7 August 2008 (UTC)[reply]
There is also a rare genetic condition which causes extreme sensitivity to touch on the front of the torso (as well as pronounced curvature to the shoulder blades). I remember the symptoms, but I have no recall of the name. Tried googling, but found nothing. -- kainaw 02:22, 7 August 2008 (UTC)[reply]
  • I've read that the skin pain caused by herpes zoster moves over time as the old chicken pox virus makes its way along the nerve endings around a rib and near the skin surface. However, that is a horizontal movement rather than an up and down movement. Per the article, herpes zoster could result in residual nerve pain, but it doesn't say anything about the residual pain moving in any direction. Suntag (talk) 16:37, 7 August 2008 (UTC)[reply]
If I had a recurring pain that seemed to be progressing through my body, I would go to see a doctor and explain my problem, since they would be far better able to help than random people on the internet. If I couldn't see a doctor for some reason, I would visit a pharmacist as pharmacists are also trained and experienced in these things and able to give an idea of whether it is likely to be serious. 79.66.38.215 (talk) 23:18, 7 August 2008 (UTC)[reply]
I doubt any pharmacist with his head on right would give you any kind of diagnosis or anything other than some over the counter crap that will do nothing to cure a serious problem. A doctor would provide a credible diagnosis.--El aprendelenguas (talk) 00:20, 8 August 2008 (UTC)[reply]
Pharmacists are not trained in medical diagnosis. And certainly no licensed pharmacist in the US would provide medical advice except on lotions and potions he's pushing. However, to answer Suntag's question, I have no clue, short of a full medical exam. Zoster does not match your symptoms, but that's about all I'll say. This is not the place to get a diagnosis. OrangeMarlin Talk• Contributions 04:26, 8 August 2008 (UTC)[reply]
Here in the UK, I'd certainly expect a licensed pharmacist to say "that sounds like something you should make an appointment with a doctor for" or "you've got the picture upside down". You know, order of magnitude sort of thing... 79.66.38.215 (talk) 17:30, 9 August 2008 (UTC)[reply]

Dental membranes?

Can anyone explain to me what dental membranes are? I understand that their purpose is to help regenerate gum tissue, but what are they exactly? What are they composed of? Is it man-made or does it contain animal parts? Are there any similar membranes used on the human body? —Preceding unsigned comment added by Cap'n Louise Redbeard (talkcontribs) 22:22, 6 August 2008 (UTC)[reply]

Guided bone regeneration might help. Julia Rossi (talk) 03:45, 8 August 2008 (UTC)[reply]

Heat rub on the testicles?

I was told today by someone who used to be in the army that it was a common practice for soldiers to smear heat rub (Deep Heat or Tiger Balm type stuff) on their testicles before embarking on long marches in cold environments, so that they wouldn't feel the chill as much. I'm pretty sure that this he was just trying me out to see if I would be dumb enough to actually do it (I'm not going to, no way!) so that he and his buddies could have a good laugh at my expense, but just to clarify -- would this actually work as stated? Or would you just keel over in burning agony after a couple of minutes? --84.70.255.204 (talk) 22:44, 6 August 2008 (UTC)[reply]

Well (sorry I cant resist it), it's possible that the person who told you was talking complete bollocks! Sorry ;( —Preceding unsigned comment added by 79.76.167.200 (talk) 23:26, 6 August 2008 (UTC)[reply]
You would keel over in burning agony after a few seconds. The Menthol and Methyl Salicylate will cause a very bad burning sensation due to the large number of nerves in the general area. Q T C 23:30, 6 August 2008 (UTC)[reply]
So would that prevent frostbite? —Preceding unsigned comment added by 79.76.167.200 (talk) 23:33, 6 August 2008 (UTC)[reply]
I doubt it. Stimulating nerve endings to feel a burn isn't the same as actually warming up flesh. There may be a small benefit through increased blood circulation that may prevent freezing. ~Amatulić (talk) 00:36, 7 August 2008 (UTC)[reply]
So, could this be the same nonsense that says: Have another whiskey to warm you up? When it doesn't at all really? —Preceding unsigned comment added by 79.76.167.200 (talk) 01:10, 7 August 2008 (UTC)[reply]
Whiskey does warm up your skin, by increasing blood flow from the warm core of your body to the cool skin. The problem is that if the room isn't warmer than your skin temperature, your core body temperature will fall. --Carnildo (talk) 20:15, 7 August 2008 (UTC)[reply]
One reads from time to time about mountaineers or cold climate travellers having suffered frostbite of the fingers or toes. Never in my life (fairly long) have I heard of frost bite of the testicles, or indeed associated anatomy. Richard Avery (talk) 07:25, 7 August 2008 (UTC)[reply]
Well that is hardly surprising is it? would you go out in cold weather with your testicles hanging out of your trousers or would you keep them snugly cossetted close to your body inside your nice warm underpants/ I know which Id choose. —Preceding unsigned comment added by 79.76.225.183 (talk) 00:43, 8 August 2008 (UTC)[reply]
Even testicles themselves don't go outside in the cold[16]. Julia Rossi (talk) 03:42, 8 August 2008 (UTC)[reply]

WARNING: DO NOT DO THIS unless you too want to know the pain of childbirth. Search You Tube for 'heat rub testicles' to hear the screams.

Please identify this spider

File:Dsc 4670.jpg
Mystery spider. Normally has eight legs; a few have been lost due to an unfortunate traumatic experience.
File:Mystery spider 2.jpg
Live specimen, dorsal view, with a U.S. penny for size comparison.
File:Mystery spider 3.jpg
Live specimen, dorsal view.
File:Mystery spider 4.jpg
Live specimen, anterior dorsal view. Isn't it adorable?

I have a bit of a spider problem. I believe I know what this spider is, as I've seen them plenty of times back home (Georgia), however nobody here (North Carolina) seems to believe they live in this area. This includes the exterminators, which puts me in a bit of a pickle. I've had five of the specimens I caught and preserved in alcohol taken by the exterminator and supposedly shipped to Atlanta for identification. They claim that Atlanta couldn't identify my spiders. Now the spiders have supposedly been shipped to Raleigh so whomever is in Raleigh can take a crack at 'em.

I find all this pretty darn silly since I'm nearly certain of what these spiders are after a few minutes of inspection with a magnifying glass. However, I'm not an entomologist, so I need other opinions (I may end up sending some of my other samples to the department of agriculture).

Just for kicks, what do you think this spider is? All of the spiders I've caught look just like this one (except with more legs), and vary in size from roughly 4 mm to 20 mm (leg span). If you happen to have studied entomology, please do mention it since that would give me a greater level of confidence. I won't tell you what I think the spider is because I'm hoping someone will provide a feasible alternative to my amateur identification.

Thanks. -- mattb 22:49, 6 August 2008 (UTC)[reply]

Does your university have a zoology department? They might enjoy a challenge. DuncanHill (talk) 22:53, 6 August 2008 (UTC)[reply]
Eh... UNC-G and Wake Forest University both have biology departments, but I can't tell whether they have anyone who specializes in entomology in particular. My best bet is probably NCSU in Raleigh, but me being in Greensboro that's a rather long drive. -- mattb 23:15, 6 August 2008 (UTC)[reply]
I assume you're afraid it's a brown recluse. It might be, but (a) Greensboro is well outside the brown recluse's range; (b) I don't think it's as easy to identify brown recluses as our article suggests; and (c) they're not as dangerous as they're often made out to be. The information here might be helpful. (Incidentally, you technically want an arachnologist and not an entomologist, although UC Riverside seems to lump them together.) -- BenRG (talk) 23:36, 6 August 2008 (UTC)[reply]
And I should add that I know zilch about spiders, I just happen to have been reading about this subject a few days ago. -- BenRG (talk) 23:41, 6 August 2008 (UTC)[reply]
I can only add, that it does resemble a brown recluse (-4 legs) unless it's a close relation to it. Spiders can migrate through soil importing, or other vehicles of transport (building materials, climbing into cars etc), so you never know. There could be reasons why they're turning up elsewhere. Out of interest, what do the North Carolignians call them? Julia Rossi (talk) 23:48, 6 August 2008 (UTC)[reply]
I'm not really afraid that IT is a brown recluse so much as I'm afraid that all fifteen or so of the identical-looking spiders I've caught are brown recluses. Far more than that I'm afraid that the spiders I have not seen or caught are brown recluses, especially since this little guy sustained its damages due to being violently swatted off my face in the middle of the night. Anyway, I've done my home work already and I'm more interested in knowing whether anyone can suggest another spider that this might possibly be. -- mattb 00:43, 7 August 2008 (UTC)[reply]

Or maybe you discovered a new species of spider. Just throwing it out there. --harej 23:49, 6 August 2008 (UTC)[reply]

A photo taken looking straight down at the body, and in daylight, would improve the accuracy of identification. It has the fiddle marking. Also, a macro closeup of the eyes would be diagnostic, since recluses have a different pattern of eye arrangement than most other spiders. They do indeed travel around outside their usual habitat, hitching rides in any old item like a box of junk, to a remote location. They seem to be getting a grand rehabilitation these days, with claims they are not very dangerous. I know 3 people who had bites from them, proven by the spider carcass being preserved, and who had a slow recvery with a depressed scar after the nasty result of the bite. They love to live in houses, and are aggressive about climbing into shoes and even into beds, then biting when we unwittingly come into contact. Yuck. Edison (talk) 18:38, 7 August 2008 (UTC)[reply]

I have no adequate macro lens, and this is quite a small spider anyway. Still, you should be able to see the eye arrangement from the full-size photo (I can). Nevertheless, I'll see whether I can take a better photo of one of the larger ones today.
I'm certainly no arachnophobe, but I'm equally unconvinced by claims of this spider being mostly benign. I too have a friend who was bitten by one and who lost a tangerine-sized chunk of flesh as a result. Honestly I'd rather be bitten by a black widow than one of these. Even if less than 10% of bites result in necrotic wounds, I don't like playing the odds after having one crawl on my face. -- mattb 19:32, 7 August 2008 (UTC)[reply]
Well I found a nice-sized undamaged live one today as it tried in vain to escape from my kitchen sink. Here are a few decent photos of the spider in a futile effort to climb out of a porcelain bowl! I have to admit, it's rather fun to watch and I'm going to feel bad when I finally commit this little guy/gal to the watery deep of the North Carolina sewage system. -- mattb 22:13, 8 August 2008 (UTC)[reply]
The dorsal markings do seem to match the Brown recluse spider. Sounds like it might have hitched a ride with someone to your current abode. Just my experience, this has been a good year for spiders: lots of insects due to the weather this spring, and the spiders are flourishing. Might be worth calling an exterminator. — The Hand That Feeds You:Bite 17:58, 9 August 2008 (UTC)[reply]
Okay, new question. Does anyone have an idea of where I can quickly find some sealed plastic containers suitable for shipping these spiders in alcohol? I've made a contact at NCSU who wants me to send him some specimens for identification. -- 98.26.182.245 (talk) 15:25, 10 August 2008 (UTC)[reply]
The local pharmacy could help -- screw-top specimen bottles or empty vitamin bottles are the go afaik. And you could ship those good pix to the article :) Julia Rossi (talk) 01:27, 11 August 2008 (UTC)[reply]
They're public domain, so you're welcome to do so yourself. -- mattb 01:44, 11 August 2008 (UTC)[reply]

August 7

Please try to disprove this theory

In the Alaskan Psychic Newsletter recently there was an article stating that George W Bush was an alien from 20 million light years away implanted here on our earth to cause havoc. How can this theory be disproved? —Preceding unsigned comment added by 79.76.167.200 (talk) 00:06, 7 August 2008 (UTC)[reply]

Maybe if he starts behaving hisself? Julia Rossi (talk) 00:17, 7 August 2008 (UTC)[reply]
It can't. Essentially you're saying, "How do I prove that Bush is not an alien?" (Or maybe, a humanoid Roomba belonging to an alien.) That doesn't mean the article is correct; it just means we have no credible evidence of an extraterrestrial civilization, especially one that would travel 20 million light years to swap babies with George H. W. and Barbara Bush. They've known George W. for quite some time, and don't really seem the Jonathan and Martha Kent sort. Those writing the article either knew it to be complete codswallop, or believe in; in the former case, they're not going to admit the joke, and in the latter, you can't get through the tinfoil.
I could as easily write an article claiming that Vladimir Putin is the illegitimate child of Elizabeth I and Rasputin, conceived as a result of time-travel technology that was hidden in the temple of Hatshepsut until brought to Paris by Freemasons, only later to be pocketed by Peter the Great after his visit to Rheims (and his reading of Anna Yaroslavna's prayer book). Equally difficult to disprove (though possibly more entertaining). OtherDave (talk) 00:24, 7 August 2008 (UTC)[reply]
To idly quote those who are wiser than I am: "Logic cannot remove what logic did not put there." SDY (talk) 00:36, 7 August 2008 (UTC)[reply]
Surely that is most illogical, Captain: after all I am a Vulcan. You're a mere human! —Preceding unsigned comment added by 79.76.167.200 (talk) 01:14, 7 August 2008 (UTC)[reply]
You can't falsify something unless it makes testable predictions. If the aliens in question are bug-eyed, for example, then this claim is falsified by a large amount of eyewitness evidence. Without that kind of detail you can't prove the claim wrong, but it also fails to qualify as a scientific hypothesis (or theory). It's no good to say that the aliens have the technology to hide their evil plot from us; that's still an unpredictive statement. -- BenRG (talk) 01:38, 7 August 2008 (UTC)[reply]
When superstitious theories are raised by those who are serious, skeptics often use Occam's razor. For jokes like this one, on the other hand, even Occam would groan. --Bowlhover (talk) 03:06, 7 August 2008 (UTC)[reply]

I might be opening up a can of worms, but I have a follow-up question. We know that this ridiculous theory can't be disproved. But isn't it also fair to say George W. Bush isn't an alien? Similarly, isn't also fair to say that god doesn't exist for the same exact reason? ScienceApe (talk) 03:40, 7 August 2008 (UTC)[reply]

It would be fair to say "we have no reason to believe that George W. Bush is an alien." which is typically as close as science ever gets to these sorts of unprovables. APL (talk) 06:46, 7 August 2008 (UTC)[reply]
What I was disproving was a theory that included "to cause havoc". Without that item, it is impossible to disprove. With that item, it would be "disproved" if he indeed stopped what he's doing and say, made daisy chains instead, because logic does depend on what is put there (or not) -- so there y' go... Julia Rossi (talk) 07:01, 7 August 2008 (UTC)[reply]
For a second there I thought you had it Julia, but no. If he was sent to cause havoc and didn't accomplish that, it would only mean that he was incompetent as a from-20 million light year-away alien. Franamax (talk) 08:07, 7 August 2008 (UTC)[reply]
Also, how would you know the aliens didn't order him to stop wreaking havoc? --Bowlhover (talk) 08:19, 7 August 2008 (UTC)[reply]
I can see only two possibilities. Alien GWB fails at causing havoc, terrestrial GWB causes havoc. Either way, EPIC FAIL. /rant Franamax (talk) 10:07, 7 August 2008 (UTC)>[reply]
Actually, I think we can logically disprove this theory. Start from the assumption that there is a lone alien who (a) can flawlessly impersonate a human being; (b) can manipulate human affairs to insert themselves into any desired position of power; (c) is rationally pursuing an objective of causing as much havoc on Earth as possible. Given this, the alien would insert themselves into a position with as much individual power as possible. The head of state or head of government in a democracy has relatively little personal power, so the position of President of the United States is not a rational target for our hypothetical alien. On the other hand, the leader of a military dictatorship in a country with a nuclear capability has a much greater potential for causing havoc ... Gandalf61 (talk) 09:09, 7 August 2008 (UTC)[reply]
You're assuming that the alien is omniscient, which may not be the case. --Bowlhover (talk) 09:33, 7 August 2008 (UTC)[reply]
Well, let's at least assume they have access to a comprehensive electronic encyclopedia of human knowledge - think I've seen one around here somewhere ... Gandalf61 (talk) 09:45, 7 August 2008 (UTC)[reply]
Ahem, you've seen the ceeohk, but have you seen the alien? Let's not assume the alien is godlike, but only acts that way, and who says he looks flawlessly human? and if he was incompetent might that be our secret weapon agaiinst invasion then? Logical presups are proliferatiing like nuclear armaments here... Julia Rossi (talk) 10:07, 7 August 2008 (UTC)[reply]
See wishful thinking --Shaggorama (talk) 17:32, 7 August 2008 (UTC)[reply]
  • Since one person's causing havoc is another person's working towards orderliness, accepting proof of the theory seems based on whether you are the hammer or nail. Suntag (talk) 19:39, 8 August 2008 (UTC)[reply]

Will going faster than the speed of light create a black hole?

I was having a conversation about the speed of light with someone I knew and he said that according to theory, if you go faster than the speed of light, it will create a black hole. Can someone please explain this theory to me, and if this is even true. Monkeynator03 (talk) 00:11, 7 August 2008 (UTC)Monkeynator03[reply]

No, that's not true. (Well, I suppose you could argue that it's vacuously true because you can't go faster than light in the first place.) I've never heard that one before. I have heard people say that something going very fast (but slower than light) will become a black hole, which is also not true. -- BenRG (talk) 01:18, 7 August 2008 (UTC)[reply]
I'm going to echo vacuous truth in non-small font, because I think it's the most useful thing you're going to get out of this question. As the first half of the statement ("things that go faster than light [in vacuum]") is an empty set, the second half of the statement can be anything and, from a logical standpoint, the collective statement is true. "If I go faster than light, I create a black hole" is the logical equivalent of "If I go faster than light, Wikipedia tastes like candy".
On the other hand, when that "[in vacuum]" caveat I added is removed, you can make a meaningful statement: when a particle exceeds the speed of light in a given medium, Čerenkov radiation is produced. But not a black hole. — Lomn 02:17, 7 August 2008 (UTC)[reply]
That's a misconception. I just tried it. Wikipedia tastes like a LCD screen. See also "Why cant I type?" below. :) Franamax (talk) 08:00, 7 August 2008 (UTC)[reply]

I think the misconception may stem from the fact that relative mass increases the faster you go. People confuse relative mass with rest mass. You can travel at 99% the speed of light, and your relative mass will be immense, but your rest mass is the same as it was at rest. Creating a blackhole depends on immense rest mass coupled with immense densityScienceApe (talk) 03:34, 7 August 2008 (UTC)[reply]

Black Holes can only be created only if a body possesses a redius less than Scwerckild Radius(2MC/C2). Travelling faster than light won't create a Black Hole.117.201.96.57 (talk)` —Preceding undated comment was added at 20:17, 9 August 2008 (UTC)[reply]

TeleDeltos paper

Where can I get it? I need to plot. —Preceding unsigned comment added by 79.76.167.200 (talk) 00:15, 7 August 2008 (UTC)[reply]

The authors of this webpage ordered it from Sensitised Coatings in England. --Bowlhover (talk) 07:21, 9 August 2008 (UTC)[reply]

Why cant I type?

Why cant I type after 8 pints. I mean, my brain knows what to do, but my fingers just dont obey me? Why is that? —Preceding unsigned comment added by 79.76.167.200 (talk) 01:20, 7 August 2008 (UTC)[reply]

Alcohol is a central nervous system depressant, so although your brain sends the signals, they don't quite make it there in good shape. Q T C 01:34, 7 August 2008 (UTC)[reply]
20 20 24 hours a day, I want to be -- typing???
Duuud! I woz sooooo wasted I couldn't even -- type???
O man! She sooooo wanted it but I had to -- type???
Youth is wasted on the wrong people! Saintrain (talk) 04:58, 7 August 2008 (UTC)[reply]
Checking my email is the first thing I do when I get in, drunk or otherwise, and I agree typing is a significant challenge. It's not just more typos, it's different typos as well... I don't generally think about it much, though - I'm drunk at the time, after all! (I do have to be careful about *what* I'm typing though!). --Tango (talk) 18:29, 7 August 2008 (UTC)[reply]
True. I find I type similar-sounding words rather than the word I want when I'm drunk or tired. Which is frankly bizarre. It's like my fingers are taking dictation from my mind, but are themselves drunk... 79.66.38.215 (talk) 23:05, 7 August 2008 (UTC)[reply]
Oh dear... I do that sober... It's terrible, it makes it look like I don't know the difference between "there", "their" and "they're", when I do, it's just my fingers that don't. --Tango (talk) 02:48, 10 August 2008 (UTC)[reply]

You might also want to check out short-term effects of alcohol, specifically this bit. —CyclonenimT@lk? 11:22, 7 August 2008 (UTC)[reply]

If sufficient pitchers of beer are consumed, it may be perceived to be an insoluble problem to pour the next glass from a newly arrived pitcher, because they put the handle on the wrong side of the pitcher! Just saying. Edison (talk) 18:30, 7 August 2008 (UTC)[reply]

Helium lifting gas

What isotope of Helium provides the most lift? ScienceApe (talk) 03:35, 7 August 2008 (UTC)[reply]

The lightest one, naturally, i.e. Helium-3. The gain isn't much, though, and you'll have trouble getting it in useful quantities. Algebraist 03:40, 7 August 2008 (UTC)[reply]
(e/c) I assume you mean "the most lift if you put it in a balloon". Helium-3 would probably provide more lift than Helium-4 because it's lighter, but it's ridiculously rare. Paragon12321 (talk) 03:41, 7 August 2008 (UTC)[reply]
Unless we start gas mining the moon or Jupiter. ScienceApe (talk) 04:55, 7 August 2008 (UTC)[reply]
Nah - make some tritium and wait for it to decay .. Philip Trueman (talk) 11:29, 7 August 2008 (UTC)[reply]
If you wanted to fly an airfoil through a purely helium atmosphere, wouldn't the more massive isotope provide more lift? Nimur (talk) 19:30, 7 August 2008 (UTC)[reply]
No. Aerodynamic lift depends on air pressure, not density. So lift should be the same for all reasonably gases (various caveats not discussed to keep it simple). --Stephan Schulz (talk) 03:48, 8 August 2008 (UTC)[reply]
And wouldn't hydrogen be even better?, in a purely helium atmosphere there is no chance it would burn. Graeme Bartlett (talk) 02:11, 8 August 2008 (UTC)[reply]

Talking about the lift of various gases, water has lift too. I once worked for an aerospace firm that took a government contract to design and build a one-man submarine. They knew nothing about submarines. They were embarassed to find that the submarine kept heading for the surface when it should have gone straight ahead. They finally found out that the shape of the hull caused lift. AndMe2 66.52.8.251 (talk) 05:47, 8 August 2008 (UTC)[reply]

Technically, Helium-2 is the lightest isotope. It has a ridiculously tiny half-life, so the result of a balloon full of it would be similar to that of a nuke going off. The convection from the superheated gasses would result in the most lift, although if you're going to do it that way, it might be better to use extremely heavy isotopes of helium, which AFAIK would release more energy for a given volume. It would, of course, be impossible to fill a balloon with a helium isotope other than 3 and 4 before it decays. — DanielLC 00:32, 10 August 2008 (UTC)[reply]

Curl of F

If the curl of F is not zero, what does it represent besides the presence of friction? For example the curl of E is the time rate of change of B. So when the curl of F is not zero, what does the quantity represent? —Preceding unsigned comment added by Kissnmakeup (talkcontribs) 04:58, 7 August 2008 (UTC)[reply]

The circulation. Have you seen Curl (mathematics)#Interpreting the curl? 81.174.226.229 (talk) 08:33, 7 August 2008 (UTC)[reply]

I see that I need to be more specific. According to Keith Symon in his text, Mechanics, if the curl of F (and consequently the line integral about a closed path by Stokes' Theorem) is zero, then the force is conservative. If it is not zero, energy is not conserved. When energy is not conserved does the quantity of the curl of F indicate anything quantitatively about the lost energy? If you do a line integral around a closed path and you don't get zero, is the quantity that you get equal to the energy dissipated by, say, heat from friction? Does the number that you get have a physical meaning specificly with regard to energy not being conserved? (By the way for those of you who are interested, I chose this strange handle after reading some of the discussion pages for articles like entropy.)Kissnmakeup (talk) 11:32, 7 August 2008 (UTC)[reply]

No, it doesn't always represent friction. In the example you gave of the electric and magnetic fields, it obviously doesn't represent friction. It represents the EMF induced by the changing magnetic field. You say "energy is not conserved", but of course the total energy of the system is always conserved. For example, if the changing magnetic field produces 1 V of EMF, and I let one electron go once around the loop, it gets accelerated by 1 eV of energy. This energy is exactly the energy that came from the magnetic field. Other non-zero curls of forces similarly represent transfers of energy. 128.165.101.105 (talk) 14:15, 7 August 2008 (UTC)[reply]

Cats and their waste

Cats are generally good about disposing of their waste – they dig a hole in loose soil or sand, do their business in it, and then meticulously cover it up. Do big cats, like lions and tigers, do the same? --Bruce1eetalk 06:07, 7 August 2008 (UTC)[reply]

Instinctively domestic cats bury their waste to protect their trail from predators. This usually only applies to subordinate cats however, and sometimes a dominant cat will actually display it's waste as a sign that "this is my territory, keep the hell out". I would imagine that this behavior is the same in big cats, as "the various species of cat are amazingly similar in both structure and behavior." Of course, some cats use other methods, though I'm not sure how a lion would look doing that! 20I.170.20 (talk) 13:24, 7 August 2008 (UTC)[reply]
It might be of interest to note that Christian the Lion used a litterbox scrupulously when he lived in London. - Nunh-huh 20:34, 7 August 2008 (UTC)[reply]

I have read that cats bury their waste so their prey will not know, by smell, that a cat is in the area. I have also read that cats frequently lick their fur so their prey will not smell the remains of earlier prey emanating from the fur. It's too bad cats are predatory animals - I really like them. They seem to know so much and they say so littls. They are a riddle wrapped in an enigma. AndMe2 66.52.8.251 (talk) 05:38, 8 August 2008 (UTC)[reply]

Air brakes

I know that fast cars, especially race car and high end road cars, have spoilers to create massive downforce. Hundreds and hundreds of pounds.

Why do I not see more cars with Air brake (aircraft)? The Veyron has one [17] --mboverload@ 06:22, 7 August 2008 (UTC)[reply]

You do. That "something" must be rather large to have a significant braking effect at the low velocity of race cars (relative to jet planes, that is), so a parachute is used, to help stop dragsters at the end of their runs. For race cars that go around a curved track, there's no hurry to slow down, as they could just go around the track a few times and coast to a stop if they wanted to. StuRat (talk) 06:31, 7 August 2008 (UTC)[reply]
So the issue is that at even high road speeds air braking would have little effect? And any effect would be mitigated by the increase in weight of the air braking system? --mboverload@ 06:49, 7 August 2008 (UTC)[reply]
If you used the same sized mechanism as on a plane it would have little effect, yes. I still say that a parachute IS a type of air braking, however, just one designed to use a much larger surface area and yet still have a low weight. StuRat (talk) 14:49, 7 August 2008 (UTC)[reply]
Air brake only makes sence if you are fast 100km/h is not enough to give you the benefit you want. But in racing the technice is old and was established in formula 1 racing through the Mercedes 300 SLR in the 1950s. This modification was forbidden, partly due to the fact that you block the view for all cars behind. --12:46, 7 August 2008 (UTC)

Loud booms

I suppose this goes in here. Hello, I'm from Temecula, California, and I've been hearing successive loud booms coming from a west by south-western direction for over an hour and it's freaking me out a bit. From a scientific standpoint, what is causing the booms, and why are there so many of them over a period of about an hour and a half? Camp Pendleton is in that direction, though I think it's too far away for anything to be heard from here if something is going boom over there. And this started around 11:00 PM local time.-- 07:41, 7 August 2008 (UTC)[reply]

You might try reading about the Sonic boom, although I don't know why they would cause such a disturbance that late at night. Kissnmakeup (talk) 11:07, 7 August 2008 (UTC)[reply]
Speaking for my area, which is near a military test facility, we're subject to destructive testing. While the base normally tries to blow things up during normal work hours, some tests don't quite go as planned -- we've had unusually large (heard across multiple counties) explosions past 11 PM on occasion. Some similar event could be the cause. However, there's no possible way for us to ascertain the cause from a scientific standpoint, as there's not enough information available. — Lomn 13:11, 7 August 2008 (UTC)[reply]

Čerenkov radiation/speed of light/follow on from previous question

Someone said earlier that:

when a particle exceeds the speed of light in a given medium, Čerenkov radiation is produced

How is this possible? I was always taught, albeit I'm only an A-Level student, that things cannot go faster than the speed of light, especially if they actually have mass. Particles have mass, don't they? —CyclonenimT@lk? 11:27, 7 August 2008 (UTC)[reply]

Things cannot go faster than the speed of light in a vacuum. Čerenkov radiation is seen in media such as water. William Avery (talk) 11:51, 7 August 2008 (UTC)[reply]
Let me clarify my point of emphasis: there is c, the "speed of light in a vacuum". c cannot be exceeded, ever. However, much like the speed of sound varies in different materials, so does the speed of light. Light travels through water ("the speed of light in water", if you will) at some rate less than c, and particles can exceed that speed. Exceeding c in water remains impossible. — Lomn 13:08, 7 August 2008 (UTC)[reply]
Incidentally, the differing speed of light in different mediums is the cause of refraction, as given by Snell's law, and diffraction. --Bowlhover (talk) 19:36, 7 August 2008 (UTC)[reply]

Keeping Cool

How can your body keep itself cool when the outdoor temperature is 37C and there is 100% humidity?

Surely there is no way for the body to keep cooler than 37C and the temperature will therefore continue to rise, how come people do not die? And yet people live in such conditions, not necessarily with air conditioning.

I am going on holiday next week and it is very likely I shall face this problem (although I will have central air most of the time). 62.24.129.68 (talk) 11:56, 7 August 2008 (UTC)[reply]

Well, firstly people do die from hyperthermia. The best way to stay alive is simply drinking plenty of water (plenty!) and to use sunscreen to help keep sunburn away. Avoid drinking alcohol and caffeine during the day, these all play games with your homeostatic mechanisms. Oh, and don't do too much vigorous exercise in the heat if you're not well-trained to! —CyclonenimT@lk? 12:10, 7 August 2008 (UTC)[reply]
The last response didn't seem to address the core issue. Let me try to state it: "If the humidity is 100% and the temperature of everything in the person's environment is significantly hotter than normal human body temp, what mechanisms does the body have to cool itself ?" In such a case evaporative cooling shouldn't work, neither should conduction of heat into a cooler object, as no cooler object is available, and neither should convection of air around the person, as that requires cooler air to replace the hot air. Perhaps radiation can still work, if the person is somehow able to radiate heat more than the objects around them, provided this difference in thermal radiation can overcome the additional heat the body absorbs due to the other forms of heat transfer.
I'd say that the way people are able to survive is that they avoid such environments except for brief periods. In places where such conditions exist, they likely hide from them until they pass, such as in undergound dwellings which remain cooler or in an adobe abodes which retain the coolness of night well into the heat of the day. However, if conditions remained like that, day and night, permanently, I'd say such a place would indeed be uninhabitable. StuRat (talk) 14:41, 7 August 2008 (UTC)[reply]
I don't think radiation is going to help - the rate at which you radiate heat depends on your colour (black radiates most), but the same colours which radiate a lot absorb a lot, so the affect is going to cancel out. Even if it doesn't cancel out exactly, it's still going to be far to low to help much. I think people would soon die in such conditions - you have to just get out. The human body can survive in very high temperatures (I'm not sure how high, but certainly much much more than body temperature) if the humidity is very low, but in high humidity, heat can easily be fatal. --Tango (talk) 18:36, 7 August 2008 (UTC)[reply]
I'd classify that as "not survivable": the heat index of 37C/100% humidity is 83C, far above what the article gives as the "extreme danger" point of 54C. --Carnildo (talk) 20:33, 7 August 2008 (UTC)[reply]

Wormholes and momentum

If wormholes exist, then wouldn't their existance allow one to violate the law of conservation of momentum, ie by placing the two wormhole mouths at a right angle to each other (if you go through one vertically, and then exit somewhere else horozontally, then momentum's not conserved because momentum is a vector). —Preceding unsigned comment added by 76.68.246.7 (talk) 13:09, 7 August 2008 (UTC)[reply]

Wow, where to begin? First of all, a real wormhole solution of general relativity is not just some magical portal where you go in one "mouth" and instantly pop out the other. There is no clearly defined "mouth", and every point of spacetime (even inside the wormhole) has a little patch of spacetime around it that looks locally flat. That's one version of the equivalence principle, and it's extremely important for general relativity (it means spacetime is actually a manifold, so it can't have any corners, edges, or abrupt changes of any kind). So it could be that if you go in one end of your wormhole, you have to fire your rockets and turn inside it in order to come out the other end. Conservation of momentum is certainly not violated.
Getting into a little more detail than perhaps you're looking for, momentum actually isn't conserved globally in GR. That statement actually makes no sense, because momentum vectors at different points in spacetime aren't even in the same vector space, so you can't compare them. Instead, if an object with a certain momentum moves through spacetime, its momentum vector undergoes parallel transport, and if the spacetime is curved, you can go around a loop and back to the point you started from, and have the vector be pointing a different direction. See holonomy. So even if you can go through your wormhole and back around to where you started and it seems like momentum isn't conserved, it's actually not a problem. Momentum is always conserved locally, that never changes. —Keenan Pepper 13:31, 7 August 2008 (UTC)[reply]
Thanks a lot! —Preceding unsigned comment added by 76.68.246.7 (talk) 20:54, 7 August 2008 (UTC)[reply]

Project Management

WHAT IS PROJECT MANAGEMENT ALL ABOUT? WHAT DOES PROJECT MANGEMENT REALLY ENTAILS?

REGARDS, J.J. HARRY —Preceding unsigned comment added by 66.178.116.10 (talk) 14:17, 7 August 2008 (UTC)[reply]

First, stop SCREAMING (typing in all caps means you are angry and screaming at everyone) and then try reading project management and come back if you still have questions. -- kainaw 14:19, 7 August 2008 (UTC)[reply]

Why would a pencil sharpener need microbial protection?

I saw a pencil sharpener advertised in an OfficePro flyer that said, "ACME PENCIL SHARPENER/ERASER COMBO \ With Microban antimicrobial protection." Why would it need this? --205.174.162.243 (talk) 14:43, 7 August 2008 (UTC)[reply]

We have an article on Microban. Supposedly, it "inhibits the growth of odor and stain causing bacteria", according to the packaging I've seen. I think it's kind of a pointless exercise unless you really dislike the smell of your pencils and/or don't wash your hands regularly. —CyclonenimT@lk? 15:00, 7 August 2008 (UTC)[reply]
Control of bacteria is sometimes critical in health care settings with vulnerable patients such as nursing homes. An object that is used by a large group of people is a good way to transmit germs (door handles are the usual suspect for this problem). The ironic twist to this, of course, is that most medical records must be permanent and indelible and are therefore not written in pencil in the first place.
It's also possibly an advertising gimmick, since many pencil sharpeners are used in schools and defending children from nasty bacteria, well... anything for the children. Too bad it increases the risk of things like MRSA. SDY (talk) 15:20, 7 August 2008 (UTC)[reply]
What I worry more about, is what do all those "anti-microbial" chemicals do to the environment, if they actually work, and if they don't, it's just snake oil. I can't picture it actually doing any good to have your pencil sharpener treated with some chemical when snot-nosed kids are lined up using it one after another. (I don't mean that in a derogatory way, just realisticly.) On the other hand, how would any of us build up any anti-bodies to harmful stuff if we OCD sanitize everything like Mr. Monk? I think it's wise to be prudent either way. Balance, moderation, and just plain old common sense are in order.Kissnmakeup (talk) 16:17, 7 August 2008 (UTC)[reply]
Is it for the benefit of people that suck on the end of their pencil while they think? I think the most likely answer, however, is that it's just a gimmick. I wouldn't buy one. --Tango (talk) 22:47, 7 August 2008 (UTC)[reply]

111-year-old reptile

Today, I read a news story about a 111-year-old reptile (lizard) that mated for the first time in decades. (See [18].) Also, I have previously read about turtles being 200+ years old. (Question A:) How in the world do "they" (scientists) know the age of such creatures? These scientists do not say "approximately" 111-years-old. They give an exact number, as if the birth were actually recorded somewhere. That was also the case on the oldest documented turtle ... it was like 225 years old or something like that. (Question B:) Is there some article in Wikipedia or on the Internet that compares average life spans of various animals (example: gnat = life span of 3 hours; turtle = life span of 125 years, etc.)? (Question C:) In terms of evolution, can anyone see the reason / purpose that some animals live for such extended periods (100+ years)? Thank you. (Joseph A. Spadaro (talk) 15:59, 7 August 2008 (UTC))[reply]

(B) Maximum life span or one of its external links [19], which has a bit more extensive table. Oops, these are max. values, not averages. Clarityfiend (talk) 16:12, 7 August 2008 (UTC)[reply]
Getting down to (C) while avoiding notions of "reasons" for evolution, it's quite clear that an extended lifespan gives a creature more opportunity to produce offspring. There will be more turtles with potentially-200-year-old parents than those with potentially-80-year-old parents, all other things being equal. — Lomn 16:36, 7 August 2008 (UTC)[reply]
To the question (C): in general, yes, longer lifespan may give an animal an opportunity to produce more offsprings; and/or may allow an animal to afford slower metabolism; and/or provide for better survival rates for long spells of unfavorable conditions; and/or allow an animal to rear its young longer so that the offspring survival rate is high; and/or allow for better selection of the fittest individuals before they have a chance to reproduce. Obviously, only some of these apply to any given long-lived organism. Shorter life-span allows for faster metabolism, higher adaptability for unexpected changes in the environment, higher rate of population increase under favorable conditions, etc. Hope this helps. --Dr Dima (talk) 18:08, 7 August 2008 (UTC)[reply]
Thanks. I guess with regard to Question C ... what I was asking was ... why would the period be so long as to be 100+ years? Certainly, these animals can reproduce / survive harsh conditions / etc. in shorter time frames that that? These very long time spans (100+ years) seem excessive, no? (Joseph A. Spadaro (talk) 19:43, 7 August 2008 (UTC))[reply]
Geez. I hope that when I hit 100+ years, my lizard still wants to get up and go for a walk. --Drop Dead Ed (talk) 19:57, 7 August 2008 (UTC)[reply]
Hi. How is it possible that a lizard could mate over 100 years? Was this a tuatara by any chance, because it did say it was endangered, and related to dinosaurs (sphenodontians, to be exact)? Do they have a slow reproductive rate or something? Thanks. ~AH1(TCU) 20:08, 7 August 2008 (UTC)[reply]
Humans are actually fairly unusual in having a menopause, and even that only affects women. Assuming that the lizards and tortoises don't invest a lot of time and energy in raising young, what would be the advantage in them living that long and not being able to reproduce? 79.66.38.215 (talk) 22:56, 7 August 2008 (UTC)[reply]
Astro, yes I'm pretty sure Joseph is referring to a tuatara. In response to the question, it's possible as others have suggestedm Henry's year of birth is known. However I'm not so sure... This (slightly older) source says he was born in the 1880s [20] which doesn't gel with an age of 111. This even older source says he was ~130 in 2001 [21]. It's possible some records were unearthed which showed Henry's likely true age. It's also possible that as part of the PR for the story, the team decided that since according to their estimates, he was at least 111 they might as well say he's 111 since it sounds better. (Or maybe even some in the press did it and it stuck.) Note that these sources describe his mate's age as 70 to 80 [22] & [23] something we may expect if the age is simply an estimate Nil Einne (talk) 14:37, 9 August 2008 (UTC)[reply]
Again, the "why" is problematic. Evolution isn't concerned with motivation. For this case, though, so long as the critter isn't going sterile, its lifespan is tied to a competitive edge in offspring quantity, and thus those genes make up a larger portion of the gene pool. — Lomn 20:45, 7 August 2008 (UTC)[reply]
Living longer is obviously better, all else being equal, since it gives more time for reproducing. The real question is: Why do other animals not live as long? Take a look at Senescence for some possible reasons. --Tango (talk) 22:56, 7 August 2008 (UTC)[reply]
As for part A, the exact ages of many animals are known because they are pets handed down from one generation to the next and the date of birth (or hatching) was noted. There may be older creatures of that species in nature, but maybe not, as they aren't as likely to be taken care of in their old age and live as long. StuRat (talk) 01:18, 8 August 2008 (UTC)[reply]
There's a joke about a fossil being 152 million and 43 years old. Why? Because the museum had been told it was 152 million years old 43 years ago. Maybe the keeper of the lizard was told that the lizard was 50 years old 61 years ago. --Kjoonlee 02:14, 8 August 2008 (UTC)[reply]

Tu'i Malila, Adwaita and Harriet might be helpful for information on old turtles/tortoises. --Kjoonlee 02:20, 8 August 2008 (UTC)[reply]

(C)They're in for the long haul so the tortoise wins the race. Julia Rossi (talk) 03:23, 8 August 2008 (UTC)[reply]
Yes, but only by a hare. StuRat (talk) 20:12, 8 August 2008 (UTC)[reply]

Thanks to all for the above info. It was helpful. Thank you. (Joseph A. Spadaro (talk) 15:47, 9 August 2008 (UTC))[reply]

Why are their bellies cut open?

 ?? —Preceding unsigned comment added by ScienceApe (talkcontribs) 19:34, 7 August 2008

See exploding whale. --Carnildo (talk) 20:49, 7 August 2008 (UTC)[reply]
Well, as the picture is about whaling, that's done because that's what you do with things you're killing for food. Gutting is a pretty necessary process if you're going to eat an animal or prepare it for eventual eating; when you take out the bowels and internal organs, you also take out a lot of the bacteria that'll make the fish (or any animal, really) spoil more quickly. Most of that stuff is inedible anyway. Also, as Carnildo points out, not cutting in a couple of holes for ventilation can also result in the build-up of gases, but I think at that point the animal is going to be pretty much unfit for consumption anyway. -- Captain Disdain (talk) 20:55, 7 August 2008 (UTC)[reply]
Slight correction: They're not fish. —Pengo 22:43, 7 August 2008 (UTC)[reply]
No, I know. I was trying to say that just like fish have to be gutted, so do whales, but that particular point didn't really come through here. -- Captain Disdain (talk) 23:29, 7 August 2008 (UTC)[reply]

Well, there's a Chinese saying:

We eat everything with four legs, except for tables and chairs. We eat everything with wings, except for airplanes and helicopters. We eat everything with fins, except for submarines.

And there's also a Korean saying:

There's nothing to waste from a bovine.

If you try hard enough, almost anything is edible. Whale intestines are consumed for food in Korea. --Kjoonlee 01:53, 8 August 2008 (UTC)[reply]

"You can eat every part of a pig except the squeak" DuncanHill (talk) 10:04, 8 August 2008 (UTC)[reply]
Sure. But you're still going to want to remove the bowels ASAP, what with them being filled with shit and all. Even if you clean them up afterwards and use them in sausages or whatever. Leaving them in isn't going to do the carcass any favors, as far as avoiding food poisoning and parasites goes. -- Captain Disdain (talk) 13:30, 8 August 2008 (UTC)[reply]
I recall hearing about a Hmong dish translated as "doodoo soup" that has apparantly uncleaned bowels. But, then again, I hear a lot of wrong things about the Hmong diet where I'm from. — Ƶ§œš¹ [aɪm ˈfɻɛ̃ⁿdˡi] 22:04, 8 August 2008 (UTC)[reply]
Well, hell, some people really like their dumplings, but I didn't think there'd be enough demand for an actual cuisine based on that. Guess it wouldn't be that much of a surprise, though, all things considered... -- Captain Disdain (talk) 23:58, 8 August 2008 (UTC)[reply]

Vectors

When dealing with vectors (ie displacement, velocity, acceleration, etc.), is it always needed to write the arrow above it? Now I know probably few people do always bother; my question is more asking if there are certain times (ie when dealing with vector components) when writing the arrow would be improper. —Preceding unsigned comment added by 76.68.246.7 (talk) 21:02, 7 August 2008 (UTC)[reply]

As long as you use consistent notation, then technically you can write it however you want. However, marking vectors (with a small arrow, or a boldface font, or a squiggle, or whatever you want to use) can be helpful especially when they're used in expressions with scalar quantities to make sure that you only perform vector operations on vectors and scalar operations on scalars. For example, writing v = xi + yj + zk is much clearer about what's going on than v = xi + yj + zk. Confusing Manifestation(Say hi!) 22:37, 7 August 2008 (UTC)[reply]
Personally, I underline vectors, but whatever notation you use, it's important to use it consistently. You could not mark vectors at all, but if you're going to mark some, you need to mark all, otherwise it will certainly cause confusion. If you don't mark any, people can probably work out from context when something is a vector and when it's not, but it's a waste of effort when you could just have marked them in the first place, and there is a risk of confusion. Also, marking them allows you to use x and x as two different variables - x often means position as a vector and x is the x-coordinate, so x=(x,y,z). If you didn't mark them, that would be extremely confusing. --Tango (talk) 22:52, 7 August 2008 (UTC)[reply]

August 8

Color of Water

What color is it? Some one said on TV tonight it was slightly blue. Is that correct? —Preceding unsigned comment added by 79.76.225.183 (talk) 00:45, 8 August 2008 (UTC)[reply]

Yes. See our cunningly-named article color of water for more information. Algebraist 00:47, 8 August 2008 (UTC)[reply]
Yes I ve just seen it. No more replies needed thank you! —Preceding unsigned comment added by 79.76.225.183 (talk) 00:49, 8 August 2008 (UTC)[reply]

Monitor colors keep changing

The colors on my CRT monitor keep changing very slightly (over a periods of a few seconds), giving pink, pale green or pale blue shades. What could be causing this please? Is it the monitor of the video card? —Preceding unsigned comment added by 79.76.225.183 (talk) 00:54, 8 August 2008 (UTC)[reply]

It kind of sounds like there is a strong but distant magnetic field near your monitor that is turning. When did this start happening? Also, colors changing on a CRT monitor usually means that it is going bad/about to die. --mboverload@ 00:57, 8 August 2008 (UTC)[reply]
I'm not sure how many replies you'll recieve here You may wish to post this at the Computing Reference Desk. —CyclonenimT@lk? 10:24, 8 August 2008 (UTC)[reply]
Also can happen if your cables are a bit loose. --Wirbelwindヴィルヴェルヴィント (talk) 16:44, 8 August 2008 (UTC)[reply]
Is your cell phone near it? They often change power output when checking in. The magnetic field will perturb the electron beams slightly and hitting the wrong parts of the shadow mask. Saintrain (talk) 22:57, 8 August 2008 (UTC)[reply]

I dont mind this thread being mved to computing if someone can do it —Preceding unsigned comment added by 79.76.208.36 (talk) 22:59, 8 August 2008 (UTC)[reply]

Anywhere on earth?

Is there any where on earth where your urine can freeze before it hits the ground (assuming your are standing up when releasing it) —Preceding unsigned comment added by 79.76.225.183 (talk) 01:03, 8 August 2008 (UTC)[reply]

No. That's a silly myth / hyperbole. The density and thermal conductivity of air is so low that even if its temperature was absolute zero it wouldn't be able to freeze your urine in the ~1 second it takes to reach the ground. Dragons flight (talk) 01:07, 8 August 2008 (UTC)[reply]
How about if I wizz off a mountain ? :-) StuRat (talk) 01:11, 8 August 2008 (UTC)[reply]
Trust StuRat to be always thinking outside the box. I often thought he was out of his box anyway ;) —Preceding unsigned comment added by 79.76.225.183 (talk) 01:16, 8 August 2008 (UTC)[reply]

The thermal conductivity of urine should be considered. If a man is relieving himself off the edge of a very high cliff, as the urine descends a tube of frozen urine would form around a liquid core. After a certain amount of fall, the urine would become a solid rod. The rod would continue to fall, with a tube above it. The walls of the tube would become thicker in the downward direction until a sold rod is formed. This assumes that the fall is large enough and the temperature is low enough. This also assumes there is no turbulence within the urine stream. AndMe2. 66.52.8.251 (talk) 04:55, 8 August 2008 (UTC)[reply]

I could maybe start to believe what Andme2 is suggesting if he could explain how to whizz off a high point and keep the liquid in a stream. My experience (not extensive you understand) is that the stream breaks up into droplets not unlike rain, as any liquid poured in a thin stream from a height. Adding the rider 'this assumes no turbulence' is not practical in the proposed scenario. Richard Avery (talk) 07:07, 8 August 2008 (UTC)[reply]

Well let's assume it is a stream of water coming from a very large hose pointing downward. I wonder how large the hose would have to be, and what the flow rate would have to be, to prevent the water from breaking into drops. Or would it always break up into drops if it falls far enough? Maybe there is a principle here about falling water that I don't know about. As for turbulence, a basic principle could assume no turbulence. This would be modified to account for practical causes of turbulence in any particular case. (For instance, according to the "law" of gravity, a feather will fall as fast as a lead weight. This principle is modified to account for air resistance in relation to weight.) AndMe2 66.52.8.251 (talk) 08:03, 8 August 2008 (UTC)[reply]

May not qualify as somewhere "on Earth", but see these news stories: [24], [25]. Gandalf61 (talk) 09:18, 8 August 2008 (UTC)[reply]
What about the freezing point depression associated with the wastes dissolved in urine. How much solute is in urine?Coolotter88 (talk) 13:00, 8 August 2008 (UTC)[reply]
These videos suggest that it's totally possible. --Shaggorama (talk) 18:25, 8 August 2008 (UTC)[reply]
On back of an envelope I'd estimate a drop of water at 30 C would need to fall about 10,000 times it's diameter in order to freeze in air of -30 C. So yes, if you toss water in the air, then you will gets some frozen mist as very tiny particles of spray freeze (< 0.1 mm, typically), but most of the water will nonetheless reach the ground as liquid. To answer Stu's question, that suggests you'd need to urinate off a 20+ m cliff before a substantial volume would freeze during the fall. BTW, there is also a Mythbusters episode where they show liquids decidedly not freezing as it falls through several feet of deeply chilled air. Dragons flight (talk) 19:24, 8 August 2008 (UTC)[reply]
Hi. However, I've heard in some places spit freezes before it hits the ground, or your breath freezes into ice crystals, but there's less mass involved, and thus less heat. Thanks. ~AH1(TCU) 23:31, 9 August 2008 (UTC)[reply]

Paradox?

A train is traveling at 100mph in a westerly direction. A small fly is traveling at 20mph (wrt to the ground) in an easterly direction along the rail track. At the moment of impact, the fly's velocity is instantaneously changed from +20 to -100 mph. At the precise moment of change of velocity, the fly must be at rest wrt ground. But since fly is in contact with train, train must also be instantaneously at rest. So, does the fly stop the train? —Preceding unsigned comment added by 79.76.225.183 (talk) 01:46, 8 August 2008 (UTC)[reply]

One second before contact, at that precise moment, must the fly be at rest with regard to the ground? --Kjoonlee 01:56, 8 August 2008 (UTC)[reply]
The error is in assuming the change is instantaneous and that the fly and the train each have a single velocity. The change will be gradual due to deformation of the fly and the train. That deformation resolves the apparent paradox - parts of the fly and parts of the train will be travelling at varying velocities. Each part of the fly will, at some point, be stationary, which I guess means some tiny amount of the train will also be stationary for a time, but that's perfectly possible if you allow for deformation of the train. --Tango (talk) 02:05, 8 August 2008 (UTC)[reply]
The problem with this 'paradox' is that it can't decide how physically realistic it's being. If it's being realistic, then the train and fly are not particles, they undergo deformation, and the change is not instantaneous, as Tango points out. Thus there is no paradox in this case. If, on the other hand, we're being very unphysical, and model the train and fly as particles, unable to deform, then the change of velocity is instantaneous, and it makes no sense to ask what the velocity of the fly is at that moment; it simply isn't well-defined in this crude model. Again, there is no paradox. The paradox arises if we unphysically model the fly and train as particles and then impose the physical condition that the fly, in changing velocity, must go through the velocities in between. Algebraist 03:05, 8 August 2008 (UTC)[reply]
Each atom of the fly is at some point stationary because its velocity has to change sign. No part of the train ever needs to come to rest. Someguy1221 (talk) 04:17, 8 August 2008 (UTC)[reply]
Are you sure? At the moment when the part of the fly in contact with the train is at rest, can that bit of train be moving? I suppose so, if we accept that "in contact" actually still involves a small separation, held apart by electromagnetism. Again, it's a matter of exactly how physical we're being. --Tango (talk) 02:53, 10 August 2008 (UTC)[reply]

The statement of the problem speaks of "the moment of impact". A moment, as the word is used here, means a point in time of zero duration. So yes, both the train and the fly are stationary for that zero duration. AndMe2. 66.52.8.251 (talk) 04:29, 8 August 2008 (UTC)[reply]

Also in a real world situation the train is pushing air out of its way, and this air will greatly slow the fly to a stop and declerate it almost very close to the trains speed well before the fly hits the train. The deformation would amount to a microscope region of metal or glass being tensioned. No part of the train would necessairly need to come to rest, if it was inside a vacum this would happen and the impact would be more severe.--Dacium (talk) 04:43, 8 August 2008 (UTC)[reply]
At the moment of change in velocity, the velocity wrt the ground is not zero. Instead, it cannot be calculated because both the distance travelled and the time are zero and 0/0 is indeterminate. --Bowlhover (talk) 04:50, 8 August 2008 (UTC)[reply]
Which is why you would take the limit as delta t approaches zero - giving the instantaneous velocity, dx/dt. So the train does not need to come to a stop, even momentarily. PhySusie (talk) 12:22, 8 August 2008 (UTC)[reply]
Ummm, are these 1-D trains and flies? Up and down and right and left, the directions the fly-parts spread, are also WRT the ground. Saintrain (talk) 22:38, 8 August 2008 (UTC)[reply]
PhySusie: The limit does not exist when, as in the OP's scenario, the change in speed is instantaneous. It's meaningless to ask for the velocity at the moment of change because it's not defined. --Bowlhover (talk) 07:07, 9 August 2008 (UTC)[reply]

Thermocouples

The flame failure devices in gas fires are usually called thermocouples, but how can a thermocouple act to shut off a gas valve in the absence of an amplifier? Are they, in fact, some other sort of heat activated mechanical device by any chance? —Preceding unsigned comment added by 79.76.225.183 (talk) 01:58, 8 August 2008 (UTC)[reply]

I think it works like an ordinary thermocouple in that when a heat is applied to one end of the thermocouple a Seebeck voltage is generated. This generated voltage will be used to activate a solenoid valve. Seebeck voltages are typically small but should be enough to power a sensitive electromagnet. Jdrewitt (talk) 09:51, 8 August 2008 (UTC)[reply]
Thermocouple voltages are in the millivolt region, so how is that enough to activate a solenoid? —Preceding unsigned comment added by 79.76.149.61 (talk) 12:16, 8 August 2008 (UTC)[reply]
According to Thermocouple#Heating_appliance_safety, the sensor is a thermopile (effectively lots of thermocouples in series), also called a "pilot generator". It generates 750 mV [26], which is enough to operate a standard "millivolt gas valve". Other systems use a single thermocouple and an amplifier. --Heron (talk) 16:45, 8 August 2008 (UTC)[reply]
Aha! Do you have any links to millivolt gas valves? —Preceding unsigned comment added by 79.76.208.36 (talk) 20:50, 8 August 2008 (UTC)[reply]
You could look at the Honeywell VS820. There's a link to the PDF data sheet at the bottom of the page. --Heron (talk) 21:17, 8 August 2008 (UTC)[reply]

Why does quantum mechanics only work on a small scale?

And what is the cutoff point? One atom = Spoooky! Two atoms? Hello, Newton!Hey, I'm Just Curious (talk) 04:13, 8 August 2008 (UTC)[reply]

Quantum mechanics applies to all objects, but for most purposes, Newton's laws are sufficiently accurate. To build a rocket, for example, it would not be necessary to calculate the wave functions of every atom in every component. Not only is that level of accuracy not required, but parts cannot be engineered with such precision. --Bowlhover (talk) 05:02, 8 August 2008 (UTC)[reply]
See also: correspondence principle. Quantum mechanics would not have been accepted as a sound theory if it conflicted with the results of classical mechanics. It's a common misconception that quantum mechanics somehow invalidates classical mechanics. It does not; rather, it (among other things) changed our view of the scope of classical mechanics and electromagnetics. -- 98.26.182.245 (talk) 05:38, 8 August 2008 (UTC)[reply]

I mean like, supposedly there's a "wave function collapse" or something like it when you look at the results of the photon double slit thing. That's one photon. Is the cutoff point where this nonsense(?) stops 17 photons at a time?, 8,099 photons at a time, or what?

Someday I'll ask you about Schrodenger's cat.--Hey, I'm Just Curious (talk) 06:05, 8 August 2008 (UTC)[reply]

Well, there is microphysics (subatomic physics), macrophysics (Newtonian physics, and gigantophysics (stellar physics). They're all different. But there is some overlap. For instance, electrons, apples, and galaxies all obey the "law" of gravity - they have mass. AndMe2 66.52.8.251 (talk) 06:22, 8 August 2008 (UTC)[reply]

The cutoff is thermodynamic irreversibility, basically. The essence of quantum mechanics is that you get wavelike interference between different "histories", i.e. different ways that you might have arrived at the same final state. To put it another way, you get quantum effects when which-way (welcher Weg) information is lost. This can't happen to thermodynamic objects because they're constantly emitting blackbody radiation, and that radiation contains information about the object's path (not necessarily in a recoverable form, but that doesn't matter for the purposes of quantum mechanics). You sometimes hear that large objects exhibit double-slit interference in principle but with such a small wavelength that it can't be observed, but that's not true; thermodynamic objects don't self-interfere at all because they broadcast information about themselves on a timescale shorter than the time it takes to do the double-slit experiment.
However you don't need thermodynamic irreversibility for the double-slit pattern to disappear; leaving any which-way information behind is enough for that ("any" meaning at least one quantum, I think). If you have a device at one slit that records the passage of a photon in a thermodynamically reversible way, say by two stable states of a molecule, the interference pattern will still disappear. The difference is that you could in principle unmeasure that information, e.g. by routing the photon through the slit a second time and reversing the measurement process, in which case the interference pattern would reappear. It would be a very difficult experiment but it's not actually forbidden by the second law of thermodynamics. It's when thermodynamic irreversibility kicks in that you can safely stop worrying about quantum interference, because the second law guarantees that you won't see it. This is, incidentally, why it's so hard to build quantum computers: they have to operate in a state of perfect thermodynamic reversibility. (More accurately, it's only the "computational degrees of freedom" that have to, but that's hard enough.)
There might also be an unknown physical process that kicks in before thermodynamic irreversibility and has the same effect of "permanently recording the which-way information". If so, we should be able in principle to detect it and find out when it happens. On the other hand, if the process kicks in after thermodynamic irreversibility then there's no way to experimentally determine when it happens or indeed whether it happens at all. People argue about whether it's logically coherent to never resolve the real path a particle takes, but you can't extract any experimental predictions out of that disagreement. Well, some people think you can. But at any rate it's a philosophical debate at the moment, not a scientific one. -- BenRG (talk) 12:37, 8 August 2008 (UTC)[reply]

Regardless of everything above quantum effects using fullerenes and complex biological molecules have been shown by Zeilinger and his team: Anton Zeilinger#Atom and macromolecule interferometry. --Ayacop (talk) 14:14, 8 August 2008 (UTC)[reply]

cooling/heating vortex

Does anyone know the name of the vortex shaped device where you put in compressed air and cold air comes out one side and hot air out the other side and it has no moving parts or power. And can they work on water?--Dacium (talk) 04:38, 8 August 2008 (UTC)[reply]

AFAIK water cannot be compressed substantially, so if device works on expansion of compressed air, then it wouldnt work with water. —Preceding unsigned comment added by 79.76.221.121 (talk) 04:41, 8 August 2008 (UTC)[reply]
It's not vortex shaped, but is it a vortex tube? --Bowlhover (talk) 06:30, 8 August 2008 (UTC)[reply]

Do any types of dementia result in loss of tact

I have noticed that many people as they age loose the social limits most people put on their communications. This results in older people being much more likely to make insulting statements that 1) they don't seem to notice they have made and 2) would never have made in their earlier years. I wondered whether this is a form of dementia or loss of societal imposed constraints. —Preceding unsigned comment added by 202.92.75.130 (talk) 05:11, 8 August 2008 (UTC)[reply]

Some old folks become crabby - many things irritate them. They respond by involuntarily becoming unpleasant. Perhaps that response is instinctive - it is a way of saying "Don't tread on me". Some sociologists claim that human beings have only two instincts - fear of loud noises and fear of falling. I think we are loaded with instincts. They all played a part in survival in the stone age, and became wired in. Instead of being a conscious action, the response became a knee-jerk reaction. That played a part in survival. They didn't have to learn the hard way, by experience. But to return to the original subject, some old folks become easily irritated, hence the instinctive response. It is instinctive self-defense. AndMe2 66.52.8.251 (talk) 06:05, 8 August 2008 (UTC)[reply]

I guess it needs to fit with other indications to know if it is specifically part of dementia, but I've observed it as coming from people with onset of dementia (later to become dotty or mildly violent) and others who are just plain frustrated with getting old and dependent reacting in a crabby way. I wonder if that's also a part of dementia, the social forgetting. Crabby ones could be reverting to type under pressure. Sweet ones probably always were sweet. Googling is tricky because "insult" is used as an effect (affect?) in medical terminology for this condition. Julia Rossi (talk) 06:19, 8 August 2008 (UTC)[reply]
Many age-related dementias do present with personality changes, including labile affect and extreme irritability. (See our article on dementia, and the links therefrom to Alzheimer's disease, frontotemporal dementia, and others.) Of course, not all crankiness is due to dementia—some people may be suffering from undiagnosed depression or other mental illness, while still others may just have a mean streak. TenOfAllTrades(talk) 13:31, 8 August 2008 (UTC)[reply]
So which of the above is bothering you? —Preceding unsigned comment added by 79.76.208.36 (talk) 22:52, 8 August 2008 (UTC)[reply]
Them insulting people may not be intentional, they may have lost the ability to know which statements will be taken as insults. This may also be true of a small child, for example, who, when introduced to a 100 year old, may ask "why aren't you dead, yet ?". StuRat (talk) 15:21, 8 August 2008 (UTC)[reply]
Hey, you kids! Get off my reference desk! Deor (talk) 15:58, 8 August 2008 (UTC)[reply]
Well, yes, Julia. I agree that some people are naturally sweet and others are naturally nasty. Saints or devils. The great majority of people are somewhere in between. Perhaps there is a bell curve for this innate characteristic. National character - a learned set of responses - also plays a part. I think you live in Paris, but I will say this anyway. Some of the French have a learned tendency to be nasty, but others are very pleasant. Perhaps the bell curve for the French is even inverted - like a shallow bowl - because of these learned national habits. The bowl is shallow because behavior taught by example does not easily overcome natural characteristics. Speaking of all nations, there is a gender bias to it as well. Women obviously tend to cluster somewhat toward the right side of the sweet/sour bell curve. (Though a few women are far over on the left!) For both genders, I think repeated past personal experience with others plays a part in it as well. From time to time, people also temporarily shift their usual position on the curve in response to outside circumstances. This could be involuntary, like a fit of rage, or voluntary as a conscious decision. Also, talented people tend to be unpleasant. They are sensitive to very faint signs, like an expert tracker. But the gross experiences of life are very irritable to them. Many, many examples could be given, in all walks of life and in all professions, of talented people who are S.O.B.'s. As exceptions, talented artists (such a painters) do not seem to be unduly crabby. However as exceptions to these exceptions, talented opera singers are sometimes prima donnas. Quite a lot of talent goes unused because irritability causes opportunities to be denied - other people don't want unpleasant persons around. AndMe2 67.150.190.106 (talk) 15:44, 8 August 2008 (UTC)[reply]
I have been told (apropos a grandparent who tended to disrobe whenever unattended) that social constraints and inhibitions are among the first learned behaviours to disappear in some types of age-related dementia or other severe stress: physical and verbal agression are two such behaviours normally controlled by social rules. Comments made under such conditions tend neither to be nice nor nasty in tone; the affect is frequently missing and statements, however unpleasant the contents, are made flatly, as fact. ៛ Bielle (talk) 16:34, 8 August 2008 (UTC)[reply]
I seem to recall that social inhibitions vary in strength as you age anyway, being strongest when you're about 14 (if I recall correctly) and weakening after that as you age. So some of this is just a natural result of getting older. 79.66.38.215 (talk) 17:20, 9 August 2008 (UTC)[reply]
@ 67.150 -- appreciate your thoughtful reply. Can relate to much about creative types and their polarities. Confidence and insecurity comes into it too I guess. From age 14 you say 79.66.38 -- there must be a precipitous drop in the graph from 14 into the teens -- that's scary. Julia Rossi (talk) 10:16, 10 August 2008 (UTC)[reply]

Physics of boat towing

Hello,

This question concerns the physics involved in towing a boat.

The scenario is a tug boat towing an oil tanker.

One argument is that at the moment t when the tug boat is actually pulling the tanker, the towing cable will be perfectly straight.

The other argument is that even at the moment t when the tug boat is pulling the tanker, the towing cable will be sagged, given the forces of gravity and friction acting on it.

I am also wondering if the shape of the cable changes when the tanker and the tug boat are maintaining the same speed and when the tug boat accelerates (the acceleration then being transmitted to the tanker through the cable).

how does it work?

if anyone could illustrate this with graphics, it would be perfect.

thanks! —Preceding unsigned comment added by 194.167.55.204 (talk) 08:17, 8 August 2008 (UTC)[reply]

The shape of a uniform cable secured at both ends and hanging under its own weight with constant horizontal tension is called a catenary. The shape of the catenary is determined by the ratio of the horizontal tension to the weight per unit length of the cable - the larger this ratio, the shallower the catenary. If the tug accelerates, the horizontal tension increases, the ratio increases, and the catenary curve becomes shallower. If the horizontal tension is very large compared to the weight per unit length of the cable (i.e. you have a very large force or a very light cable) then the catenary may become so shallow that it is indistinguishable from a straight line. Gandalf61 (talk) 09:08, 8 August 2008 (UTC)[reply]
A comment on the "indistinguishable from a straight line" part. This may well be true when viewing the cable from the side. However, if you look down the length of the cable from one end to the other, even a very subtle deviation from a line becomes quite visible. StuRat (talk) 15:15, 8 August 2008 (UTC)[reply]

-- Thank you for this very precise answer. —Preceding unsigned comment added by 194.167.55.204 (talk) 11:01, 8 August 2008 (UTC)[reply]

Also note that the shape will somewhat distort if the cable droops down into the water. The additional buoyancy will push that section upwards slightly from where it would otherwise fall. Then, when lifted out of the water, the additional weight of the water will make it lower than it would otherwise be, until all the water drips off and dries up. StuRat (talk) 15:13, 8 August 2008 (UTC)[reply]
The water does not just drip and dry. Instead, the tension in the cable causes the strands of the cable to tighten relative to each other, which forcefully expells a lot of water (at least for hemp cables, and at least according to Patrick O'Brian.) Also tension will lengthen the cable, which changes the mass/length. -Arch dude (talk) 14:21, 10 August 2008 (UTC)[reply]
Apparently William Whewell's 'Elementary Treatise on Mechanics.' (1819) contains the accidentally-poetic lines: "...no force, however great/Can stretch a cord, however fine/Into an horizontal line/Which is accurately straight." Whewell was not amused when this was pointed out and changed the text in the next edition. Links to more detail here AndrewWTaylor (talk) 16:50, 9 August 2008 (UTC)[reply]

Water to Ice

Why is it that water increases its volume when it turns to ice? Minor Contributer (talk) 08:40, 8 August 2008 (UTC)[reply]

Our article on ice says that water expands on freezing "due to hydrogen bonds forming between the water molecules, which line up molecules less efficiently (in terms of volume)". Gandalf61 (talk) 08:52, 8 August 2008 (UTC)[reply]
Also Bismuth expands slightly when cooling from a melt —Preceding unsigned comment added by 79.76.149.61 (talk) 12:19, 8 August 2008 (UTC)[reply]

Permeability of plastic

Anyone know where I can find data on high permeable plastic films? Particularly ones that will allow o2 and CO2 to pass? For instance, how permeable (perms?) is cellophane (plastic wrap)? —Preceding unsigned comment added by 128.113.56.182 (talk) 13:28, 8 August 2008 (UTC)[reply]

The relative permeability of most plastics is 1. :) -- Katavothron (talk) 13:35, 8 August 2008 (UTC)[reply]
Thats great, though some what outside the scope of my question. :P —Preceding unsigned comment added by 128.113.56.182 (talk) 13:51, 8 August 2008 (UTC)[reply]
Semipermeable membranes are often made of plastic. Rmhermen (talk) 14:26, 8 August 2008 (UTC)[reply]
Contact_lens#By_constructional_material, and links therein, may be of some use regarding oxygen permeability. -- Coneslayer (talk) 15:32, 8 August 2008 (UTC)[reply]
Dialysis tubing is pretty permeable stuff. According to this study it's certainly permeable to CO2 and probably O2 as well (the study reports negligible change in [O2], but my guess is that was due to the strength of hemoglobin). --Shaggorama (talk) 18:05, 8 August 2008 (UTC)[reply]
Yeah, we used the stuff for a biology GCSE practical. —CyclonenimT@lk? 19:11, 8 August 2008 (UTC)[reply]

I'll tell you where I look for such info. My workplace has a subscription to Knovel, which gives us online access to many technical reference books. There are books in this collection that have exactly the type of data you are seeking, I believe. Sorry I cannot point to an "open" source. ike9898 (talk) 13:20, 9 August 2008 (UTC)[reply]

Beta plus decay and conservation of mass

What am I missing? A proton decays to a more massive neutron, and in the process ejects a positron, losing mass, AND 0.96 MeV of energy? -- Aeluwas (talk) 14:52, 8 August 2008 (UTC)[reply]

See Talk:Positron emission#Error?. -- Coneslayer (talk) 15:03, 8 August 2008 (UTC)[reply]
Thanks, but, um... The two article mismatch. Positron emission (and Carrionluggage on the talk page) says energy comes out of the decay (on the right side); Beta decay says energy is used (on the left side). Which one is it? :)
I'm guessing the beta decay article needs editing? -- Aeluwas (talk) 17:20, 8 August 2008 (UTC)[reply]
According to this link, among others, the beta decay article is wrong. I'll get to it, then. :) -- Aeluwas (talk) 17:24, 8 August 2008 (UTC)[reply]
Actually, I'll leave it to someone else. I can't correct the sentence after the equation (which would have to be part wrong for the article to be wrong). -- Aeluwas (talk) 17:37, 8 August 2008 (UTC)[reply]
I think the positron emission reaction "takes" energy from the binding energy of the parent nucleus, and some of that energy ends up as kinetic energy in the outputs. So whether the energy term is on the left or the right depends on whether you count the parent nuclear binding energy as being an input or not. I never took nuclear physics, though, so hopefully someone else can comment. -- Coneslayer (talk) 17:45, 8 August 2008 (UTC)[reply]
Yes, Beta decay#β+ decay is describing the case of an isolated proton, which requires energy input, while Positron emission is describing the decay of a carbon-11 nucleus, which doesn't. I added a bit to the Positron emission article to clarify this. The proton and neutron masses that are usually quoted only make sense for nucleons in isolation. The mass of a nucleus is less than the sum of the masses of the constituent nucleons considered as free particles (except of course for hydrogen-1 and administratium-1, where it's equal). -- BenRG (talk) 20:17, 8 August 2008 (UTC)[reply]

Water for the West

Wondering why it wouldn't be possible to increase rain fall in the West by pumping sea water into the many large dry lake beds. They already contain salt, so that shouln't be a problem. 129.112.109.253 (talk) 15:33, 8 August 2008 (UTC)[reply]

The volume of pumping required, were the rest of the scheme reasonable, renders the whole thing moot. You're talking hundreds of trillions of gallons of water per year. — Lomn 16:02, 8 August 2008 (UTC)[reply]
However, as Death Valley is below sea level, one could theoretically connect a siphon from the Pacific Ocean and have the water flow by force of gravity alone. The actual effect on local climate would likely be negligible though; probably not enough to justify the cost of constructing (and maintaining - salt water is very corrosive to both concrete and metal) the 400+ km long pipe (about one third of the length of the Trans-Alaska Pipeline). -- 128.104.112.147 (talk) 17:36, 8 August 2008 (UTC)[reply]
What you're suggesting has actually sort of happened already (by accident). An engineering failure caused the Colorado river to be diverted into the Salton sink, resulting in the formation of the Salton Sea. It's the largest lake in California, but its existence doesn't make socal any less of a desert. It takes more than a lake to make rain. --Shaggorama (talk) 17:58, 8 August 2008 (UTC)[reply]
It's an interesting concept, and it could work, provided you can feed ocean water into a below sea-level area using gravity. However, the resources needed to do this might be better spent on a desalination plant to provide fresh water directly. StuRat (talk) 18:28, 8 August 2008 (UTC)[reply]
Totally possible if you wanted to dig a canal from the ocean to Death Valley in California. It would make quite a nice lake. There are easier ways to get filtered water, however. --mboverload@ 19:11, 8 August 2008 (UTC)[reply]
I commute across Death Valley a dozen times a year, and one of our topics of conversation (the drive gets a bit long...) is "What if the Sierra Nevada sprung a leak and this all suddenly filled up to sea level?" --jpgordon∇∆∇∆ 16:57, 9 August 2008 (UTC)[reply]
I believe there were plans some time ago to do this sort of thing with a huge pipeline through the mountains in North Africa. More recently they've discovered water under the sand and have begun extracting it - I don't know what a huge saltwater lake would have done for that. Dmcq (talk) —Preceding undated comment was added at 16:08, 10 August 2008 (UTC)[reply]

Too Many Magnetic Minerals in me Coffee

I like me coffee. Sometimes though, depending on where I get it. Maybe the Volcanic soil is higher in Iron or something else, I'm not sure what, but some Magnetic, or Electromagnetic reaction activation sequence in my coffee mug. Basically, if I'm drinking my coffee, and lots of it, and listening to my radio driving by an electro field called the highways and streets, in Los Angeles, there are power lines galore. And/or if I'm listening to my Ipod, especially ecstatic. I feel like my brain is out of wack. My concentration is taken away and focusing on anything for a long period of time is hard work. Like reading a simple book. I could give up all these things and live the secluded life, or I could find some sort of fix. I'm thinking of rigging something up in the kitchen. My question is this: Could I, when boiling my water, put a pretty good sized magnet on the side of the pan minimize ingestion of Magnetic particles? I mean, would this help or make it worse? I'm thinking the magnetic particles would stay in the pan. Or would it just activate them and make them even more magnetic? Regarding the ipod. I feel like I'm doing that old Electromagnetic Therapy shock stuff they do to people in the Psych wards. In a mass population, I see how this can help in a pessimistic attitude. Instead of healing the Mentally Ill, just make the average citizen mentally unstable, so in a sense, the mentally ill will not be alone. Balancing the averages I guess. Anyways, if any scientists here know of great fixes for this silent epidemic of policy/protocol/politic in control theory, I'd greatly appreciate any helpful advice. Cheers, --i am the kwisatz haderach (talk) 16:29, 8 August 2008 (UTC)[reply]

Wow. Are you sure you're not just feeling the effects from the caffeine? -- MacAddct  1984 (talk &#149; contribs) 17:02, 8 August 2008 (UTC)[reply]
I agree with MacAddct; this is probably the most bizarre question I've ever read here, and I've read some doozies. I have something to tell you that might help, or it might make your freak out even more. If you're worried about "magnetic particles", you shouldn't be worried about your coffee, but your breakfast cereal. Many breakfast cereals contain an ingredient called "reduced iron" which is actually metallic iron particles, which you can extract with a magnet. See [27]. They're perfectly harmless though; your stomach acid oxidizes the iron and after that it's not ferromagnetic any more. 128.165.101.105 (talk) 18:41, 8 August 2008 (UTC)[reply]
"Sometimes though, depending on where I get it." Is that an actual pattern ("often happens with Store A coffee, never with Store B"), or is this really two unrelated variables (you get it from different places, and there's no relationship between where you get it and the occurrence of...this effect). DMacks (talk) 18:56, 8 August 2008 (UTC)[reply]
1. Electromagnetic fields are not trying to kill you. 2. Don't take this the wrong way but many things can cause the symntoms you describe. Heck, you could be right in some sense and have heavy metal poisoning. Who knows? We can not give medical advice so all I advise you to do is see a doctor and tell him your feelings. The only thing I drink is highly filtered water so I can assure you I am not under their control. --mboverload@ 19:09, 8 August 2008 (UTC)[reply]
I think something other than your coffee needs to be checked. :) Just kidding. In all seriousness, it is probably the effects of the caffeine rather than magnetic iron dissolved in your coffee. bibliomaniac15 19:12, 8 August 2008 (UTC)[reply]
That too. Caffine IS a drug, after all. And drugs have side effects. --mboverload@ 19:16, 8 August 2008 (UTC)[reply]
Thanks. I see that my question isn't structured properly. A quasi-subconscious-metaphor-ishness. And it is because the information I'm getting is based on personal collection, like a stamp, coin, or baseball card collection. LSD. No, I'm not on it. Basically, I test certain markets. Using my body as a lab. A Consumer Report in Biology of Chemical Engineering and the FDA. I'll go to different demographic neighborhoods and drink or eat only these types of drinks or food. I was using coffee as an example. Lets say for a week I'll drink only Arrowhead water. Then I'll switch it up and drink a bunch of Coca-Colas, or Starbucks. Hence, Coffee the example in research questioning. This may deal with Soundwaves and Light. For instance, drinking a cup of jo, then going to see 'The Incredible Hulk'. Maybe the industrial sound, lights, and magic (what's in the cup) can give me a headache. Maybe it's just the caffeine. I'm just thinking of a GamesTheory on research done on human beings. Unfortunately much of the psychological drug treatment is done on mentally ill peoples, so when using this data to control a mass population, it could lead to an entire population of people being treated as you would that of an insane person. Possibilities people, I'm not saying that this is happening or that I'm a conspiracy theorist. My question is it possible to a) put some sort of additive into food supply, b) add some sort of electrowave/pulse into a targeted environment, like an Electro Magnetic Pulse, or like an Electric Grid you use to keep your dog in your yard, c) something to the effect of Transcranial_magnetic_stimulation on a Mass Population? Lets not bandy insane/sane questioning, and/or whether this is implemented in any cities in the world, but the question-thought should be looked at as--Could we do this, and if so, HOW? Cheers, --i am the kwisatz haderach (talk) 20:14, 8 August 2008 (UTC)[reply]
You need to separate your questions then. If you are talking about your practical experience, the best thing to do would be to start documenting your own experience in a notebook. Maybe a pattern will emerge if you keep records. If you want to ask a theoretical question, maybe you should start a new thread. But to answer your last question - yes, yes and yes. But not with any technology we possess today or are likely to have in the near future. Franamax (talk) 20:46, 8 August 2008 (UTC)[reply]
I'm going to try to respond to your questions in turn. First, A) there are lots of additives in your food already. For instance, see Water fluoridation. B) you answered your own question with the dog fence example, but it's worth noting that the mechanism of adog fence and an EMP are incredibly different. An EMP a wide range magnetic pulse that is a side effect of a high altitude nuclear blast that wipes out the activity of electronics in a large area. An electric dog fence is a perimeter that causes an electric dog collar the activate and shock the dog if it is crossed. EMPs can't be used for population control beyond interfering with electronics, and electric fences need their targets to wear a device to control them. C) TMS is focused on isolated areas of the brain, so there isn't really any reason to fear "TMS on a mass population" because there is no way to perform such a focused procedure on a wide area unless everyone was wearing some peice of equipment on/in their heads that would permit it.
In general, I think your fears are largely unwarranted and you should further research the effects of electromagnetic (more specificaly magnetic) energy on the body and brain. If you are interested in how large organizations manipulate the psychology of wide populations to their benefit, you should read up on advertising, propaganda, and subliminal messages. If you want to protect yourself from the influences of others on your mind, you would be more effective turning off your TV and closing your newspaper than putting a magnet against your coffee maker. --Shaggorama (talk) 14:24, 10 August 2008 (UTC)[reply]

Structure of liquid water

I was just watching 'The enemies of Reason', the documentary by Richard Dawkins and someone in it, a proponent of homeopathy, said that no one knows the structure of liquid water. Now since no-one picked him up on this, I would be lead t believe this is true but I thought I should best check here before I commit myself to it. Is it true? 92.4.189.176 (talk) 19:41, 8 August 2008 (UTC)[reply]

No, it's not true. The structure is one oxygen atom with two hydrogen atoms joined by single bonds at an angle of 104.45° to each other. Perhaps they are talking about the types of bonds between water molecules in the liquid state ? This is primarily hydrogen bonds. See water#Chemical and physical properties. StuRat (talk) 20:03, 8 August 2008 (UTC)[reply]
I assumed it was something more complex than that because the man who said it was a doctor of rheumatology and I learnt about bond angles and hydrogen bonding for my A level. Thanks anyway. 92.4.189.176 (talk) 20:06, 8 August 2008 (UTC)[reply]
Water molecules do form large-scale networks of hydrogen bonds that are quasi-stable and this is an area of ongoing research. No connection yet to homeopathy. Franamax (talk) 20:58, 8 August 2008 (UTC)[reply]
The homeopathy bit was irrelevant; I was just trying to set the scene in the hope that someone else may have watched it. 92.4.189.176 (talk) 22:18, 8 August 2008 (UTC)[reply]
You're probably referring to water memory. Useful if you need to store some data for 50 femtoseconds. --Heron (talk) 21:07, 8 August 2008 (UTC)[reply]
Actually, the claims are perfectly correct. The structure of liquid water is indeed an on going area of research. It is correct that the local structure is "one oxygen atom with two hydrogen atoms joined by single bonds at an angle of 104.45° to each other" (Having said that, the bond angles and interatomic distances are sometimes disputed). However since water is a liquid, it has a disordered structural arrangement and so the atomic configuration of water at long range or even intermediate length scales (anything beyond around 0.5 - 1 nm or so) is not easily known or predicted, since it is not simply a periodic arrangement of basic structural units as in a crystalline solid. The basic structural units (water molecules H2O) are instead arranged in a disordered (but not neccessarily random) manner. There are many studies of liquid water by the method of neutron diffraction. However, this is an extremely challenging technique to apply to liquids and amorphous materials since the atomic correlations become so diffuse at extended length scales. Methods such as isotopic substitution in neutron diffraction may help to yield these atomic correlations but this research is still on going and there is still much controversy over the intepretation of the results. The same applies to many liquids, amorphous solids and glasses since the experimental techniques available to obtain the atomic structure of materials (e.g. neutron and x-ray diffraction) are still extremely challenging when applied to non-crystalline materials. Ultimately any theoretical models on the structure of water must be based on definitive experimental data which is not yet available...I'm taking a wikibreak so might not check this page but if you are interested in getting some more information on the extent of current understanding on the atomic structure of water and the current experimental limitations then please leave a message on my talk page and I could point you to a few references. Jdrewitt (talk) 21:41, 8 August 2008 (UTC)[reply]

August 9

Mosquitos

If a mosquito can spread malaria and West Nile virus, why can't it spread AIDS? 99.245.92.47 (talk) 00:52, 9 August 2008 (UTC)[reply]

Because the HIV is digested by the mosquito. Unlike the other viruses and mosquito-borne parasites, HIV hasn't evolved the mechanism to avoid the insect's stomach. See here.--Lenticel (talk) 02:23, 9 August 2008 (UTC)[reply]

Botany(project topic)

Good day Pls I want to know growth of garlic (Allium sativum) is affected by temperature variables of 28 degree and 37 degree.When grown in Bauchi,North West of Nigeria;West Africa and also, method of observing this changes Thanks —Preceding unsigned comment added by Rukima (talkcontribs) 01:07, 9 August 2008 (UTC)[reply]

This sound like an interesting topic. I assume that you intend to do long-term growth experiment. This will be feasible for you only if you can create two environments (such as small greenhouses) that are identical except for temperature, and you can control the temperature. Equipment to control the temperature at 28 and 37 degrees in Bauchi for enough time to grow your plants may be difficult, especially since you will need to control the relative humidity to avoid a second independent variable. According to the internet, the temperature in Bauchi often rises above 27, so you will need a cooling system: this raises the expense of the experiment beyond the amateur level. (A heating system is a lot cheaper.) Alternatively, you might try to do a respiration experiment whereby you somehow grow the plants for an hour inside two small glass jars and measure the oxygen content inside the jar, at two different temperatures. Good luck! -Arch dude (talk) 14:07, 10 August 2008 (UTC)[reply]

obstertricians

what classes do i need to take in high school to give me a head start to become an obstertrician? —Preceding unsigned comment added by Goals101 (talkcontribs) 02:53, 9 August 2008 (UTC)[reply]

Obstetrics is a specialty within medicine. Medical students don't choose their specialties until they've already had a few years of med school, so if by "head start" you mean for getting into an obstetrics program, then you should focus on the same classes you would to get into a good college with a strong science program. First college, then med school, then obstetrics. If you are looking to actually start learning about obstetrics, then I would suggest taking sexual education classes and perhaps even getting EMT-B (or better yet EMT-P) certified. You may also be able to volunteer at an obstetrics ward at a local hospital if you're lucky. --Shaggorama (talk) 04:31, 9 August 2008 (UTC)[reply]
Different medical schools have different requirements, so be sure to check the website of the college you intend to enter. Generally, high-school courses in science (especially biology), math, and other analytical subjects are more useful for a medical career than art-related courses. --Bowlhover (talk) 07:00, 10 August 2008 (UTC)[reply]

Conservation of linear/angular momentum

How is it that someone is able to walk? I mean, when you push against the surface of the earth, you're really applying a torque, and thus giving the earth angular momentum rather than linear momentum. So where does a person's linear momentum come from? I've given this some thought, and the explanation I came up with is that we don't gain linear momentum, we gain angular momentum; basically, we push against the earth with one foot, and the rest of our body rotates around our ankle, which makes some sense. But this explanation doesn't seem to work in all cases. Consider a car, how is its momentum conserved? Well, I would say that when the wheels of a car push against the earth, the angular momentum lost by the earth is transfered to the car's wheels. Everything seems to make sense, until the body of the car is considered: where does it's linear momentum come from? Finally, let's say someone jumps against a wall, while the earth gains angular momentum, the person in question gains linear momentum. How is this possible? Thanks in advance. — Preceding unsigned comment added by 76.68.246.7 (talkcontribs) 04:28, 9 August 2008 UTC (UTC)

Good question.
What you have to remember is that a body has angular momentum when it is revolving around an external point, as well as when it is rotating around its own center. For example, the Moon has angular momentum due to its rotation around the Earth. Well, if you press horizontally against the Earth with your foot, or you press horizontally against a wall, or a car presses horizontally against the Earth with its tires, you or the car are putting yourself/itself into motion around the Earth. You are revolving around the Earth's center and that gives you the angular momentum to offset the angular momentum that your torque applied to the Earth.
If you jump vertically off the Earth, then you are in a non-rotary motion and you are acquiring linear momentum, and you are giving the Earth the same momentum in the direction that is downward for you. And real-life movements are a combination of the two cases, as a car rolls over hills and valleys and irregularities in the road, and your body moves up and down with each step.
--Anonymous, 07:00 UTC, August 9, 2008.
Angular momentum of an object can be seen as the sum of the linear momenta of all its particles. It's just a handy way of describing thing going around in circles, but angular motions of systems can always be seen as linear motions of particles. (Until one starts talking about quantum "spins" and such, and nice people don't do that.)
Hitting a wall is a force to the brick but a torque to the earth.
Torque (twisting force) is a linear force applied at a (linear) distance. Pistons push on the crankshaft (a "circular" lever) which twists more circular levers until the last ones, the wheels, push on the ground. It's all linear if you look at it closely.
How do you walk? First you have to start falling over, then move your feet under you to keep you from doing it. Saintrain (talk) 13:41, 9 August 2008 (UTC)[reply]

Triceratops vs Tyrannosaurus Rex

With their features and body parts, which do you think will win in a fight between a Triceratops and Tyrannosaurus Rex?MachaLeague (talk) 09:31, 9 August 2008 (UTC)[reply]

You say "will," which implies future, so I'll go out on a limb: a tie, since they're both extinct. For another opinion, see The Truth About Killer Dinosaurs. (They think it'd be a tie, too.) OtherDave (talk) 11:29, 9 August 2008 (UTC)[reply]
I'd probably say Tyrannosaurus as a 'trained' predator, whereas Triceratops were herbivores. That said, Triceratops may have had experience in defending itself when confronted by a predator. I don't really know. —CyclonenimT@lk? 12:16, 9 August 2008 (UTC)[reply]
God, you guys looovvvee being literal-minded, don't you? --98.217.8.46 (talk) 13:30, 9 August 2008 (UTC)[reply]
I can't speak for anyone else. I was having fun with the future tense, but I did post a link to help answer the actual question. And I signed it, too. OtherDave (talk) 17:07, 9 August 2008 (UTC)[reply]
I'd point out that this is not purely a theoretical discussion. Both the Triceratops and Tyrannosaurus lived in the Late Cretaceous (68 to 65 million years ago), with overlapping ranges (e.g. both were found in what is now Montana). If we operate under the assumption that the Tyrannosaurus was an active hunter (as opposed to being a scavenger - see Tyrannosaurus#Feeding strategies), one can conclude that there likely has been numerous clashes between the two. Indeed, the Tyrannosaurus article notes "A Triceratops was found in Mexico found with bite marks on its ilium. These were also inflicted by a tyrannosaur and they too appear healed, indicating active predation by the tyrannosaur." (And survival of the incident by the Triceratops, I might add.) This is also mentioned extensively in the Triceratops article. One might surmise that such "fights" generally went the way of most predator/prey encounters. A fraction of the time the predator will win the "fight", bringing down the prey. However, the majority of the time either the prey escapes, or the predator decides it isn't worth the hassle and gives up. Usually the predator will only go for the young, the elderly and the weak. Normally you wouldn't see a predator take on a healthy adult prey - it isn't worth it, as there will probably be opportunities for an easier kill later. I seriously doubt that most encounters would be the epic showdowns normally depicted in fiction - it's not worth the risk to the Tyrannosaurus. (Be honest, if your steak fought back you'd probably just order the fish, even if the resulting clash would have made for good cinema.) -- 128.104.112.147 (talk) 20:40, 9 August 2008 (UTC)[reply]

Pollution and cycling

Have there been studies of the impact on the respiratory system of riding a bike in the city? Thanks. 190.244.186.234 (talk) 14:23, 9 August 2008 (UTC)[reply]

Any particular city in mind? I don't think it matters what form of exercise you're doing, just how much of it. With the Beijing Olympics, the authorities are saying the pollution is only a risk for events lasting over an hour (the marathon, for example), so if you're in a city with similar pollution levels to Beijing, just ride your bike for less than an hour, and you should be fine. --Tango (talk) 15:21, 9 August 2008 (UTC)[reply]
Found this but not much more.
"even when account is taken of effort (a cyclist breathes on average two to three times as much as a motorist), the cyclist emerges as the victor of this comparison" (quoted in Cycling: the way ahead for cities and towns). (1996)
The thing I'm more worried about is all the fine particles. I sometimes ride behind buses in their thick black smoke. Fourty-five minutes twice a day. The article has got some real scare stories too and it talks about Australia and Europe that are not nearly as polluted as South American capitals. Any more links? 190.244.186.234 (talk) 19:38, 9 August 2008 (UTC)[reply]

Supertime

What would it mean to physicists if a second dimension of time were discovered ? [28] 69.157.227.243 (talk) —Preceding undated comment was added at 15:59, 9 August 2008 (UTC)[reply]

The first dimension of time as I understand it is the interval between one event and another or speed as we know it. As I understand it the change in this interval between repeated occurrences of events is the second dimension of time called acceleration. But then I've never slept in a Holiday Inn. —Preceding unsigned comment added by 71.100.162.249 (talk) 16:18, 9 August 2008 (UTC)[reply]
You do know that's garbled nonsense, right? 79.66.38.215 (talk) 17:12, 9 August 2008 (UTC)[reply]
It could have some interesting effects. I'm not sure exactly, but I think it would make time travel possible. The same as you can use a second space-like dimension to change your direction along a first space-like dimension (without having a slow down and stop momentarily), the same is probably true of time. I'd have to dig out my GR notes to work it out properly. --Tango (talk) 17:32, 9 August 2008 (UTC)[reply]
In one dimension there are two discrete directions which you can identify with past and future. In two or more dimensions there's a continuum of directions and you can't make that distinction any more. It's hard to see how such a universe could have a second law of thermodynamics or anything associated with the second law, like biological evolution. The one-dimensionality of time is almost its defining characteristic. -- BenRG (talk) 18:37, 9 August 2008 (UTC)[reply]
The other time dimension could be compact, like the extra dimensions in (some versions of) string theory, so it's hardly noticeable on an everyday scale. I'm really not sure of the details of how that would work. I'm not sure what the 2nd law of thermodynamics has to do with evolution, but that's not really important, an extra time dimension would mess with causality, and that's going to mess with everything. Time machines and temporal wormholes and things usually work in such a way that you at least have local causality, it's just globally that things get mess up. This would mess it up locally as well, which is asking for trouble! Could be interesting as a purely intellectual excercise, though. --Tango (talk) 19:11, 9 August 2008 (UTC)[reply]
It's not a new idea, there's been some real research into it. I haven't read much of it yet, but it's there. --Tango (talk) 19:14, 9 August 2008 (UTC)[reply]

Centrifugal Sorting of Saturn's Rings

The rings around Saturn are believed to consist mainly of pieces of water ice and a small amount of other substances. The material varies in size from dust-like particles to lumps the size of a small automobile. (See "Saturn" in Wikipedia.) There are probably individual atoms and molecules in the rings as well.

The rings vary in the brightness of the light they reflect from the sun. Each ring has the same brightness all the way around it and across its width. There are also gaps between some of the rings. Various theories have been advanced to account for the differences in brightness and the gaps.

The brightness variation suggests that the various rings are comprised of different material. My theory, which I have not seen elsewhere, is that the material in the rings has been sorted by centrifugal separation. (Think of a centrifuge.) Impurities in the water ice would give pieces of it a different density and color.

I conjecture that the densest material appears in the outermost rings, with decreasing density of the material in rings progressively closer to the planet.

Some of the gaps are caused by moons and moonlets clearing their orbit of small-size material by gravitational attraction. Other gaps, perhaps, are due to the fact that there is no material with a density that will place it in the gap.

The centrifugal separation would have taken place regardless of how the rings were formed.

The rings are comprised of an estimated 93 percent water ice. But the ice would not be pure. There would be many kinds and strengths of impurity. This would account for the large overall width of all the rings together

The differences in density of the impure ice particles would probably be small; but over millions of years, sorting would take place.

Material that contains no water would take its place in the rings. These rings would probably be at the outer and inner edges of the ring structure because the density of the material might be considerably different from that of impure water. —Preceding unsigned comment added by 67.150.190.122 (talk) 17:11, 9 August 2008 (UTC)[reply]

The centrifugal sorting may have worked together with other causes that have been suggested for the formation of the rings.

AndMe2

67.150.190.122 (talk) 16:30, 9 August 2008 (UTC)[reply]

It's an interesting hypothesis! Are the planets in our solar system in order of mass or density? DMacks (talk) 16:51, 9 August 2008 (UTC)[reply]
No. See Table of the largest objects in the Solar System. --Tango (talk) 17:38, 9 August 2008 (UTC)[reply]
Er yeah, I'm aware of that. It was a continuation of poster's thought experiment, giving him an opportunity to do some actual science and test his idea (make hypothesis, use it to make predictions, check if those predicions agree with reality). If centrifugal sorting worked on "dust particles orbiting a planet", it would likely also be evident in "planets (really big particles) orbiting the sun" (that latter having been taking place for eons longer that Saturn's rings). So if not, it means hypothesis isn't true (or needs to be adjusted to account for this new data). DMacks (talk) 05:33, 10 August 2008 (UTC)[reply]
I don't think that's likely. An object's orbit depends only on it's velocity, position and the mass of the central object (assuming there is a big difference in the masses, as there is in the case of dust orbiting Saturn), density doesn't come into it. A centrifuge is very different from orbiting dust - particles of dust don't interact with each other much, the various things in a centrifuge do, and that's why density becomes a factor. The gaps are caused by resonance with the moons, I believe. --Tango (talk) 17:38, 9 August 2008 (UTC)[reply]
What if the rings formed in a gas-rich environment? Then they would be particles in a fluid. Would there be a centrifugal effect in that case? Just asking as I have no idea. Franamax (talk) 19:58, 9 August 2008 (UTC)[reply]
I'm just guessing now, but I expect if there was enough gas to make a difference, the orbits would just decay and the rings would crash into the planet. Even if they didn't, I think it would only the difference in density between the gas and the ring matter would be significant, so you would get one ring at the bottom of the gas made up of heavier stuff and one at the top made up of lighter stuff. It's an extremely hypothetical situation, though, I don't think you would ever have things orbiting in dense enough gas for it to be significant. --Tango (talk) 21:28, 9 August 2008 (UTC)[reply]
Well presumably the gas would be orbiting with similar velocity, so I'm not sure I see the mechanism to crash the rings, since there would be no significant drag in that case. Given a sufficient gas component though, there would be a mechanism to transfer momentum between the solid particles similar in a way to actual collisions between the particles themselves? If they're building a new planet anytime soon, I guess we should try that out. :) Franamax (talk) 01:30, 10 August 2008 (UTC)[reply]
Hmmm... I hadn't thought of the possibility of the gas being in orbit, I was thinking of it being like an atmosphere. I honestly have no idea what would happen in that case... --Tango (talk) 02:55, 10 August 2008 (UTC)[reply]
This is very interesting and suggests to me that the rings around Uranus may be similarly governed. —Preceding unsigned comment added by 79.76.242.64 (talk) 23:03, 9 August 2008 (UTC)[reply]

What STD can be transmitted through oral sex?

Which is the most common? Mr.K. (talk) 18:39, 9 August 2008 (UTC)[reply]

Well firstly particularly for the 'most common' bit, it'll be helpful if you specify what & who you're referring. The partner performing oral sex on the genitals of a female partner? The partner performing oral sex on the genitals of a male partner? A male receiving genital-oral sex from a partner? A female receiving genital-oral sex from a partner? (The sex of the performing partner doesn't tend to matter for obvious reasons.) Something else? The rates tend to vary a bit depending on what and who. But to answer the first part of your question simplisticly, I don't know of any STD that can't be transmitted via oral sex. Logically there's no reason an STD won't be transmitted via oral sex unless the STD doesn't infect and/or can't be passed via the mouth/oral region. Of course, STDs which only infect a local area will only generally infect the area they are passed to be it the oral region or the genitalia region. According to this [29] HPV is perhaps the only one which isn't that much of a concern when it comes to oral sex (but it can be transmitted in rare instances) and herpes is the most common (but that probably has a lot to do with do with how common it is in the general population and the fact it tends to be lifelong with occasional flareups). Nil Einne (talk) 19:27, 9 August 2008 (UTC)[reply]
I'm not so sure HPV isn't much of a concern. According to this, "A recent study conducted by Dr. Maura Gillison at the Johns Hopkins Oncology Center furthered the premise that HPV is linked with certain types of oral cancer." --71.185.73.240 (talk) 04:18, 10 August 2008 (UTC)[reply]
Chlamydia [30]. Franamax (talk) 00:46, 10 August 2008 (UTC)[reply]
I remember reading a while back that oral gonorrhea was on the rise amongst U.S. and U.K. teenagers. I can't seem to find any statistics that list STDs by type of infection, however. Horselover Frost (talk) 04:05, 10 August 2008 (UTC)[reply]

Educational videos in psychology

Where can I found a source of real case studies in psychology Q! Ĩ am trying to find videos of patients with some sort of mental illness or descriptions of a disorder with plenty of examples. Mr.K. (talk) 18:40, 9 August 2008 (UTC)[reply]

That would be cool. *adds name to MOAR list* --mboverload@ 19:09, 9 August 2008 (UTC)[reply]
Check out www.learner.org. They have many videos on demand that you can watch for free. The videos they make are often shown on PBS, because I specifically remember watching one of their videos on TV about schizophrenia (sp?). You might have to navigate that site for a while, but I'm sure you'll eventually find their archieved psychology videos.--El aprendelenguas (talk) 22:17, 9 August 2008 (UTC)[reply]
You'd be surprised what you can find on youtube. My interests lie more in [neurology], but still I'd suggest you run searches on youtube with the titles of conditions in which you are interested in or experiments you would like to find documentation of. --Shaggorama (talk) 13:59, 10 August 2008 (UTC)[reply]

Academic, corporate and free researchers

These three are probably closely entagled with each other. However, I would like to know how much researchers work in an academic, corporate or free environment, where is the money, and, in terms of patents and publications, how productive they are.Mr.K. (talk) 18:42, 9 August 2008 (UTC)[reply]

Generally speaking, money and often patents are in corporate, publications are in academia. But that's very generally speaking...it varies among the different areas of science, and a famous academic who becomes head of some huge non-profit research center would make more (esp. in honoraria) than an entry-level BS in some company. DMacks (talk) 20:38, 9 August 2008 (UTC)[reply]
This varies quite a lot country-by-country. For example, in 1995, in the USA around 17.5% of all researchers were in higher education (as opposed to government or corporate work), whereas in Japan it was more like 36.1%. However in the same period Japan spent only 75% of what the US spent on higher education R&D. Productivity varies a lot per country as well. Some countries, practically all of their research goes into industry or government sources (like China), whereas some are almost all government (like Russia). The OECD has tons of statistics about this sort of thing. If you only care about the US case, the NSF keeps very detailed statistics about how money is being spent and what comes from it. --98.217.8.46 (talk) 15:31, 10 August 2008 (UTC)[reply]

Automobile bumpers

Why do cars don't have a bumper car-type rubber bumpers? Mr.K. (talk) 18:49, 9 August 2008 (UTC)[reply]

Because cars need to absorb the energy of a collision, and thus need to bend and break to absorb it. We could make cars that would not be damaged in a collision, but at the expense of damaging the human inside instead. Crumple zone, bumper, Car crash. Any car with a bumper-car style bumper would not even be legal to drive in any first world countries. Any car produced today with no crumplezones and a completely rigid frame and bumper would probably not pass the safety requirements in many countries. --mboverload@ 19:04, 9 August 2008 (UTC)[reply]
In simplistic terms, this is achieved by controlled weakening of outer parts of the car while strengthening the inner (passenger cabin) part of the body --mboverload@ 19:15, 9 August 2008 (UTC)[reply]


Because SPEED does the matter. With the highest speed of rubber bumper car can't even kill a man, but the speed of car on the road makes much higher impact. Even we can found material in the future to adsorb collision energy, the static velocity of the people in the car may kill them anyway....NINJAW —Preceding unsigned comment added by 124.120.200.29 (talk) 07:05, 10 August 2008 (UTC)[reply]

As above, Speed matters: the energy to be absorbed goes up withthe square of the collision speed. Another important point: you can in fact build a bumper that can absorb and release all that energy, but it will make the vehicle unacceptably large and unacceptably heavy, thus ruining the gas milage. -Arch dude (talk) 13:34, 10 August 2008 (UTC)[reply]

Control group in statistics

In statistics, it is considered necessary to have a control group (placebo group). However, in some situations (heart surgery, smoking and cancer, etc) a control group is not possible. How can I deal statiscally with these cases? Mr.K. (talk) 18:50, 9 August 2008 (UTC)[reply]

You might try the Mathematics desk for this question. Also, maybe a bit more detail. What sort of data do you have in mind, and what sort of questions do you want to ask? --Allen (talk) 19:01, 9 August 2008 (UTC)[reply]
In medical research it is unusual for the control group to be untreated, since it would be unethical to not treat a group of people. Trials are usually done comparing two different treatments (one established and the other new). The other option is to test the new treatment on people that have exhausted all other treatment options, so in the absence of the trial, they wouldn't be treated anyway, so there is no ethical dilemma in not treating half the patients (at least until the treatment is shown to work). --Tango (talk) 19:24, 9 August 2008 (UTC)[reply]

Caffeine timing and sleep

If I want to fall asleep by 12:30 am, what time of day do I need to stop having caffeine? NeonMerlin 19:49, 9 August 2008 (UTC)[reply]

My experience says that caffeine sensitivity varies wildly between people both in terms of length and powerfulness of effect. I personally need about 6-8 hours for a large amount of caffeine, but I've known other people with other timing. Magog the Ogre (talk) 19:58, 9 August 2008 (UTC)[reply]
Concurred...it's very variable (see Caffeine and maybe Caffeine and healthCoffee and health for general info). I can down 4-6 cups and go to sleep within an hour, but I'm a heavy coffee drinker and am usually sleep-deprived. DMacks (talk) 20:35, 9 August 2008 (UTC)[reply]
(fixed link) DMacks (talk) 21:54, 9 August 2008 (UTC)[reply]
Well the half-life of caffeine is 9-11 hours, according to our article. It depends on how quickly your body absorbs the caffeine and how good your liver is at breaking down the stuff. —CyclonenimT@lk? 00:21, 10 August 2008 (UTC)[reply]

Space-time equation

Since space and time are relative, could scientists ever come up with an equation to turn time into space and vice-versa ? 69.157.227.243 (talk) 20:08, 9 August 2008 (UTC)[reply]

The long answer: almost anything is theoretically possible, given how little we know about the nature of the universe, and the possibilities that may exist.
The short answer: no: the two are completely different measures. Magog the Ogre (talk) 20:24, 9 August 2008 (UTC)[reply]
I don't know about "completely different measures." See Spacetime. --Allen (talk) 20:41, 9 August 2008 (UTC)[reply]
I'm not sure what you mean... an equation describes something, it doesn't do anything. Space and time are distinct (it comes down to a minus sign in the appropriate places), but are closely interlinked. You can come up with a new co-ordinate system that combines the usual space and time co-ordinates, but you'll always have one time-like and three space-like co-ordinates. Interestingly, within the event horizon of a black hole, time becomes space-like and the radial space direction becomes time-like, but that's not really time and space swapping round, it's just a technical thing. --Tango (talk) 21:33, 9 August 2008 (UTC)[reply]
Yes, I suppose I didn't quite read the question carefully. The answer is actually yes, they can be interchanged, but not in a practical way. By traveling at an extremely high rate of velocity (i.e., very close to the speed of light), one can essentially skip the time in between. This is time dilation. One could theoretically travel to the other side of the galaxy and back in a few minutes, and millions of years would have passed back at Earth. This would of course require astronomical amounts of energy.
This might not be the pretty solution you're looking for, but I don't know what else you might want: it's not like scientists could just remove a triangular piece of space. Magog the Ogre (talk) 02:50, 10 August 2008 (UTC)[reply]
That's not really interchanging space and time, it's just interchanging the rate at which you travel through each. --Tango (talk) 02:57, 10 August 2008 (UTC)[reply]
I thought the whole point of talking about "space-time" is that they are just properties of the same stuff, dimensions within the structure of the universe, and that our manner of perception creates the duality. I feel like your question is similar to asking if you could interchange between surface area and depth. Is my understanding of spacetime that far off? --Shaggorama (talk) 14:46, 10 August 2008 (UTC)[reply]
The whole point of Einstein about spacetime is that when you try to measure space, you are also making measurements of time, and when you try to measure time, you are also making measurements of space. This itself is not very surprising, except when you throw into this bit, relative mix the fact that there are a number of key invariants, things that aren't relative, like the speed of light. Then there are important consequences. It is a geometrical argument about the structure of measurement and its consequences. It's quite clever. But it's not "space and time are exactly the same thing and you can change one into another" or anything like that. It's that space and time are, quite logically and not mysteriously, inextricably linked to one another and there are practical consequences of that. --98.217.8.46 (talk) 15:21, 10 August 2008 (UTC)[reply]

Recent Horrible News!

Recently I've gone through an article of NASA where it was stated that the Earth will be annihilated in 2012. Is it so truely? Is there no way to escape it? Anyone to answer is thankfully welcome.117.201.96.57 (talk) 20:04, 9 August 2008 (UTC)[reply]

No, it's completely false, despite claims by some (e.g., An Inconvenient Truth) - where did you read this, and did you truly find it plausible? Magog the Ogre (talk) 20:18, 9 August 2008 (UTC)[reply]
An Inconvenient Truth does not claim the Earth will be annihilated in 2012. I'm sure you were joking, but someone not familiar with the movie might not realize that. --Allen (talk) 20:22, 9 August 2008 (UTC)[reply]
Yeah, I was kidding (sorry, too much Glenn Beck on CNN anymore). Magog the Ogre (talk) 20:25, 9 August 2008 (UTC)[reply]
Actually, I've read these news on the Magazine 'Science Reporter'. Thank you all, for answering. I am relieved from a tension now.117.201.96.57 (talk) 20:27, 9 August 2008 (UTC)[reply]
The Reference Desks have already had two discussions on this matter; see [31] and [32]. --Bowlhover (talk) 20:32, 9 August 2008 (UTC)[reply]
An article by NASA? That sounds unlikely... was it actually from NASA or was it from some random person claiming that it was NASA that had originally said it? If the latter, they are simply lying. It's just a load of crackpots talking nonsense. --Tango (talk) 21:35, 9 August 2008 (UTC)[reply]
I'm with tango. I'm challenging you to produce this article, as I'm sure we'd all love to read it. --Shaggorama (talk) 14:40, 10 August 2008 (UTC)[reply]

Comet of Sodom

Look at this : they found the bits and pieces of an asteroid that destroyed the Sodomites in 3123 BC. [33] Explain this one ! San Francisco watch out! 69.157.227.243 (talk) 20:36, 9 August 2008 (UTC)[reply]

What do you want explained? An asteroid hit an ancient city, people at the time didn't understand what had happened and assumed it was the wrath of god and, after a few rounds of Chinese whispers, it got written down in a book that eventually became part of the Old Testament. Doesn't seem at all odd to me. There is no reason to believe it actually had anything to do with the residents of the city being particularly sinful. --Tango (talk) 21:39, 9 August 2008 (UTC)[reply]
I wonder if "March 31, 2008" explains this one? I think we have more to fear from people who don't use all the facts at hand and look for excuses to justify their biases than from comets:) But even today, otherwise-intelligent people forget (or willfully ignore if it suits their agenda) that correlation does not imply causation. Stock market went up the other day just after I drank coffee, but I'm not gonna sink my life savings on Wall Street and head to Starbucks to make myself rich. DMacks (talk) 22:38, 9 August 2008 (UTC)[reply]
From the article: "Dr Hempsall said that at least 20 ancient myths record devastation of the type and on the scale of the asteroid’s impact, including the Old Testament tale...." Emphasis mine and mine alone. -- MacAddct  1984 (talk &#149; contribs) 22:40, 9 August 2008 (UTC)[reply]
Ummmm, there is no mention of any "bits and pieces" being found. The article is about working from half of a 4-inch clay copy of a 2.4 thousand year old clay tablet to pinpoint the hour, 5000 years ago, of an observation that lasted no more than 10 seconds.
Meteorites have impacted Earth many times. How about this and this and this and this or any of these? It happens because of gravity and chance. Saintrain (talk) 23:17, 9 August 2008 (UTC)[reply]
Asteroids of the size mentioned in the article, however, only strike Earth once every 80 000 years. If an asteroid "more than half a mile across" really did wipe out Sodom, the incident would be the only major impact event in human history that we know about.
An impactor usually creates a crater twenty times its own diameter, so a half-mile-wide asteorid would create a 10-mile-wide crater, one that should have been discovered long ago. It's also interesting that the article claims the asteroid's impact released energy "equivalent to 1000 tons of TNT exploding", which would imply it was much less energetic than the Tunguska event. A realistic figure would be 50 000 million tons. --Bowlhover (talk) 06:25, 10 August 2008 (UTC)[reply]
OK, yeah, glad to see I'm not the only one casting a skeptical eye on this (I've seen other junk articles from timesonline, btw). A piece of pottery talking about a meteorite from 2300 years beforehand, before there was writing, while there were no accurate measurements around? I suppose it is possible (the Semites had a sick amazing oral tradition), but it seems highly unlikely - along the lines of Atlantis. Magog the Ogre (talk) 02:57, 10 August 2008 (UTC)[reply]
As Macaddict and Magog tO, there may be the impeccably si-ick oral traditions that leave our media in the shade with feats of memory making silicon valley look primitively mechanistic but when it comes to the causal connections, there's magical thinking to consider. Julia Rossi (talk) 10:01, 10 August 2008 (UTC)[reply]

I can't find my mistake

I want to find an equation for the final velocity of an object that is dropped from a height h (not assuming g remains constant), but the answer I keep getting is wrong.

G is less than 0.
where G=gravitational constant, re=radius of the earth, Me=mass of the earth, and h=height the object was dropped at.

Everything seems right, but the final answer's imaginary: somewhere I've made a mistake with a negative, but I don't know where. Please help, and thanks.

Edit: I'm considering down (i.e. towards the surface of the earth) as positive. —Preceding unsigned comment added by 76.68.246.7 (talk) 23:25, 9 August 2008 (UTC)[reply]

Why have you said that G is less than zero? It isn't. G is positive, and you need a minus sign in the second equation. Those cancel out, though, so won't affect the final answer. Next, why are the limits on your integral -re and -(re+h)? Where did those minus signs come from? The radius of the Earth is a positive number. I think making those limits positive will fix your problem. --Tango (talk) 00:09, 10 August 2008 (UTC)[reply]
Sometimes G is considered negative and the minus sign in the second equation is ignored, it doesn't make a difference, people have different preferences. As for the limits in the intergral, they're negative because I'm considering the downwards direction as positive. In other words, the initial position of the object is -(re+h) because the object would need to travel a positive distance (re+h) to reach the earth's centre. Notice that change in displacement is a positive number (h), but if I treated the limits as positive, the change in displacement would have been negative.
To better illustrate this, consider the object as undergoing a uniform acceleration (down is positive). In this case, v=2aΔs, v=2a(rf-r0), and because rf is 0, this equation wouldn't make sense if r0 was positive.
You can't both take G as negative and down as positive, that's self-contradictory. Gravity is directed towards the center of the Earth and hence the force of gravity must have the same sign as "down". Dragons flight (talk) 01:47, 10 August 2008 (UTC)[reply]
Exactly. Gravity acts to reduce distance, which is why there's a minus sign in Newton's universal law of gravity (or, equivalently, G is taken as negative, although I've never seen that). If you measure distance backwards, you'll have to get rid of that minus sign. --Tango (talk) 02:13, 10 August 2008 (UTC)[reply]
Well, if down is taken as positive, then r would be negative, and so G (or Newton's universal law of gravity) would still have to be negative, right?
Edit: Of course, r^2 is still positive, my bad. Thanks. —Preceding unsigned comment added by 76.68.246.7 (talk) 03:10, 10 August 2008 (UTC)[reply]
I think it's best just to keep G as positive always and add in minus signs where you need them. Changing the sign of a physical constant to suit your co-ordinate system just seems confusing to me... --Tango (talk) 03:30, 10 August 2008 (UTC)[reply]


August 10

Herbivores and omnivores in same species

Are humans the only species in which some members are exclusively herbivorous and others aren't? NeonMerlin 00:20, 10 August 2008 (UTC)[reply]

Some cats are omnivorous. There might be other species whose diet is modified based on the availability of food. − Twas Now ( talkcontribse-mail ) 01:27, 10 August 2008 (UTC)[reply]
all humans are omnivorous, very few people have never eating plants or meats. even if many people choose to permanently eat both or either. i.e. most muslims know what pork tastes like, they've tried it, and many are made to try it so they can identify it.MYINchile 03:07, 10 August 2008 (UTC)[reply]
Careful, you don't want a fatwa. I strongly suspect that as most muslims live in countries or communities where Islam is endemic, pork simply isn't available, and the social penalty for eating it is severe. —Preceding unsigned comment added by 86.128.192.207 (talk) 17:40, 10 August 2008 (UTC)[reply]
Well, I know dogs are omnivores and can live on vegetarian diets or on mixed diets, as humans can. I imagine there are other species for which this is true too. I have a suspicion the same is true for pigs, but I can't find a reliable looking source either way right now. 79.66.38.215 (talk) 04:57, 10 August 2008 (UTC)[reply]

Clearly example is pyrhana, there are 2 types seperately (omnivore and herbivore)....Ninjaw —Preceding unsigned comment added by 124.120.200.29 (talk) 06:59, 10 August 2008 (UTC)[reply]

Note that being an omnivore isn't really a question of choice, it's a question of biology. Limiting yourself to a certain diet doesn't make you any less of an omnivore, even if it makes you a vegetarian. -- Captain Disdain (talk) 09:23, 10 August 2008 (UTC)[reply]

i need help trakcing down sources for wet wipe markers, also known as chalk in or fluorescent markers, you know they ones they use on the outdoor menu displays at restaurants, that look like chalk but wont get all messy, yeah those. theres a bunch of alleged sources here, but i can't find them on their own, would anyone care to help?MYINchile 00:38, 10 August 2008 (UTC)[reply]

I'd try staples. --Shaggorama (talk) 14:37, 10 August 2008 (UTC)[reply]

Density of human blood

Water is 1 gm/cc. what is blood density? please? —Preceding unsigned comment added by 79.76.242.64 (talk) 01:29, 10 August 2008 (UTC)[reply]

According to our Blood article its 1060 kg/m³. -hydnjo talk 01:49, 10 August 2008 (UTC)[reply]
Thats not much denser than water then!, So 8 pints of blood (the content on the human body) only weighs about 10 pounds? —Preceding unsigned comment added by 79.76.242.64 (talk) 02:00, 10 August 2008 (UTC)[reply]
If I typed the right numbers into Google calculator, it comes out to just under 9 pounds, but yes, it's barely more than water. --Tango (talk) 02:16, 10 August 2008 (UTC)[reply]
It would depend on whether you mean US or Imperial pints. Anyway, the number is about right. The Blood article says the typical volume in a human is about 5 liters, which would therefore weigh 5 × 1.06 = 5.3 kg or almost 12 pounds. But there'd be enough variation that you couldn't count on the second digit there. --Anonymous, 04:57 UTC, August 10, 2008.
True, I assumed US pints since usually online, if people don't specify, they're more likely to mean US. (I'm British, so it's not that I'm bias in favour of US measurements!) --Tango (talk) 05:13, 10 August 2008 (UTC)[reply]

If everything in the universe suddenly doubled in size, would it matter?

Would it be noticeable on some level? Like physical laws changing? I thought like Einstein and realized space is relative so at first I thought no. Then I remembered something- singularities. How could you double the "size" of that without seriously disrupting something? It would probably spawn some new emergent property with disastrous consequences! 0_0 -----Sam Science (talk) 01:38, 10 August 2008 (UTC)[reply]

Some formulas for natural phenomena include a parameter that is "squared". If everything doubled, these phenomena would be 4 times as great. That might produce unexpected results. Incidentally, I wonder why "squared" crops up so often in formulas for natural phenomena. An example is e=mc2. There are many, many other examples. Why should something be multiplied by itself in natural phenomena? AndMe2 66.52.8.89 (talk) 02:19, 10 August 2008 (UTC)[reply]
A special case of this is that if you go from the first to the second dimension (length to area) you multiply lengths so you get squares. --Ayacop (talk) 06:45, 10 August 2008 (UTC)[reply]
The simplist way a squared can come up is because a factor appears in two different things that end up being multiplied. First thing that springs to mind is air resistance which is proportional to the square of velocity because velocity affects how many molecules of air you hit and how much energy you hit each one with. --Tango (talk) 02:29, 10 August 2008 (UTC)[reply]
With any "What would happen if this changed?" type question, it's important to state precisely what's changing and what's staying the same. How are you doubling the size of everything? You would need to change some physical constants appropriately (reduce the electric constant to increase the size of atoms, amongst other things). You have to make sure the physical laws are still obeyed, for example, , (those are the speed of light, electric constant, vacuum permeability, in that order), so if you change the electric constant, you have to change either the speed of light, or the permeability to compensate. If you change everything just right, you wouldn't notice any difference (for example, if you doubled the speed of light, the speed of light measured in terms of the radius of one of the objects you've doubled the size of would be the same). --Tango (talk) 02:29, 10 August 2008 (UTC)[reply]
Let's consider what would happen if all objects doubled in linear dimensions without changing any physical constants or other material properties. The Earth is 8 times as massive, but its surface is twice as far from its centre, so the acceleration due to gravity at the surface is 2g. Your body is 8 times as massive as it was, so you weigh 16 times your normal weight. However, your muscles are only 4 times as powerful (muscle strength varies with cross-sectional area). You need 8 times as much oxygen as you do now, but your lungs' surface area has only increased by a factor of 4. Fortunately, the air is twice as dense (due to increased surface gravity) so you only need to breathe at the same rate as normal. But you are moving a chest that weighs 16 times its normal weight with muscles that are only 4 times as powerful. Doesn't sound like a pleasant experience ! Gandalf61 (talk) 10:05, 10 August 2008 (UTC)[reply]

Thremocouple power output

What is it? —Preceding unsigned comment added by 79.76.242.64 (talk) 02:34, 10 August 2008 (UTC)[reply]

Take a look at thermocouple and see if that answers your question. Otherwise, you'll need to be more precise about what you want to know. --Tango (talk) 02:58, 10 August 2008 (UTC)[reply]

Cooking Pasta

Standard instructions for cooking spagetti (and other dry pasta) are:

bring water to a boil, then put the pasta in the boiling water.

Is there any good reason to do it this way, or will it work just as well to put the pasta into cold or warm water, then bring the water to a boil?

Based on limited experiment, I suspect the latter is the case.

Thanks, Wanderer57 (talk) 02:54, 10 August 2008 (UTC)[reply]

It makes estimating the cooking time a little difficult, since it depends on how quickly the water heats up. Other than that, I can't see any problem. It's more energy efficient, however, to heat the water in a kettle and then add it to the pan already boiling - kettles heat the water quicker, so there's less time to lose heat. --Tango (talk) 03:01, 10 August 2008 (UTC)[reply]
Also, the pasta will tend to clump and stick together during the heat-up. -hydnjo talk 03:18, 10 August 2008 (UTC)[reply]

If you add a little olive oil, the pasta won't stick together. AndMe2 66.52.8.89 (talk) 03:25, 10 August 2008 (UTC)[reply]

I am not a science type but I am a chef. It makes little difference to the resulting pasta if we're talking about a single serving; a small portion in an amount of water that comes to a boil quickly. However, it makes a big difference if we're talking about a large amount of pasta in a corresponding large amount of water that takes a significantly longer time to come to a boil. What you want is al dente. I donlt know the exact scientific reason why but pasta which soaks for any length of time before coming to a boil doesn't yield al dente pasta. Instead, what you get is mushy pasta (mushy pasta=bad). There is another reason for putting the pasta in boiling water, and this applied to both large and small servings: you need to stir pasta to avoid clumping. putting the pasta in immediately means you're going to be standing over the pot stirring for a longer period of time. Efficiency in cooking matters a lot when you're doing it over and over. By the way, putting oil in pasta water as an inoculation against sticking isn't effective, is a waste of oil and shouldn't be done for a very good reason: the sauce wonlt stick well to the pasta, it slides off. Don;lt use oil in the water, just stir. Some other things you might want to know: always wash dry pasta in water just before you put it in the pot. This takes off the starch (if you put it in a bath you'll see how cloudy the water gets); you can't make good pasta without salting the water (use kosher salt, never iodized); Once your pasta is at the perfect state, and you've drained, add your sauce immediately which cools it down. If you take a bit mass of hot, al dente pasta and let it sit, it will continue to cook and voila, mushy pasta.--70.107.9.159 (talk) 03:47, 10 August 2008 (UTC)[reply]
Two thoughts about your practical observations:
1. Do you really come to a rolling boil first, or just gentle boil? A rolling boil could itself help avoid clumping by keeping the pasta moving around. That is, it's more than just reducing the need for "standing over the pot stirring for a longer period of time".
2. If soaking pasta leads to mush but plunging into boiling water gives good cooking, I wonder if the boiling water (nearly) instantly cooks the outside a bit (gelling the starch, or whatever happens when carbs "cook"), and this coating prevents the mushing. Either sealing out too much water or preventing the whole structure from dissolving (it's just a chunk of flour:). Something like searing a steak or blanching french fries?
Dang, now I'm hungry. DMacks (talk) 05:23, 10 August 2008 (UTC)[reply]
I second the "don't waist oil in the water" and add don't time it, throw your pasta at the window, wall, fridge or whatever "technique" used to time the cooking. Just TASTE it and soon you'll know exactly when they need, 30 seconds or a minute more to be exactly the way you like. 190.244.186.234 (talk) 05:54, 10 August 2008 (UTC)[reply]

Ants climbing trees

Recently while up a ladder to cut a branch off a tree, I noticed ants on the bark of the tree.

Why might ants go to all the work of climbing trees?

The tree was not doing something that might have provided food to the ants, like flowering or dripping sap.

Thanks, Wanderer57 (talk) 03:05, 10 August 2008 (UTC)[reply]

Cutting the leaves and collecting them for the nest is what I remember from a documentary. Don't they let themselves fall down with it, too? --Ayacop (talk) 06:36, 10 August 2008 (UTC)[reply]
Right at the tips of the branches there may have been herds of aphids being farmed by ants collecting the honeydew. Ants don't seem to know the meaning of distance and will walk a long way for some honeydew. Richard Avery (talk) 06:47, 10 August 2008 (UTC)[reply]
There appears to be only one kind of ant that does the cutting: leafcutter ant. --Ayacop (talk) 06:50, 10 August 2008 (UTC)[reply]
There are also ants that live in trees. See Acacia cornigera and Acacia drepanolobium--Lenticel (talk) 10:33, 10 August 2008 (UTC)[reply]
Thanks. I'm in southern Ontario, Canada, a long way from where leafcutter ants or those acacia species are found. I will take a closer look at the tree.
Maybe the ants just went up for the view. Wanderer57 (talk) 16:38, 10 August 2008 (UTC)[reply]

How do we know Planck measurements are the ultimate?

For example, how do they know the "smallest" planck length is 10 to the negative 20th power? Couldn't you just divide again? The problem could be scale. If something the size of the universe was trying to measure something on our scale it would be meaningless to observer.- And observed.--Sam Science (talk) 03:58, 10 August 2008 (UTC)[reply]

My not very well educated answer is that all quantum mechanics appears to work in multiples of the constant or the constant divided by 2(pi). There is not a single part of quantum mechanics that does not, so far as I can tell. Additionally, it has something to do with the c (the speed of light in a vacuum), and is thus clearly somehow integral to the very nature of the universe. In fact, with e=mc2, were Planck's constant infinitely small, c would be infinitely large, and there would be infinite energy in the universe. Unfortunately, Wikipedia's article on the constant is extremely lacking (Brittanica's article is much better).
Sorry for being so nonspecific. Magog the Ogre (talk) 04:24, 10 August 2008 (UTC)[reply]
You're getting confused between the Planck length and Planck constant, I think. --Tango (talk) 04:26, 10 August 2008 (UTC)[reply]
Yes but aren't they ultimately related? Magog the Ogre (talk) 04:29, 10 August 2008 (UTC)[reply]
In that the Planck constant is one of the constants used in the definition of the Planck length (together with the gravitational constant and the speed of light), yes. --Tango (talk) 04:31, 10 August 2008 (UTC)[reply]
Planck length#Physical significance has a thought experiment that tries to explain it. --Tango (talk) 04:31, 10 August 2008 (UTC)[reply]

Centrifugal Separation of Materials

OK. The rings of Saturn were not separated by centrifugal force. I never quite believed it anyway.

But the discussion has led me to some further thoughts about centrifugal separation.

Assume a mixture of material is being separated in a centrifuge. Is the size of the particles significant in centrifugal separation? Conventional (high school) wisdom says "No". But I am wondering.

To illustrate, let us say a quantity of glass marbles is mixed with a quantity of steel balls. All balls have the same diameter. Each steel ball therefore has a greater mass than each glass marble. The mixture is put in a large centrifuge and spun. The steel balls will more strongly resist deviation from motion in a straight line and will push their way between the marbles. The steel balls will thus work their way to the outer edge of the spin path. The inevitable vibration (from imperfect bearings, etc.) would hasten the process. A thin coat of non-sticky lubricant on each ball would also assist by reducing friction as the balls slide past each other. Or for simplicity to establish the basic principle, we could assume there is no friction between any of the balls. (Centrifugal separation of gases avoids most of the friction, I think.)

Now imagine a mixture of glass marbles and lead shot. The marbles are all the same diameter. Each lead shot is the same diameter, but it is small enough that it has less mass than each marble. Spin the mixture in a centrifuge. The marbles (having greater mass) will more strongly resist deviation from motion in a straight line Will the marbles work their way past the lead shot to the outer edge of the spin path? If so, the materials will become separated by their density, rather than their mass. Let us assume there is no friction.

I don't have an answer to this. Maybe someone can come up with one.

Andme2 (talk) 07:44, 10 August 2008 (UTC)[reply]

see Brazil nut effect. Saintrain (talk) 13:50, 10 August 2008 (UTC)[reply]
To clarify, if in the thought experiment you provide you are saying that the marbles and lead shot are the same diameter but different masses, then the brazil nut effect is inapplicable because it is a funciton of volume and not mass. --Shaggorama (talk) 14:34, 10 August 2008 (UTC)[reply]

Eucalyptas Tree,nor Cal

Greetings.

We have a stand of rather tall Eucalyptas tree in our yard.

We live in Carmel Valley,Ca

Just recently r deck in the back by the stand is getting bombed with PURPLE droppings,,looks like bird poooooooop, but it is a berry color..!!!

Not trying to stop it, just would like to know what the heck is really going on,,?

Regards.............and................Thanks --Ddemos (talk) 09:04, 10 August 2008 (UTC)[reply]

Is it mulberry season where you are? Julia Rossi (talk) 09:51, 10 August 2008 (UTC)[reply]
Bat shit is supposedly purple..87.102.45.156 (talk) 13:24, 10 August 2008 (UTC)[reply]
or the blackberry season? Richard Avery (talk) 14:51, 10 August 2008 (UTC)[reply]

Flu like symptoms

As I understand, flu like symptoms are a reaction of the body to a pathogen which arise in the process of immune defense. But how exactly are these symptoms (or what immediately causes them) helping to defend the pathogen? 93.132.159.115 (talk) 10:56, 10 August 2008 (UTC)[reply]

There are several symptoms which you are referring to, I presume. Coughing is the mechanism the body uses to clear the trachea of foreign matter, be it fluid or a solid. This is particularly relevent in Influenza and other upper respiratory tract infections like the common cold since the viruses bind on epithelial cells. The remaining symptoms, bar one or two, are more side effects of the infection rather than a method for helping to remove the virus. Fever, for example, is usually the result of pyrogens released by bacteria or viruses. However, such substances are also released by the cells of your immune system. This particular topic is covered in more detail here. Individual symptom pages usually have a mechanism section, so you may wish to try: Fatigue (medical), Sneeze, Fever and Headache. —CyclonenimT@lk? 12:16, 10 August 2008 (UTC)[reply]

The Matrix

What are the references to mdern physics in the movie The Matrix ? 69.157.227.243 (talk) 13:40, 10 August 2008 (UTC)[reply]

One of the points of the movie was that the main characters "hacked" the matrix and so could violate it's rules. Among those rules were the rules of physics. I don't think there was any reference to physics in general in the movie beyond the characters capacity to violate fundamental laws such as gravity. If you want to know what physical laws were violated, I'm sure other members of the RD will be happy to list a few for you. As far as "modern physics" goes, I typically take this phrase to mean materials studies, relativity, and quantum mechanics, which weren't referenced in the film (to the best of my knowledge). --Shaggorama (talk) 14:30, 10 August 2008 (UTC)[reply]
The Matrix, like most high-budget special-effects-filled Hollywood movies, is a travesty against physics. It's not even internally consistent, though I doubt the effects team had a physicist on board to ensure plausibility. Your only recourse is to suspend disbelief.
I can't recall any notable references to modern physics in the movie either, just plenty of special effects which confound good 'ol Newtonian mechanics. -- 98.26.182.245 (talk) 17:53, 10 August 2008 (UTC)[reply]

Animals

Do animals get zits? If they do can they squeeze them? —Preceding unsigned comment added by 86.128.192.207 (talk) 17:15, 10 August 2008 (UTC)[reply]

Umbilical Cord

Do animals have umbilical cords? If so, what do wild animals do with them? Humans usually cut them and tie them up, but of course animals can't do the tying, so what happens? I am just thinking this, because they must have them, otherwise the foetus would not be able to feed inside the mother. I've seen calves getting born, and they just seem to 'fall out', so to speak, totally unattached. Does it just come off inside the mother? If so, why is a human umbilical so long?--ChokinBako (talk) 17:44, 10 August 2008 (UTC)[reply]